You are on page 1of 338

Cracker Book for Bank (IBPS | SBI | RRB PO | Clerk) Mains Exams

1 Adda247 Publications For any detail, mail us at


Publications@adda247.com
Cracker Book for Bank (IBPS | SBI | RRB PO | Clerk) Mains Exams

Chapter

1 Seating Arrangement

BEST APPROACH TO SOLVE THE QUESTIONS

Being a student, you should know about the recent changes that have been
seen in the recent exams i.e SBI PO/ IBPS PO/RRB Clerk/SBI Clerk/RBI
Grade B/Insurance etc. In recent exams, puzzles are tricky but not lengthy
or difficult. So, we are providing you some questions based on the latest
exams and also providing you the detail solution of it. So that you may get to
know how to solve these questions. Here is the approach to solve such
questions.

Example-1:

Directions: Study the information carefully and answer the questions given
below.
(SBI PO Mains-2018)
Certain number of persons are sitting around a circular table, which has a
circumference of 546 cm. All the persons are facing towards the center.
They are sitting at distances to each other which are consecutive multiple of
six. A is 3rd to the left of I. Two persons are sitting between K and I. M is
immediate right to L. H sits to the left of G at a distance of 72 cm. The
distance between A and D is 18 cm. The number of persons sitting between
J and B is same as between B and F. The distance between E and F is LCM of
6 and 5. Neither M nor L is neighbor of K and H. The number of persons
sitting between C and I is same as between I and E. The distance between K
and I is not more than 162cm. Either C or E is neighbor of K.

Explanation (The Approach):


Certain number of persons are sitting around a circular table but not given
the number of persons, but from some other information we can conclude
the number of persons in the circle. As it is given that all persons are sitting

2 Adda247 Publications For any detail, mail us at


Publications@adda247.com
Cracker Book for Bank (IBPS | SBI | RRB PO | Clerk) Mains Exams

at distances to each other which are consecutive multiple of six and


circumference of circle is 546 cm, so between persons distances are 6, 12,
18, 24, 30, 36, 42, 48, 54, 60, 66, 72 and 78. From that condition we deduce
that there are only 13 persons in the circle.
Now we try to solve Puzzle by using some other given information-
A is 3rd to the left of I. Two persons are sitting between K and I. The distance
between A and D is 18 cm. H sits to the left of G at a distance of 72 cm. The
distance between K and I is not more than 162cm. also persons are sitting
at distances to each other which are consecutive multiple of six, so we can
get-

Now from the other conditions-


The distance between E and F is LCM of 6 and 5 that means distance
between E and F is 30. From that Case- 3 is eliminated. Either C or E is
neighbor of K. The number of persons sitting between C and I is same as
between I and E. from that condition Case-2 and Case-4 is eliminated. So
rest Case-1 is left.
3 Adda247 Publications For any detail, mail us at
Publications@adda247.com
Cracker Book for Bank (IBPS | SBI | RRB PO | Clerk) Mains Exams

Neither M nor L is neighbor of K and H. M is immediate right to L. The


number of persons sitting between J and B is same as between B and F. So
the final arrangement are-

Example-2:
Directions: Study the following information carefully and answer the
questions given below.
(SBI Clerk Mains-2018)
There are eight members i.e. A, B, C, D, E, F, G and H are sitting around a
square table such that four of them likes flowers i.e. Lily, Rose, orchid and
Sunflower and four of them likes fruits i.e. Mango, Kiwi, Apple, Banana but
not necessarily in the same order. Those who likes Fruits sits at the corner
and those who like flower sits at the middle of the table. Some of them face
inside and some of them face outside.
The one who likes Orchid sits third to the right of H. A sit second to the right
of the one who likes Orchid. A is not the immediate neighbour of H. The one
4 Adda247 Publications For any detail, mail us at
Publications@adda247.com
Cracker Book for Bank (IBPS | SBI | RRB PO | Clerk) Mains Exams

who likes Banana is an immediate neighbour of A. The one who likes


Banana sits opposite to the one who likes Kiwi. H does not like Kiwi. The
one who likes Apple sits second to the left of the one who likes Kiwi, who is
not the immediate neighbour of A. B sits third to the right of the one who
likes Apple. F likes Lily. Only one person sits between B and the one who
likes Sunflower. F faces the one who likes Sunflower. E and G sits opposite
to each other. E does not like Banana. C sits second to the right of G. C does
not face Inside. C and D face same direction as G.

Explanation (The Approach):


Sol. The one who likes Orchid sits third to the right of H. A sit second to
the right of the one who likes Orchid. A is not the immediate neighbour
of H. The one who likes Banana is an immediate neighbour of A. The
one who likes Banana sits opposite to the one who likes Kiwi. H does
not like Kiwi. Those who likes Fruits sits at the corner and those who
like flower sits at the middle of the table. So we can get that-

From the other conditions, the one who likes Apple sits second to the left of
the one who likes Kiwi, who is not the immediate neighbour of A.

5 Adda247 Publications For any detail, mail us at


Publications@adda247.com
Cracker Book for Bank (IBPS | SBI | RRB PO | Clerk) Mains Exams

Now, B sits third to the right of the one who likes Apple. Only one person
sits between B and the one who likes Sunflower. F faces the one who likes
Sunflower. F likes Lily. From that conditions Case-3 and Case-4 are
eliminated. So, case-1 is continued.

E and G sits opposite to each other. E does not like Banana, So G likes
Banana. C sits second to the right of G. C does not face Inside. C and D face
same direction as G. From that conditions Case-1 is eliminated. Case-2 is
continued.
So, the final arrangement are-
6 Adda247 Publications For any detail, mail us at
Publications@adda247.com
Cracker Book for Bank (IBPS | SBI | RRB PO | Clerk) Mains Exams

Practice Exercise Based on new Pattern

Directions (1-5): Study the following information carefully and answer the
questions that follow:

Seven people having their name as consecutive alphabets, are sitting in


an alphabetical order from west to east direction in a straight line. Four of
them are facing south and remaining of them are facing north. All of them
are of different age. The person whose age is square of four sits second to
the left of K. Two persons sit between the one whose age is square of four
and the one whose age is six years. Both the immediate neighbours of the
one, whose age is six years faces opposite direction to each other (i.e. if one
faces to north then the other faces to south vice versa). L’s age is twice of
the age of the one who sit second left of K. The one whose age is four years
more than the half of L’s age sits second to the right to L. Only one person
sits between M and the one, whose age is 5/4 of L’s age. The persons whose
age is six years and the one whose age is 0.5 times of 18 are immediate
neighbours. The one, whose age is square of five sits third to the right of the
person whose age is four years more than the half of L’s age. L and M face
opposite direction to each other (i.e. If one faces north then the other faces
south vice versa). The one whose age is six years faces south. The person
whose age is 5/4 of L’s age faces north.

7 Adda247 Publications For any detail, mail us at


Publications@adda247.com
Cracker Book for Bank (IBPS | SBI | RRB PO | Clerk) Mains Exams

1. Who among the following is sitting in the row?


(a) G (b) P (c) Q
(d) J (e) H

2. Who among the following person faces north direction?


(a) K (b) J (c) M
(d) Q (e) O

3. Which of the following is the age of M?


(a) 16 (b) 40 (c) 9
(d) 25 (e) None of these

4. If 25 is related to K and 16 is related to M then 32 is related to whom?


(a) L (b) J (c) Q
(d) G (e) None of these

5. What will be the age of the one who sit third to the right of L?
(a) 20 (b) 6 (c) 40
(d) 25 (e) 9

Directions (6-10): Study the following information carefully and answer


the given questions.
P, Q, R, S, T, U, V and W are eight cricketers sitting around a circular table.
Some of them are facing the center while others are facing opposite to the
center. Each of them plays for different team viz. M, K, J, X, N, L, Y and Z but
not necessarily in the same order. Each of them except U and R played
different number of matches for their team. S played two more matches
than his immediate right neighbour. S sits third to the left of P. R sits second
to the left of P. The difference between the number of matches played by Q
and the one who plays for J is two less than the number of person sitting
between them. Q who plays for N is an immediate neighbour of both S and
V. The one who sits to the immediate right of Q plays for X. V who plays for
M is sitting third to the right of P. The sum of the total number of matches
played by the immediate neighbours of P is equal to 9. W who plays for Y
sits third to the right of T. Neither P nor W is an immediate neighbour of the

8 Adda247 Publications For any detail, mail us at


Publications@adda247.com
Cracker Book for Bank (IBPS | SBI | RRB PO | Clerk) Mains Exams

person who plays for L. U does not play for J. Only two such persons are
there who do not have anyone sitting in front of them. The number of
matches played by T is twice the sum of the number of matches played by
his immediate neighbors. The person who plays for Z is sitting opposite T.
The one who plays for N played 5 matches. P and Q are not facing each
other. The one who played for K played 3 matches. No one played more
than 8 matches.

6. Who among the following sits fourth to the right of P?


(a) Q (b) R (c) S
(d) T (e) None of these

7. What is the sum of the total number of matches played by the


immediate neighbours of W?
(a) 11 (b) 8 (c) 7
(d) 9 (e) None of these

8. Who played the least number of matches?


(a) P (b) Q (c) R
(d) T (e) None of these

9. What is the difference between the number of matches played by the


immediate neighbours of U?
(a) 6 (b) 3 (c) 2
(d) 4 (e) None of these
10. Who among the following sits fifth to the right of Q?
(a) P (b) U (c) T
(d) V (e) None of these

Directions (11-15): Study the following information carefully and answer


the questions given below.
Some students are sitting in two rows waiting for their turn to deliver their
speech in the annual function such that all are facing towards north
direction. Some are sitting in row 1 which has more number of seats than
row 2. The student sitting on the left end of the row 1 is the first one to
deliver the speech.
9 Adda247 Publications For any detail, mail us at
Publications@adda247.com
Cracker Book for Bank (IBPS | SBI | RRB PO | Clerk) Mains Exams

Note: The student on the left end of row 1 is at the 1st position and when he
goes to deliver his speech then the student sitting (next) at position 2 will
shift to the first position of the row 1 and all the students will shift in the
same manner. Only three students got the chance to deliver the speech on
the day of the annual function.

All the students will deliver speech according to their seating arrangement
as 1st student seating at left end of row-1 will deliver the speech first then
his immediate neighbour deliver and so on. Total 8 students are sitting in
both rows.

The student sitting at the extreme left end of row 2 is the first student to
shift to row 1 from the extreme right end of row-1. Initially the seating
arrangement of students is based on the given information. Student sitting
on the immediate left of B is the student who is going to deliver the speech
second. B is the last student to deliver the speech on the first day. A and B
does not sit in the same row. No two students sits adjacent to each other
according to the English alphabet(i.e. A does not sit adjacent to B and B does
not sits adjacent to C and A and so on). C does not get the chance to deliver
the speech on the first day of the annual function. C is not the first person to
shift from row 2 to row 1. E is not the first student to deliver the speech nor
he sits in the same row with A. Both F and C sits in the same row and at
least one students sits between them. F is not the first student to deliver the
speech. G is the last student who shift from row 2 to row 1 and is sitting at
extreme right end of row. Only one person sits on the immediate right of A.
D and H does not sit in the same row. H does not deliver the speech on first
day.

11. Who among the following pair sits in the row 2 initially?
(a) C and A (b) B and G (c) G and D
(d) H and G (e) None of these

12. Who among the following is the first to deliver the speech on the
annual day?
(a) D (b) H (c) E
(d) F (e) None of these
10 Adda247 Publications For any detail, mail us at
Publications@adda247.com
Cracker Book for Bank (IBPS | SBI | RRB PO | Clerk) Mains Exams

13. Who among the following student replaces B after the second student
goes to deliver the speech?
(a) H (b) E (c) A
(d) G (e) None of these

14. How many students are there in the first row?


(a) 5 (b) 6 (c) 7
(d) 4 (e) None of these

15. Who among the following sits second to the right of F?


(a) A (b) D (c) E
(d) H (e) None of these

Directions (16-20): Study the following information carefully to answer


the questions given below:

Ten people K, L, M, N, O, P, Q, R, S and T are sitting in a cabin as per the


given formation in the figure such that four of them are facing in west
direction, four are facing east while the other two are facing north direction.

R is sitting second to the right of T. Neither R nor T sits opposite to P. K is an


immediate neighbour of P. N sits opposite to the one who sits immediate
left of L. Only one person sits between P and S. Either three persons sit to
the left of N or no one sits to the right of N. S is not an immediate neighbour
of N and L. P does not face north direction. Q sits to the immediate right of
M. O sits opposite to the one who sits third to the left of N. Neither N nor T
is an immediate neighbour of Q.

11 Adda247 Publications For any detail, mail us at


Publications@adda247.com
Cracker Book for Bank (IBPS | SBI | RRB PO | Clerk) Mains Exams

16. Who among the following sit opposite to O?


(a) L (b) R (c) N
(d) S (e) None of these

17. What is the position of P with respect to N?


(a) Immediate left (b) Second to the left (c) Second to the right
(d) Immediate right (e) None of these

18. How many persons sit between Q and K, counting from the left of K?
(a) One (b) Three (c) Two
(d) No one (e) None of these

19. Which of the following pair faces north direction?


(a) L, O (b) M, Q (c) S, K
(d) R, K (e) None of these

20. How many persons sits between O and T?


(a) Two (b) One (c) Three
(d) No One (e) None of these

Directions (21-25): Study the following information carefully to answer


the questions given below:
Seven boxes B, Y, R, I, O, G, V are placed one above another in a loaded truck.
All these boxes are placed in a linear row from west to east when truck
unloaded. Also after unloading each of the box assigned a unique number.
Initially when boxes are arranged vertically in the truck, the arrangement
is-
More than three boxes are placed between V and R. Only one box is placed
between G and O. Box O is placed above G. Three boxes are placed between
Y and I. Box O is not placed above B and Y. Box V is above box I.
After the unloading of the boxes, Now Only two boxes are placed between
the boxes which was third from the bottom vertically and the box which
was at top. The box which is third from the west end is assigned a number
300
which is % of the number of the box which is two places towards east of
6
it. Only one box is placed between the boxes which was at top and the box
12 Adda247 Publications For any detail, mail us at
Publications@adda247.com
Cracker Book for Bank (IBPS | SBI | RRB PO | Clerk) Mains Exams

which was at bottom. Box I is not placed next to box R. The number
assigned to the box which was at top is 44. The box which was exactly in the
middle is placed adjacent to box R. More than three boxes are placed
between the boxes which was second from the top and box O. The number
assigned to box which is at east end is average of the number assigned to
boxes B, Y and O. Box V is not in the east of box O. Box G is placed
immediate next to box B. Box G is not placed immediate next to the box
which was third from the bottom. The number assigned to box O is 15/11 of
the number assigned to box B. The sum of the number assigned to the boxes
placed at both the ends is 67. The number assigned to the box placed
immediate next to V is 4/7 of the number assigned to it. The number
assigned to box G is twice of the difference of the numbers assigned to box Y
and I.

21. How many boxes are placed between box Y and O when boxes are
placed in vertical manner?
(a) One (b) Three (c) Two
(d) More than three (e) None of these

22. Which of the following box is placed second to the right of box B
horizontally?
(a) V (b) O (c) Y
(d) R (e) None of these

23. What will be the difference of the number assigned to box G and I?
(a) 25 (b) 26 (c) 28
(d) 24 (e) None of these

24. What is the new position of the box which was placed at the bottom?
(a) Second from right end
(b) Immediate left of box G
(c) Third from the left end
(d) Immediate right of box O
(e) None of these

13 Adda247 Publications For any detail, mail us at


Publications@adda247.com
Cracker Book for Bank (IBPS | SBI | RRB PO | Clerk) Mains Exams

25. What is the number assigned to the box which is placed exactly in the
middle horizontally?
(a) 22 (b) 44 (c) 48
(d) 24 (e) None of these

Directions (26-30): Study the information carefully and answers the


question given below.
There are nine-person U V T Z S X W Y and E Sitting in a row where some of
them are facing North and remaining are facing South direction.
Note: Opposite direction means (If one face South then another person
faces north and vice versa). Also, their ages are between 35-65 (means all
nine members age between 35-65, and none of them have same age). W sits
middle of the row, and one person sit between Y and W. E sits second to left
of Y, and face opposite direction with respect of both W and Y. S and T both
are immediate neighbours and one of them sits extreme end of row and
both face same direction with respect of E. Number of persons sit between
T and W and Z and W are same. One person sit between Z and the one
whose age is a perfect square of an even number which is more than 6. U
and V are immediate neighbours and difference between the ages of them is
one year. Either U’s or V’s age is a perfect square of a number which is more
than 5. The ages of both the immediate neighbours of W are perfect
squares. U, who is 48-year-old sits third to left of X. The person who is 55-
year-old sit sixth to right of T. E’s age is 10 years less than his neighbour. Z
faces opposite direction with respect to both U and Y. W age is 3 years more
than V. Difference between ages of Y and T is 5 years. T is older than Y, who
is 2 years younger than Z. Not more than three person faces south direction
and S is youngest amongst Nine. The one whose age is 37 sits at extr8eme
end. E’s age is not 37. There is only one person sits between S and U. There
are only 2 persons sit between the one whose age is 37 and the one who sits
immediate right of W.

26. Which of the following persons sits third to the left of the one whose
age is 64 years?
(a) S (b) T (c) U
(d) V (e) None of these.

14 Adda247 Publications For any detail, mail us at


Publications@adda247.com
Cracker Book for Bank (IBPS | SBI | RRB PO | Clerk) Mains Exams

27. How many persons sit to the right of Y?


(a) One (b) Two (c) Three
(d) None (e) More than Three

28. What is the age of the person who is immediate left of V?


(a) 48 (b) 49 (c) 53
(d) 64 (e) None of these

29. Which group of persons represent whose ages are perfect square?
(a) S T (b) X V (c) X W
(d) V U (e) None of these

30. How many persons sit between W and the person whose age is 45
years?
(a) One (b) Two (c) Three
(d) None (e) More than three

Directions (31-35): Study the following information carefully and answer


the questions that follow:

In the given figure the four-line segments 1,2,3,4 are PA, QB, RC, SD
respectively. The lengths of the line are 50m,35m,24m and 62m
respectively.
Six people Dev, Ram, Ravi, Raju, Sonu and Monu are standing in line 1-PA.
All of them are facing north. With distance between them increasing in
multiples of 9 from the left end (i.e. suppose Dev is left end of the line at
point P then the remaining people will stand at a distance as follows -9m,
18m, 27m…. from the end)

15 Adda247 Publications For any detail, mail us at


Publications@adda247.com
Cracker Book for Bank (IBPS | SBI | RRB PO | Clerk) Mains Exams

Two persons are standing between Dev and Ram. Dev stands to the left of
Ram. Ravi stand second to the left of Ram. Monu is an immediate neighbor
of Ravi. Only one person stands between Monu and Raju. Dev is not an
immediate neighbour of Monu. Raju is neither an immediate neighbour of
Dev nor stand at an extreme end.
Similarly, six people Diya, Riya, Piya, Jiya, Tia and Kiya are standing in line-4
SD. All of them are facing south. with distance between them increasing in
multiples of 11 from the end (i.e. if first person is at right end of the line at
point S and the remaining will be stand at the distance as follows-11m,
22m, 33 m…. from point S).
Piya is standing at 3rd position from the right end. Tia is an immediate
neighbor Of Piya. Two people are standing between Tia and Kiya. More than
three persons stand between Kiya and Diya. Jiya stands to the immediate
left of Kiya. Riya and Tia are immediate neighbours.
* Point Q and Point R is left end of the row in line QB and line RC.

31. In the line-4 SD, what is the distance between Tia and point D?
(a) 29m (b) 16m (c) 22m
(d) 24m (e) None of these

32. If in line-1 PA, if peoples stand at distance of multiples of 11 in the


same order, and rest of them move to line-2 QB and stands from left
end in the same order. And if in line 4-SD, if people stand at distance
of multiples of 13 in the same order and rest of them move to line-3
RC, stands from left end in the same order. Then total together how
many people are standing in lines 2 and 3?
(a) 1 (b) 2 (c) 3
(d) 4 (e) None of these

33. If in line 4-SD, if people stand in multiples of 14 in the same order and
rest of them move to line-3 RC and stands from left end in the same
order. Then the distance between Diya and point C is?
(a) 2 (b)12 (c)24
(d)14 (e) None of these

16 Adda247 Publications For any detail, mail us at


Publications@adda247.com
Cracker Book for Bank (IBPS | SBI | RRB PO | Clerk) Mains Exams

34. Suppose line-1 PA is joined to line2-QB to form a new straight-line PB


in such a way that distance between point A and Q is 5m. If the people
of line-1 PA are made to stand in multiples of 12 in that new line, then
how far is Ram from point B?
(a) 16 (b) 21 (c) 26
(d) 54 (e) None of these

35. If in line-1 PA people stand in distance at multiples of 12 in the same


order and rest of them move to line-2 QB in the same order from left
end, then who will be standing in line-2?
(a) Raju (b) Monu (c) Ram
(d) Ram and Monu (e) None of these

Direction (36-40): Study the following information carefully and answer


the question below-
* Numbers are given in the figure considered as their positions.

Eight persons Ravi, Arjun, Rahul, Ram, Raj, Ayush, Sahil, Shyam are sitting
around the circular table having eight chairs such that some of them are
facing towards the center while some are facing outside the center.
Shyam sits second to the left of Ravi. Only two persons sit between Shyam
and Raj. Raj is not an immediate neighbour of Ravi. Both Raj and Ram are
facing outside the center. Ayush sits on the immediate right to Raj. Sahil sits
third to the right of Arjun. Only Rahul sits between Shyam and Sahil. Both
Arjun and Shyam are facing towards the center. Both Ayush and Rahul are
facing the same direction as Shyam. Sahil is facing opposite direction of
Ravi.
17 Adda247 Publications For any detail, mail us at
Publications@adda247.com
Cracker Book for Bank (IBPS | SBI | RRB PO | Clerk) Mains Exams

Now they start playing cards game. They shuffled a pack of cards. 1. Ravi
draws one card and changes his place according to the given conditions, 2.
Ram draws one card and changes his place according to the given
conditions similarly others draw cards given in the condition below-
Conditions:
1. If the card drawn is spade, the person who draws it first moves to the
outer square at position 5 facing opposite direction of his current
direction, then the second person who also draw spade moves to
position 6, similarly 7 and 8.
2. If card drawn is heart, person who draws it moves to the immediate
left of his current position facing same direction.
3. If the card drawn is diamond, the person who draws it first moves to
the inner square at corner 1 facing same direction, then the second
person, who also draw diamond moves to corner 2, similarly 3 and 4.
4. If card drawn is club, person remains on the same position facing
opposite direction of current direction.
Draws:
1. Ravi draws queen of Club
2. Ram draws Jack of Club
3. Shyam draws 3 of Spade
4. Rahul draws a King of Diamond
5. Sahil draws Ace of Heart
6. Raj draws 9 of Heart
7. Ayush draws 7 of Diamond
8. Arjun draws 4 of Spade
* Note: All the persons draws card in a serial way as given above.
(ex- First Ravi draws then Ram draws ….. so on.)

36. After all persons changed their places based on the above conditions,
who among the following is sitting at the corner 2 of the square?
(a) Arjun (b) Rahul (c) Sahil
(d) Ayush (e) No one

37. After all persons changed their places based on the above conditions,
how many persons will remain in the circle?
(a) Two (b) Four (c) One
(d) Three (e) None of these
18 Adda247 Publications For any detail, mail us at
Publications@adda247.com
Cracker Book for Bank (IBPS | SBI | RRB PO | Clerk) Mains Exams

38. After all persons changed their places based on the above conditions,
then who among the following is sitting to the immediate right of
Sahil?
(a) Raj (b) Ravi (c) Rahul
(d) Shyam (e) None of these

39. After all persons changed their places based on the above conditions,
then how many persons facing away from the center?
(a) Two (b) One (c) Five
(d) Three (e) None of these

40. After all persons changed their places based on the above conditions,
then who among the following is sitting at the position 6 of the
square?
(a) Sahil (b) Ravi (c) Arjun
(d) Shyam (e) None of these

Directions (41-45): Study the following information carefully and answer


the question given below:
Eight students namely A, B, C, D, E, F, G and H study in the school. The Class
teacher asked them to sit in a circle in one arrangement and after lunch in a
straight line in another arrangement. While sitting around a circle each
student are facing outward and while sitting in a straight line each student
are facing north. But in both arrangements each student likes same
cartoons. The cartoons are i.e. Chota bheem, Ben 10, Tom & Jerry, Oggy,
Doraemon, Samurai jack, Johnny Bravo and Generator Rex, but not
necessarily in the same order.
F likes Johnny Bravo. F and C are the immediate neighbours of B in both the
arrangements, but C is not at the extreme ends of the line. The one who
likes Samurai jack sits immediate right of G in the circle. One of the
immediate neighbours of H in the straight line sits opposite to H in the
circle. The one who likes Chota bheem sits third to the right of the one who
likes Johnny Bravo in the Straight line. The one who likes Tom & Jerry sits
second to the left of the one who likes Samurai jack in the straight line. The
19 Adda247 Publications For any detail, mail us at
Publications@adda247.com
Cracker Book for Bank (IBPS | SBI | RRB PO | Clerk) Mains Exams

one who sits on the extreme left end sits second to the right of E in the
circle. H is not on the immediate left of F in both the arrangements. The
Persons who like Chota bheem and Ben 10 sits opposite to each other in the
circle. D sits third to the right of F in the straight line. The one who sits on
the immediate left of B in the straight line is sitting on the immediate right
of B in the circle. G sits on the immediate left of H in the circle, but both are
not immediate neighbours of each other in the straight line. Person likes
Doraemon sits second to the left of the one who likes Oggy in the straight
line. E sits third to the right of B in the circle, while fourth to his left in the
straight line.

41. Which of the following pairs sit at the extreme ends of the row?
(a) The Person who likes Samurai jack, E
(b) F, G
(c) Can’t be determined
(d) G, The Person who likes Chota bheem
(e) None of these

42. The one who is sitting at the extreme left end of the straight line likes
which of the following cartoon?
(a) Tom & Jerry (b) Chota bheem (c) Ben 10
(d) Generator Rex (e) None of these

43. Who among the following sits on the immediate right of E in the
circle?
(a) A (b) G (c) C
(d) D (e) None of these

44. The one who sits third to the right of C in the circle is at what position
in the straight line with respect to G?
(a) Third to the left (b) Third to the right (c) Immediate right
(d) Fourth to the right (e) None of these

20 Adda247 Publications For any detail, mail us at


Publications@adda247.com
Cracker Book for Bank (IBPS | SBI | RRB PO | Clerk) Mains Exams

45. The person who is sitting between E and G in the circle is sitting at
what position in the straight line?
(a) Extreme left end of the straight line
(b) Fourth from the right end
(c) Third from the left end
(d) Extreme right end of the straight line
(e) None of these

Directions (46-50): Study the following information carefully and answer


the questions given below:
Nine people are sitting in a straight line, some are facing south and some
are facing north. Each of them has got married during the years 1972-1980
in the months of Jan to Sept but not necessarily in the same order and only
one person got married in one year and in one month.
C is not the youngest. C got married in a leap year and in a month of 31
days. I is older than B and faces north direction. E got married in Sept. D is
older than H but H is younger than F. C sits 3rd left of H while I sits 2nd
right of D. The persons sitting adjacent to the ones who got married in a
month of 31 days except May but not sitting at extreme ends are facing
opposite directions. Oldest of them sit at an extreme end and got married in
the month which has 31 days. The person who got married in 1977 sits
immediate left of the person, who got married on September. The persons
adjacent to the youngest person face opposite direction. While youngest of
them got married in July sit adjacent to those, who got married in 31 days
month but none of them sit at any extreme position. A sit on the extreme
end. And both the youngest and the oldest face the same direction. A sit 3rd
left of G and face in same direction as G who is not sitting adjacent to I. The
one who got married in Jan is facing south and is sitting exactly in the
middle and his immediate neighbours face opposite directions. I got
married one of the year before B. G got married in 1980. The one who got
married in Aug is facing south and got married in 1974. The ones who got
married in a month having 30 days are sitting adjacent to each other but
none of them is E. E got married in 1978. H who is facing north, got married
in a year between the years of D and A got married and none of them got
married in a leap year. The marriage difference between B and I is 7 years

21 Adda247 Publications For any detail, mail us at


Publications@adda247.com
Cracker Book for Bank (IBPS | SBI | RRB PO | Clerk) Mains Exams

and they got married in May and March respectively. H and D sit adjacent to
each other facing same direction. B and E faces same direction. Persons
sitting adjacent to A face north direction.

46. Who among the following person sits fourth to the right of C?
(a) H (b) A (c) E
(d) I (e) None of these

47. In which of the following year B got married?


(a) 1978 (b) 1980 (c) 1975
(d) 1979 (e) None of these

48. How many persons sit between the person who got married in 1979
and the person who got married in the month of June?
(a) Four (b) Two (c) Three
(d) Can’t be determined (e) Five

49. In which of the following month D got married?


(a) April (b) August (c) June
(d) September (e) Either (a) or (c)

50. Who among the following person got married in the month of August?
(a) Person, who sits immediate left of C
(b) G
(c) Person, who sits immediate left of D
(d) Both (a) and (c)
(e) A

Directions (51-55): Read the following information carefully and answer


the questions given below:
There are six cars – K, L, M, N, O, P – parked in a row facing north direction,
but not necessarily in the same order. The distances between two adjacent
cars are multiples of three.
Information regarding all the cars:
22 Adda247 Publications For any detail, mail us at
Publications@adda247.com
Cracker Book for Bank (IBPS | SBI | RRB PO | Clerk) Mains Exams

The total distance between all the parked cars is 198m.


The distance between ‘N’ and ‘K’ is 105 m and car ‘N’ is second to the left of
car ‘K’.
The distance between the cars ‘N’ and ‘M’ is 132 m.
The car ‘P’ is 84m away from car ‘O’. Car ‘P’ is at one of the position to the
left of car ‘O’.
Only one Car is parked between L and P. P is to the immediate left of K and
distance between them is 36m.
Car L is not in the right of car N and more than two cars are parked between
car N and car O.
Car ‘L’ starts moving towards south and after going 10 m, it turns left, then
it moves 114 m and then it goes another 5 m to its left and stops at point ‘X’.
An another Car ‘C’ moves 15 m in a certain direction, and then takes a left
turn and goes 27 m straight. Then it turns to its right and moves another 5
m and halts at point ‘Y’.
Point Y is in the east of point X and is 36m apart from each other.
Now car ‘N’ starts moving in north direction. After moving 15m it turns to
its right move 105m and stopped at point Z.

51. How many cars are parked there in between cars ‘M’ and ‘L’?
(a) Two (b) None (c) More than three
(d) One (e) Three

52. What is the distance between point ‘Z’ and point ‘Y’?
(a) 25 m
(b) 18 m
(c) They don’t align in the same straight line
(d) 20 m
(e) 27 m

53. What is the distance and direction of final position of Car ‘C’ with
respect to the Car ‘P’?
(a) √1334 m towards north-west
(b) 50 m towards south-east
(c) √1321 m towards south-east
(d) 5√34 m towards south-east
(e) None of these
23 Adda247 Publications For any detail, mail us at
Publications@adda247.com
Cracker Book for Bank (IBPS | SBI | RRB PO | Clerk) Mains Exams

54. Which car will be met first, if ‘L’ moves through the shortest distance
from point ‘X’?
(a) N (b) O (c) K
(d) P (e) None of these

55. What is the current position of car ‘M’ with respect to car ‘O’?
(a) 75 m towards right
(b) 21m to the immediate left
(c) 36 m towards left
(d) 69 m towards left
(e) None of the above

Directions (56-60): Study the following information to answer the given


questions:
Ten persons are sitting in 2 parallel rows containing 5 persons in each row.
In 1st row G, F, H, I and J are seated and face south and in 2nd row E, D, A, B
and C are seated and face north, but not necessarily in the same order.
Therefore, in the given seating arrangement, each member seated in a row
faces another member of the other row. They own different dogs i.e. Great
Dane, Dobermann, Pug, Golden Retriever, German sphered, doodle, Pitbull,
beagle, Labrador, bull dog but not necessarily in the same order.
E sits 2nd left to D, none of them is sitting at the corner. G faces one of the
immediate neighbor of D.H and J sit together and only one of them is
neighbor of G. The one who owns Pitbull sits at one of the corner. Neither D
nor his neighbors face the one who owns Dobermann. B owns pug. Two
people sit between B and the one who owns golden retriever. E does not
own golden retriever. J faces E. The one who owns German sphered sits
opposite to F, who owns Labrador. F does not sit next to the one who owns
Great Dane. The one who owns great Dane does not face north. A does not
face G. Neither I nor his neighbors face the one who owns doodle. The one
who owns Dobermann faces south. I does not own Doodle.

56. Who among the following faces the one who owns Pitbull?
(a) The one who owns Pug
(b) I
(c) D
(d) the one who owns bulldog
(e) none of these
24 Adda247 Publications For any detail, mail us at
Publications@adda247.com
Cracker Book for Bank (IBPS | SBI | RRB PO | Clerk) Mains Exams

57. Who are immediate neighbors of the one who owns Doodle?
(a) E, J (b) B, C (c) C, D
(d) C, A (e) None of these

58. Four of the following five from a group, which among the following
does not belong to this group?
(a) H (b) B (c) C
(d) A (e) F

59. G owns which of the following dogs?


(a) Pug (b) Labrador (c) golden retriever
(d) bulldog (e) none of these

60. How many persons sit to right of the person who owns German
sphered?
(a) one (b) two (c) three
(d) no one (e) four

Directions (61-65): Study the following information to answer the given


questions:
There are eight persons namely P, Q, R, S, T, U, V and W sitting around a
square table in such a way that two persons sit on each side. All the persons
are facing towards the center. They like different cafes in Himachal Pradesh
i.e. Johnson, Evergreen, Cafe 1947, Freedom cafe, Moon dance, Jim Marison,
Moonpeak espresso, Shiva but not necessarily in the same order.
S and P sit on the same side of the table. Neither P nor his neighbors like
Shiva and Moon dance. The one who likes Jim Marison sits 2nd right to S.
Three people sit between the one who likes Jim Marison and W. Neither W
nor his neighbors like Johnson. R does not sit next to the one who like
Johnson. R who likes evergreen faces S, who likes Cafe 1947.T sits opposite
to Q.V sits 3rd right to U. More than three persons sit between the one who
likes Johnson and the one who like Moon dance, when counted in clockwise
direction from of the one who likes Johnson. The one who likes Jim Marison
does not face the ones who like Moonpeak espresso and Johnson. No one
sits between T and the one who likes freedom café.

25 Adda247 Publications For any detail, mail us at


Publications@adda247.com
Cracker Book for Bank (IBPS | SBI | RRB PO | Clerk) Mains Exams

61. Which among the following represents the correct combination?


(a) V-Cafe 1947
(b)T-Johnson
(c) U-Moon dance
(d) P-Moonpeak espresso
(e) none of these

62. Four of the following five form a group, which among the following
does not belongs to that group?
(a) S-R (b) V-P (c) W-U
(d) T-Q (e) S-V

63. What is the position of W with respect to R?


(a) 5th from the left (b) 2nd to the right (c) 3rd to the left
(d) 4th to the right (e) none of these

64. If S and the one who likes Jim Marison exchange their positions, then
who among the following sits 4th right to S?
(a) R (b) P (c) U
(d) T (e) None of these

65. Which the following is not true regarding T?


(a) he likes Shiva
(b) he faces the one who likes Jim Marison
(c) only V sits between T and R.
(d) S sits 3rd left to T
(e) none of these

Directions (66-70): Study the following information to answer the given


questions :
There are eight head of NGO’s namely A, B, C, D, E, F, G and H sitting in a
linear row facing north. All of them distribute free meals to poor on
different festivals i.e. Republic day, Independence Day, Christmas, Diwali,
Lohri, Budh Purnima, Eid-ul-fitr, Mahavir Jayanti but not necessarily in the
same order. No two persons sit next to each other according to the English

26 Adda247 Publications For any detail, mail us at


Publications@adda247.com
Cracker Book for Bank (IBPS | SBI | RRB PO | Clerk) Mains Exams

alphabetical order (i.e. A does not sit adjacent to B and B does not sit next to
A & C and so on).
A sits at one of the ends of the row. More than four persons sit between A
and D. C who is one of the immediate neighbors of H, sits 3rd left to the one
who distributes meal on Mahavir Jayanti. Only one person sits between the
ones who distribute meal on Christmas and Eid-ul-fitr and none of them sit
at the corner. More Than one person sits between the ones distributing
meal on Independence Day and Eid-ul-fitr. The ones who distribute meal on
Diwali and Lohri sit together. Neither A nor C distributes meal on Lohri and
Diwali. H sits 3rd left to B, who does not distribute meal on republic day. A is
not neighbor of H. Odd number of persons sit between H and D. The one
who distributes meal on Budh Purnima sits at extreme left end. There is one
person less sits between A and G as compared to the number of persons
sitting between G and F. The one who distributes meal on Mahavir Jayanti
sits to the immediate left of B. The one who distributes meal on republic
day sits 3rd right to the one who distributes meal on Lohri, who sits
immediate left to the one who distributes meal on Diwali.

66. Who among the following sit to immediate right to the one who
distributes free meal on republic day?
(a) G (b) C (c) D
(d) F (e) none of these

67. Who among the following sit immediate right of D?


(a) C (b) F (c) E
(d) G (e) B

68. How many persons are sitting between the one who distributes on
Eid-al-fitr and F?
(a) no one (b) one (c) two
(d) three (e) more than three

69. Which among the following represents incorrect combination?


(a) C-Eid-ul-fitr
(b) F-christmas
(c) B-mahavir jayanti
(d) H-republic day
(e) none of these
27 Adda247 Publications For any detail, mail us at
Publications@adda247.com
Cracker Book for Bank (IBPS | SBI | RRB PO | Clerk) Mains Exams

70. Which among the following pairs of person represents 2nd right to H
and 4th left to H respectively?
(a) D, A (b) A, D (c) G, C
(d) B, E (e) none of these

Solutions

Directions (1-5):
The person whose age is square of four sits second to the left of K. Two
persons sit between the one whose age is square of four and the one whose
age is six years. The one whose age is six years faces south.

Both the immediate neighbours of the one, whose age is six years faces
opposite direction to each other.

28 Adda247 Publications For any detail, mail us at


Publications@adda247.com
Cracker Book for Bank (IBPS | SBI | RRB PO | Clerk) Mains Exams

As there is only one immediate neighbour of the one whose age is 6 years,
so case-4 will be eliminated. Now, L’s age is twice of the age of the one who
sit second left of K. from this, case 1a, 2a, 3a will be eliminated as it is clear
that L’s age is 32 years.

The one whose age is four years more than the half of L’s age sits second to
the right to L. From this case-1b will be eliminated. Further, Only one
person sits between M and the one, whose age is 5/4 of L’s age.
29 Adda247 Publications For any detail, mail us at
Publications@adda247.com
Cracker Book for Bank (IBPS | SBI | RRB PO | Clerk) Mains Exams

The persons whose age is six years and the one whose age is 0.5 times of 18
are immediate neighbours. The one, whose age is square of five sits third to
the right of the person whose age is four years more than the half of L’s age.
From this case-2b will be eliminated. Now, with case-3b, 5a, 5b------

L and M face opposite direction to each other. The person whose age is 5/4
of L’s age faces north. From this case-5a will be eliminated. Both the
immediate neighbours of the one, whose age is six years faces opposite
direction to each other. So, from this case-5b also get eliminated as only
four persons are facing south. So, the final solution is------------

30 Adda247 Publications For any detail, mail us at


Publications@adda247.com
Cracker Book for Bank (IBPS | SBI | RRB PO | Clerk) Mains Exams

1. (d); 2. (e); 3. (a);

4. (a); 5. (c);

Directions (6-10):
Step 1. From the information given in the question,
S sits third to the left of P. R sits second to the left of P. There can be two
possible cases i.e. case 1 when P is facing towards the center and case 2
when P is facing outside the center. Q who plays for N is an immediate
neighbour of both S and V. It means Q and P are sitting opposite to each
other. P and Q are not facing each other. It means there will three possible
cases.
So we have,

Step 2. The one who sits to the immediate right of Q plays for X. V who
plays for M is sitting third to the right of P. So our case 1 and case 2 will be
eliminated and we will proceed with case 3. in which Q and P are facing
opposite to the center. Only two such persons are there who do not have
anyone sitting in front of them. It means all the other persons except P and
31 Adda247 Publications For any detail, mail us at
Publications@adda247.com
Cracker Book for Bank (IBPS | SBI | RRB PO | Clerk) Mains Exams

Q will be facing towards the center. W who plays for Y sits third to the right
of T. It means T sits to the immediate right of V. The person who plays for Z
is sitting opposite T. It means R plays for Z and U is sitting between T and P.
The one who plays for N played 5 matches. S played two more matches than
his immediate right neighbour.
So we have,

Step 3. Neither P nor W is an immediate neighbour of the person who plays


for L. So, clearly T plays for L. U does not play for J. Means P plays for J and U
plays for K. The difference between the number of matches played by Q and
the one who plays for J is two less than the number of person sitting
between them. The one who played for K played 3 matches. The sum of the
total number of matches played by the immediate neighbours of P is equal
to 9. It means W played 6 matches. U and R are the only two persons who
played the same number of matches. It means R played 3 matches and P
played four matches. The number of matches played by T is twice the sum
of the number of matches played by his immediate neighbors. No one
played more than 8 matches. It means V played 1 match while T played
eight matches.
So we have our final solution as,

32 Adda247 Publications For any detail, mail us at


Publications@adda247.com
Cracker Book for Bank (IBPS | SBI | RRB PO | Clerk) Mains Exams

6. (a); 7. (c); 8. (e);

9. (d); 10. (b);

Solutions (11-15):
Student sitting on the immediate left of B is the student who goes to deliver
the speech second. B is the last student to deliver the speech on the first
day. A and B does not sit in the same row. G is the last students who shift
from row 2 to row 1 and sits at an extreme right end of row-2. Only one
person sits on the immediate right of A. No two students sits adjacent to
each other according to the English alphabet (i.e. A does not sit adjacent to
B and B does not sits adjacent to C and A and so on). So, A sits in row 2.

C does not get the chance to deliver the speech on the first day of the annual
function. C is not the first person to shift from row 2 to row 1. E is not the
first student to deliver the speech nor he sits in the same row with A. Both F
and C sits in the same row and at least one student sit between them. F is
not the first student to deliver the speech. Total 8 students sit in both the
rows.

D and H does not sit in the same row. H does not deliver the speech on first
day.

33 Adda247 Publications For any detail, mail us at


Publications@adda247.com
Cracker Book for Bank (IBPS | SBI | RRB PO | Clerk) Mains Exams

Now, the students shift according to their chance to deliver the speech as it
is given that only three students get the chance to deliver speech so clearly
1st D goes to deliver speech then 2nd F goes and 3rd B goes and accordingly
the shifting takes place as 1st H shifts to row-1 at the right end, 2nd A shifts
at right end as H moves to immediate left of A, 3rd G shifts at the right end as
accordingly. So, the arrangement will be-------

11. (d); 12. (a); 13. (b);

14. (a); 15. (c);

Solutions (16-20):
Either three persons sit to the left of N or no one sits to the right of N. N sits
opposite to the one who sits immediate left of L. O sits opposite to the one
who sits third to the left of N. From this there will be two possible cases-

Only one person sits between P and S. S is not an immediate neighbour of N


and L. K is an immediate neighbour of P. P does not face north direction.
From the given conditions there will be further three possibilities in case2
for the position of P.

34 Adda247 Publications For any detail, mail us at


Publications@adda247.com
Cracker Book for Bank (IBPS | SBI | RRB PO | Clerk) Mains Exams

Q sits to the immediate right of M. From this case-2(c) will be eliminated. R


is sitting second to the right of T. Neither R nor T sits opposite to P.

Neither N nor T is an immediate neighbour of Q. So, from this case-1 and


case-2(a) will be eliminated. So, the final arrangement is—

16. (d); 17. (a); 18. (a);

19. (b) 20. (a);

35 Adda247 Publications For any detail, mail us at


Publications@adda247.com
Cracker Book for Bank (IBPS | SBI | RRB PO | Clerk) Mains Exams

Solutions (21-25):
First we start with vertical arrangement of boxes- More than three boxes
are placed between V and R. Only one box is placed between G and O. Box O
is placed above G. Three boxes are placed between Y and I. Box V is above
box I.

Box O is not placed above B and Y. So, from this Case-2 and Case-3 will be
eliminated. So, the final vertical arrangement before unloading is----

Boxes
Y
V
B
O
I
G
R
Now, after unloading the linear arrangement of the boxes will be as follows-
-- Now Only two boxes are placed between the boxes which was third from
the bottom vertically and the box which was at top. That means two boxes
are placed between box Y and I. Only one box is placed between the boxes
which was at top and the box which was at bottom. That means one box is
placed between box Y and R. Box I is not placed next to box R. The number
assigned to the box which was at top is 44. So, there will be two possible
cases---

36 Adda247 Publications For any detail, mail us at


Publications@adda247.com
Cracker Book for Bank (IBPS | SBI | RRB PO | Clerk) Mains Exams

The box which was exactly in the middle is placed adjacent to box R. That
means Box O is placed adjacent to box R. More than three boxes are placed
between the boxes which was second from the top and box O i.e. more than
three boxes placed between V and O. Box V is not in the east of box O.
So, from this it is clear that box V is placed at west end, box R is at the east
end and rest box O is adjacent to box R. Further Box G is placed immediate
next to box B. Box G is not placed immediate next to the box which was
third from the bottom which means G is not placed next to I. So, the linear
arrangement will be----

The box which is third from the west end is assigned a number which is
300
% of the number of the box which is two places towards east of it. So,
6
clearly number of box B is 22. The number assigned to box O is 15/11 of the
number assigned to box B. Clearly number of box O is 30. The number
assigned to box which is at east end is average of the number assigned to
boxes B, Y and O. So, number of box R is 32. The sum of the number
assigned to the boxes placed at both the ends is 67. So, number of box V is
35. The number assigned to the box placed immediate next to V is 4/7 of
the number assigned to it. So, number of box I is 20. The number assigned
to box G is twice of the difference of the numbers assigned to box Y and I.
So, number of box G is 48. So, the final arrangement is-------

37 Adda247 Publications For any detail, mail us at


Publications@adda247.com
Cracker Book for Bank (IBPS | SBI | RRB PO | Clerk) Mains Exams

21. (c); 22. (c); 23. (c);


24. (d); 25. (c);

Solutions (26-30):

26. (c); 27. (e); 28. (a);

29. (b); 30. (c);

Solutions (31-35):
Step I:Let us start with the detail solution. As given in the diagram that
there is a line-1 PA and Six people Dev, Ram, Ravi, Raju, Sonu and Monu are
standing in line 1-PA. All of them are facing north. With distance between
them increasing in multiples of 9 from the left end (i.e. suppose Dev is left
end of the line at point P then the remaining people will stand at a distance
as follows -9m, 18m, 27m…. from the end). So, the arrangement will be----

In this arrangement the distance between two persons standing next to


each other is nine whereas from first person standing at left end the
distance will be in multiple of 9. And as the total length of the line is 50m
and all persons are standing in the line at a distance of multiple of 9 so it
will be 45m, remaining distance will be 50-45=5m from the right end.
Step II: Now, let us start with the sitting arrangement of line 1-PA. Two
persons are standing between Dev and Ram. Dev stands to the left of Ram.
Ravi stand second to the left of Ram. Monu is an immediate neighbor of
Ravi. Only one person stands between Monu and Raju. Dev is not an
immediate neighbour of Monu. Raju is neither an immediate neighbour of
Dev nor stand at an extreme end. So, the final arrangement of line1-PA is----
----

38 Adda247 Publications For any detail, mail us at


Publications@adda247.com
Cracker Book for Bank (IBPS | SBI | RRB PO | Clerk) Mains Exams

Step IV: Similarly, six people Diya, Riya, Piya, Jiya, Tia and Kiya are standing
in line-4 SD. All of them are facing south. with distance between them
increasing in multiples of 11 from the end (i.e. if first person is at right end
of the line at point S and the remaining will be stand at the distance as
follows-11m, 22m,33 m….from point S)

In this arrangement the distance between two persons standing next to


each other is eleven whereas from first person standing at right end the
distance will be in multiple of 11. And as the total length of the line is 62m
and all persons are standing in the line at a distance of multiple of 11 so it
will be 55m, remaining distance will be 62-55=7m from the left end.
Step V: Piya is standing is at 3rd position from the right end. Tia is an
immediate neighbor Of Piya. Two people are standing between Tia and
Kiya. More than three persons stand between Kiya and Diya. Jiya stands to
the immediate left of Kiya. Riya and Tia are immediate neighbours.

Now, final solution is-----------

31. (a);

32. (b); As only five people can stand in both lines PA and SD at a
distance in multiples of 11m and 13 respectively, So one person
from each line will move to line 2-QB and line 3-RC respectively.
39 Adda247 Publications For any detail, mail us at
Publications@adda247.com
Cracker Book for Bank (IBPS | SBI | RRB PO | Clerk) Mains Exams

33. (c); As only four people can stand in line 4- SD at a distance in


multiples of 14m, So one person from line 4-SD will move to line
3-RC respectively.

34. (d);

40 Adda247 Publications For any detail, mail us at


Publications@adda247.com
Cracker Book for Bank (IBPS | SBI | RRB PO | Clerk) Mains Exams

35. (e);

Solutions (36-40):
From the given condition first we make the sitting arrangement of the
persons—
Step I: Shyam sits second to the left of Ravi. Only two persons sit between
Shyam and Raj. Raj is not an immediate neighbour of Ravi. Both Raj and
Ram are facing outside the center. Ayush sits on the immediate right to Raj.
Sahil sits third to the right of Arjun. Only Rahul sits between Shyam and
Sahil. Both Arjun and Shyam are facing towards the center. Both Ayush and
Rahul are facing the same direction as Shyam. Sahil is facing opposite
direction of Ravi. So, the sitting arrangement is-----

Step II: Now, according to the given conditions - 1. Ravi draws queen of
Club, 2. Ram draws Jack of Club and If card drawn is club, person remains
on the same position facing opposite direction of current direction. So, Ravi
and Ram remains on the same position facing opposite direction of current
direction.

41 Adda247 Publications For any detail, mail us at


Publications@adda247.com
Cracker Book for Bank (IBPS | SBI | RRB PO | Clerk) Mains Exams

Step III: 3.Shyam draws 3 of Spade, If the card drawn is spade, the person
who draws first moves to the outer square at position 5 facing opposite
direction of current direction, then the second person who also draw spade
moves to position 6, similarly 7 and 8. So, it is clear that Shyam will move to
position-5 and will now face outside the center. Further 4. Rahul draws a
King of Diamond, If the card drawn is diamond, the person who draws first
moves to the inner square at corner 1 facing same direction, then the
second person who also draw diamond moves to corner 2, similarly 3 and 4.
So, it is clear that Rahul will move to corner-1 facing same direction.

Step IV:5. Sahil draws Ace of Heart, 6. Raj draws 9 of Heart and If card
drawn is heart, person who draws it moves to the immediate left of his
current position facing same direction. So, both Sahil and Raj moves to the
immediate left of their current position facing same direction.
42 Adda247 Publications For any detail, mail us at
Publications@adda247.com
Cracker Book for Bank (IBPS | SBI | RRB PO | Clerk) Mains Exams

Step V: Further, Ayush draws 7 of Diamond and If the card drawn is


diamond, the person who draws first moves to inner square at corner 1
facing same direction, then the second person who also draw diamond
moves to corner 2, similarly 3 and 4. So, Ayush move to corner 2 of the
inner square. Now, 8. Arjun draws 4 of Spade and If the card drawn is
spade, the person who draws it first moves to the outer square at position 5
facing opposite direction of current direction, then the second person who
also draw spade moves to position 6, similarly 7 and 8. So, Arjun moves to
the position-6 of the outer square.
So, the final arrangement is------------

36. (d); 37. (b); 38. (a);

39. (c); 40. (c);

43 Adda247 Publications For any detail, mail us at


Publications@adda247.com
Cracker Book for Bank (IBPS | SBI | RRB PO | Clerk) Mains Exams

Solutions (41-45):

41. (d); 42. (a); 43. (d);

44. (b); 45. (d);

Directions (46-50):

46. (b); 47. (d); 48. (d);

49. (e); 50. (d);

Solutions (51-55):
The distance between ‘N’ and ‘K’ is 105 m and car ‘N’ is second to the left of
car ‘K’. Only one Car is parked between L and P. P is to the immediate left of
K and distance between them is 36m. The car ‘P’ is 84m away from car ‘O’.
Car ‘P’ is at one of the position to the left of car ‘O’. Car L is not in the right of
car N and more than two cars are parked between car N and car O. The
distance between the cars ‘N’ and ‘M’ is 132 m. The total distance between
all the parked cars is 198m.

44 Adda247 Publications For any detail, mail us at


Publications@adda247.com
Cracker Book for Bank (IBPS | SBI | RRB PO | Clerk) Mains Exams

Car ‘L’ starts moving towards south and after going 10 m, it turns left, then
it moves 114 m and then it goes another 5 m to its left and stops at point ‘X’.
An another Car ‘C’ moves 15 m in a certain direction, and then takes a left
turn and goes 27 m straight. Then it turns to its right and moves another 5
m and halts at point ‘Y’.
Point Y is in the east of point X and is 36m apart from each other.
Now car ‘N’ starts moving in north direction. After moving 15m it turns to
its right move 105m and stopped at point Z.

51. (e); 52. (d); 53. (c);

54. (d); 55. (b);

Solutions (56-60):
Given, “E sits 2nd left to D, none of them is sitting at the corner. G faces one
of the immediate neighbor of D.H and J sit together and only one of them is
neighbor of G.”, J faces E. So, only place left for H and J is 2nd right and
immediate right to G respectively.
B owns pug. Two people sit between B and the one who owns golden
retriever. Either B can sit at extreme left or extreme right end but since E
does not own golden retriever, B sits at left end and D owns golden
retriever.

45 Adda247 Publications For any detail, mail us at


Publications@adda247.com
Cracker Book for Bank (IBPS | SBI | RRB PO | Clerk) Mains Exams

A does not face G. So, A will sit at the extreme right end of the row. Further
The one who owns German sphered sits opposite to F, who owns Labrador.”
Therefore, F faces A, who owns German sphered, we get the following
arrangement:

Given, “The one who owns Pitbull sits at one of the corner.” which means H
owns Pitbull. From, “Neither D nor its neighbor faces Dobermann.” Means
Neither I nor G owns Doberman. The one who owns Dobermann faces
south. So, J owns Dobermann. Given, “F, who owns Labrador and does not
sit next to the one who owns Great Dane. The one who owns great Dane
does not face north.”, we get, that G owns Great Dane.
Given, “Neither I nor its neighbor faces the one who owns doodle.”, I does
not own Doodle. E owns doodle. Rest Either C or I owns Bulldog and beagle.
So, the final arrangement is:

56. (a); 57. (b); 58. (c);

59. (e); 60. (d);


46 Adda247 Publications For any detail, mail us at
Publications@adda247.com
Cracker Book for Bank (IBPS | SBI | RRB PO | Clerk) Mains Exams

Solutions (61-65):
Given, “S and P sit on same side of the table. The one who likes Jim Marison
sits 2nd right to S. Three people sit between the one who likes Jim Marison
and W. R who likes evergreen faces S, who likes Cafe 1947. T sits opposite
to Q.V sits 3rd right to U”, we get the following arrangement:

Given, “Neither P nor its neighbors likes Shiva or Moon dance. More than
three persons sit between the one who likes johnson and the one who like
Moon dance, when counted in clockwise direction from of the one who likes
Johnson. Neither W nor his neighbors like johnson. T R does not sit next to
the one who like Johnson he one who likes Jim marison does not face the
ones who like moonpeak espresso and johnson.”
From these conditions only place left for the ones who likes Johnson that U
likes Johnson, V likes Moon dance, Also, since, neither P nor its neighbor
likes shiva café so T likes shiva café.
Also, No one sits between the one who likes freedom cafe and T. Only
position left, T is immediate left to W, who likes freedom cafe and P likes
moonpeak espresso.

47 Adda247 Publications For any detail, mail us at


Publications@adda247.com
Cracker Book for Bank (IBPS | SBI | RRB PO | Clerk) Mains Exams

61. (d); 62. (e); 63. (a);

64. (e); 65. (d);

Solutions (66-70):
A sit at one of the ends of the row. More than four persons sit between A
and D.H sits 3rd left to B. A is not neighbor of H. Odd number of persons sit
between H and D. we get four possibilities:

C who is one of the immediate neighbors of H, sits 3rd left to the one who
distributes meal on Mahavir Jayanti. The one who distributes meal on
Mahavir Jayanti sits immediate left of B, So, case4 gets eliminated as C
cannot be placed next to D. The one who distributes meal on Budh Purnima
sits at extreme left end, therefore in all the remaining cases A distributes
meal on Budh Purnima.

There is one person less sit between A and G as compared to the number of
persons who are sitting between G and F, as there is no place left for G and F
in case 1 according to this condition, therefore case1 gets eliminated.
48 Adda247 Publications For any detail, mail us at
Publications@adda247.com
Cracker Book for Bank (IBPS | SBI | RRB PO | Clerk) Mains Exams

The ones who distribute meal on Diwali and Lohri sit together. Neither A
nor C distributes on Lohri and Diwali. The one who distributes meal on
republic day sits 3rd right to the one who distributes on Lohri, who sits
immediate left to the one who distributes on Diwali, but given that B does
not distribute meal on republic day so case2 gets eliminated.
Continuing with case 3, one person sit between A and G and two persons
are there in between G and F. So, only place left for E is immediate right to
A. E distributes on Lohri, G on Diwali and H on republic day.
Only one person sits between the ones who distribute meal on Christmas
and Eid-ul-fitr and none of them sit at the corner. So, B distributes meal on
Independence Day. More Than one person sits between the one who
distribute meal on Independence Day and Eid-ul-fitr. So, C distributes on
Eid-al-fitr .
The final arrangement is:

66. (d); 67. (e); 68. (b);

69. (c); 70. (a);

49 Adda247 Publications For any detail, mail us at


Publications@adda247.com
Cracker Book for Bank (IBPS | SBI | RRB PO | Clerk) Mains Exams

 Key Points to Remember for New Pattern Questions


➢ Solving of these questions is similar to a chain process in
which each condition or information in some way or the
other is linked with each other. If you get that one key root
or node of it you will be able to solve it easily.
➢ New concept of undefined number of persons is introduced
in the recent examination. In this type of questions first you
should start adjusting the defined persons in the
arrangement and after going forward with the given
conditions you will be able to find the total number of
persons in that arrangement.
➢ There is also a new pattern introduced of linear
arrangement in the mains exam in which the total length of
the row is given and you have to adjust the given persons in
that row and all the persons are at a given multiple from
each other, so while solving this the factor of distance
should be kept in mind as it plays a major role.
➢ Some times you will see in the exam that the liking of the
persons differ according to their position. For ex- those
who are sitting at the corner like fruit while those who are
sitting in the middle like colours so be careful while reading
the direction also solve the arrangement accordingly.
➢ One new thing that has been seen in the arrangement is the
figure based arrangement in which a figure is given and you
have to place the persons as per the given conditions within
that figure, so the method should be that first we should go
on with the seating arrangement as start placing the
persons around that figure.
➢ A new concept of undefined persons has also been seen in
which you get to know that there are seven or eight
persons and they are in alphabetical order but those
alphabets are not defined so in this type you have to take all
the possible combinations as if K, L and M are given then
you have to take J, K, L, M… or K, L, M, N…. and so on…

50 Adda247 Publications For any detail, mail us at


Publications@adda247.com
Cracker Book for Bank (IBPS | SBI | RRB PO | Clerk) Mains Exams

Adda247 Publications For any detail, mail us at


1
Publications@adda247.com
Cracker Book for Bank (IBPS | SBI | RRB PO | Clerk) Mains Exams

Chapter

2 Puzzle

BEST APPROACH TO SOLVE THE QUESTIONS

Being a student you should know about the recent changes that have been
seen in the recent exams.
So, we are providing you some questions based on the latest exams and also
providing you the detail solution of it. So that you may get to know how to
solve these questions. Here is the approach to solve such questions.

Example-1:

Directions: Study the following information carefully and answer the


questions given below. (IBPS Clerk Mains-2017)
There are five doctors of different types viz., Ophthalmologist, Orthopedic
surgeon, Psychiatrist, General physician and Dentist in a hospital but not
necessarily in same order. They attend their patients at different timing in a
single day. The duration of their meeting with the patient is different. The
total duration of the meeting with the patients by all doctors is of 11 hour.
There is no gap between the meeting time of all doctors. The duration of
meeting either full hour or half an hour but not in one third or one fourth.
The one who is orthopedic surgeon attends their patients from 12:30 to
3pm.The dentist attends their patients immediate before or immediate after
the Orthopedic surgeon. The meeting hour of Dentist with the patients is
three hours. The duration of meeting with patients of General physician is
more than duration of meeting of Psychiatrist with patients by 1 hour.
General physician attends meeting just after the meeting of Psychiatrist.
The general Physician does not attend patients at the last. The
Ophthalmologist attends their patients before the Orthopedic surgeon but
not immediate before. The duration of meeting of Ophthalmologist is of 1.5
hours. The timing of Psychiatrist meeting is after 6:00 am.

Adda247 Publications For any detail, mail us at


2
Publications@adda247.com
Cracker Book for Bank (IBPS | SBI | RRB PO | Clerk) Mains Exams

Solutions: The one who is orthopedic surgeon attends their patients from
12:30 to 3pm.The dentist attends their patients immediate before or
immediate after the Orthopedic surgeon. The meeting hour of Dentist with
the patients is three hours. So, from this we get that----

Case-1:
Doctors Meeting hour
Orthopedic surgeon 12:30- 3:00 pm
Dentist 3:00- 6:00 pm

Case-2:
Doctors Meeting hour
Dentist 9:30am-12:30pm
Orthopedic surgeon 12:30- 3:00 pm

The Ophthalmologist attends their patients before the Orthopedic surgeon


but not immediate before. General physician attends meeting just after the
meeting of Psychiatrist. The duration of meeting of Ophthalmologist is of
1.5 hours. The general Physician does not attend patients at the last. The
duration of meeting with patients of General physician is more than
duration of meeting of Psychiatrist with patients by 1 hour.
Case-1:
Doctors Meeting hour
Ophthalmologist 7:00-8:30am
Psychiatrist 8:30-10:00am
General physician 10:00am-12:30pm
Orthopedic surgeon 12:30- 3:00 pm
Dentist 3:00- 6:00 pm
Case-2:
Doctors Meeting hour
Psychiatrist 4:00-5:30am
General physician 5:30-8:00am
Ophthalmologist 8:00-9:30 am
Dentist 9:30am-12:30pm
Orthopedic surgeon 12:30- 3:00 pm

Adda247 Publications For any detail, mail us at


3
Publications@adda247.com
Cracker Book for Bank (IBPS | SBI | RRB PO | Clerk) Mains Exams

Case-3:
Doctors Meeting hour
Ophthalmologist 4:00-5:30am
Psychiatrist 5:30-7:00am
General physician 7:00-9:30am
Dentist 9:30am-12:30pm
Orthopedic surgeon 12:30- 3:00 pm

The timing of Psychiatrist meeting is after 6:00 am. So, from this case-2 and
3 gets eliminated. So, the final arrangement is----
Doctors Meeting hour
Ophthalmologist 7:00-8:30am
Psychiatrist 8:30-10:00am
General physician 10:00am-12:30pm
Orthopedic surgeon 12:30- 3:00 pm
Dentist 3:00- 6:00 pm

Example-2:

Directions: Study the following information carefully and answer the


questions:
(SBI Clerk Mains-2018)
There are three compartments A, B, C such as compartment A is in west of
compartment B and compartment A and compartment B is in west of
compartment C. Twelve boxes P, Q, R, S, V, X, Y, Z, K, L, M, N are placed in
three different compartments such as four boxes are placed in each
compartment . And these four boxes are placed one above another. Only
one box is placed in between V and Z in compartment B. R is placed on top
in compartment A. Box Y is in the immediate west of L. Box L is placed
between box M and N. M is placed above N. Two boxes are placed between
R and S in same compartment. Box Q is placed immediately above P in the
same compartment. Box X is placed above box V and Z in the same
compartment. Box V is placed above box Z.

Adda247 Publications For any detail, mail us at


4
Publications@adda247.com
Cracker Book for Bank (IBPS | SBI | RRB PO | Clerk) Mains Exams

These boxes are shifting in other compartments as per the cards drawn and
only two cards drawn at a time -
I. If both the card drawn is heart then the box placed at the top in
compartment B will be interchanged with the box placed at the bottom
of Compartment C.

II. If among the card drawn one is diamond and another is spade then the
box which is second from the bottom in Compartment A will be inter
changed with box placed at second from the top in Compartment C.

III. If among the two cards drawn one is Club and another is Heart then the
box placed at top and the box placed at the bottom will be interchanged
in compartment B.

IV. If among the two cards drawn one is club and another is diamond then
the box which is third from the bottom in compartment B is
interchanged with the box which is third from the top in compartment
A.

The Cards Drawn are---


1. Club-Heart
2. Heart-Heart
3. Club-Diamond
4. Spade-Diamond
Note- The cards will be drawn in the given serial order.

Sol. Only one box is placed in between V and Z in compartment B. R is


placed on top in compartment A. Box Y is in the immediate west of L.
Box L is placed between box M and N. M is placed above N. Two boxes
are placed between R and S in same compartment. Box Q is placed
immediately above P in the same compartment. Box X is placed above
box V and Z in the same compartment. Box V is placed above box Z.

Adda247 Publications For any detail, mail us at


5
Publications@adda247.com
Cracker Book for Bank (IBPS | SBI | RRB PO | Clerk) Mains Exams

Compartment Compartment Compartment


A B C
R X K
Q V M
P Y L
S Z N

Step II: According to the given conditions, The cards drawn is- 1. Club-
Heart, for this If among the card drawn one is Club and another is Heart
then the box placed at top and the box placed at the bottom will be
interchanged in compartment B. So, from given conditions, Box X and Z will
be interchanged with each other.

Compartment Compartment Compartment


A B C
R Z K
Q V M
P Y L
S X N

Step III:2. Heart-Heart, If both the card drawn is heart then the box placed
at the top in compartment B will be interchanged with the box placed at the
bottom of Compartment C. So, from given conditions, Box Z and N will be
interchanged with each other.

Compartment Compartment Compartment


A B C
R N K
Q V M
P Y L
S X Z

Adda247 Publications For any detail, mail us at


6
Publications@adda247.com
Cracker Book for Bank (IBPS | SBI | RRB PO | Clerk) Mains Exams

Step IV: 3. Club-Diamond, If among the card drawn one is club and another
is diamond then the box which is third from the bottom in compartment B
is interchanged with the box which is third from the top in compartment A.
So, from the given conditions, Box P and box V will be interchanged.

Compartment Compartment Compartment


A B C
R N K
Q P M
V Y L
S X Z

Step V: 4. Spade-Diamond, If among the card drawn one is diamond and


another is spade then the box which is second from the bottom in
Compartment A will be inter changed with box placed at second from the
top in Compartment C. So, from the given conditions, Box V will be
interchanged with M.

Compartment Compartment Compartment


A B C
R N K
Q P V
M Y L
S X Z

Adda247 Publications For any detail, mail us at


7
Publications@adda247.com
Cracker Book for Bank (IBPS | SBI | RRB PO | Clerk) Mains Exams

Practice Exercise Based on new Pattern

Directions (1-5): Read the given information carefully to answer the


following questions.
There are seven cars viz. A, B, C, D, E, F and G on two parallel roads X and Y.
Four of the given seven cars parked while rest of the three are moving along
the road. The parked cars are facing either north or south direction. The
roads are narrow and are to be considered as a straight line. Distance
between two cars on road X is a multiple of 5, while the distance between
two cars on road Y is a multiple of 3. There are three cars to the right of A.
Distance between B and F is 45 meters. Distance between E and the car in
front of it is two-third of the distance between car E and the car behind it.
Distance between G and A is half of the distance between G and D. Distance
between car C and D is a multiple of 3 but less than 28m. C is 25 meters
ahead of G. B is to the left of G. The moving car on road Y is moving towards
B. Car F is to the North-East of car D. E is moving in a direction opposite to G
and C. Car D is not anywhere behind car G. Only three cars are there on road
Y. F is not behind car G. Distance between G and D is more than 20. No Car is
moving towards F.

1. Which of the following car is facing/moving towards West direction?


(a) A (b) C (c) F
(d) Cannot be determined (e) None of these
2. What is the shortest distance between car F and A?
(a) More than 65 meters (b) 65 meters (c) 57 meters
(d) 55 meters (e) 45 meters

3. Which of the following car is to the left of F?


(a) E (b) B (c) Both B and E
(d) Cannot be determined (e) None of these

4. What is the direction of car E with respect to A?


(a) North East (b) North West (c) South West
(d) South East (e) None of these
Adda247 Publications For any detail, mail us at
8
Publications@adda247.com
Cracker Book for Bank (IBPS | SBI | RRB PO | Clerk) Mains Exams

5. What is the distance between car A and D?


(a) 60 meters (b) 75 meters
(c) 33 meters (d) 55 meters
(e) None of these

Directions (6-10): Study the following information carefully and answer


the given questions.
Seven people A, B, C, D, E, F, G participated in a race which is of a certain
duration, also each of them got different ranks. The track is a linear one and
is of 100m. The one who got first rank run complete 100m but rest all run
different distance but less than 100m in that time. C ran 10 m more than G.
Both A and B run less distance than D. One of the person covers 85m
distance. The one who got II rank is only 5m behind the border line. C
covers less distance than F. The difference between the distance covered by
C and D is 5m. None of the given person runs 90m and nobody covers less
distance than 75m. The difference between the distance covered by C and G
is same as the difference between the distance covered by F and A. F runs
less distance than E. The difference between the sum of the distance
covered by A and D and sum of the distance covered by E and F is 23. B
covers 3m less distance than G.

6. Who among the following got III rank?


(a) D (b) C (c) F
(d) Cannot be determined (e) None of these

7. What is the sum of the distance covered by A and B?


(a) 190 m (b) 165 m (c) 164 m
(d) 155 m (e) None of these

8. What is the difference between the distance covered by D and E?


(a) 15 m (b) 16 m (c) 14 m
(d) 13 m (e) None of these

9. Who got fifth rank?


(a) G (b) A (c) D
(d) C (e) F
Adda247 Publications For any detail, mail us at
9
Publications@adda247.com
Cracker Book for Bank (IBPS | SBI | RRB PO | Clerk) Mains Exams

10. How much distance covered by G?


(a) 92m (b) 79m (c) 95m
(d) 82m (e) None of these

Directions (11-15): Study the following information carefully to answer


the given questions.
Eight football players R, Q, P, O, N, M, S and T have different annual earnings
viz. one million, three million and ten million. Out of these, two are females.
They all live in a building having eighteen floors with top floor being
numbered as 18th , second top floor is number 10th and bottom-most being
numbered as 1st . Only one player lives on each floor. It is given that top
floor and tenth floor is occupied by two of these players, one by each player.
Each of the given eight players likes the following boxers viz. A, B, C, U and
V. Two of them likes A, two likes U, two likes B and one likes C and one likes
V. Q earns three millions and likes U, and her friend N likes A and earns ten
millions. O, M and N lives on consecutive floors. The one who earns three
millions and likes A lives on topmost floor. S lives below the one of the man
who earns three millions. None of the players lives between T and S. M
likes U, and earns the same amount of money as S earns, who likes B. No
two players liking same boxers earns the same amount of money. O lives
above M who live below N. O likes B and earns one million while his friend
T who likes A earns three millions with only one another person. Not more
than three of them earns ten millions. S, R and the one who likes U are living
on consecutive floors respectively. The players who earns one million lives
on the floors which are divisible by three. No one live on fourth and fifth
floor. No female likes C and V. Females players earns either three millions
or ten millions. Neither T nor M is a female. There are only two such pairs of
three persons who live on consecutive floors. The one who earns ten million
lives on the bottom most floor. R lives above the floor of Q.

11. R likes which of the following boxers?


(a) A
(b) V
(c) C
(d) Can’t be determined
(e) None of these

Adda247 Publications For any detail, mail us at


10
Publications@adda247.com
Cracker Book for Bank (IBPS | SBI | RRB PO | Clerk) Mains Exams

12. Which of the following pair represents females?


(a) QS (b) NQ (c) NS
(d) Either (a) or (b) (e) Either (a) or (c)

13. Who among the following earns three millions?


(a) O (b) M (c) S
(d) Q (e) R

14. Which of the following is correctly matched?


(a) P – one million – B (b) Q – ten millions– U
(c) S –three millions – A (d) M –ten millions– U
(e) None of these

15. Who amongst the given eight players stays between Q and P?
(a) M (b) Q (c) S
(d) None of these (e) No one

Directions (16-20): Read the following information carefully and answer


the questions.

In a family 12 members i.e. A, B, C, D, E, F, G, H, I, K, L and J are from 4


different rooms, only 3 members live in each room. Rooms are numbered as
14, 15, 16, 11. Each room is painted with different colours Red, Green, Blue,
Yellow (not necessarily in same order). Each room is situated in different
apartments i.e. Parsavanath, Pragati, Antariksh and Ansal (not necessarily
in same order).

A and G are from same room and their room is an odd numbered room. J’s
room colour is Green. B’s and K’s room colour is red. E and D, are from
different rooms but they are from even numbered room. Yellow and blue
are colours of even numbered room. B neither share the room with E nor
with D. H is from odd numbered room. F and C are not from same room. C is
from blue coloured room. Room number 14 colour is not yellow. C’s room in
Pragati apartment. 11 numbered room is in Antariksh apartment but it is
not red coloured room. D and L share same room in Ansal apartment.

Adda247 Publications For any detail, mail us at


11
Publications@adda247.com
Cracker Book for Bank (IBPS | SBI | RRB PO | Clerk) Mains Exams

16. What is the number of B’s room and name of B’s apartment?
(a) 15, Pragati (b) 16, Ansal (c) 15, Ansal
(d) 15, Parsavanath (e) None of these

17. What is the colour of the room which is situated in Parsavanath


apartment?
(a) Blue (b) Yellow (c) Red
(d) Green (e) None of these

18. Which of the following is a colour of room number 11?


(a) Red (b) Green (c) Blue
(d) Yellow (e) None of these

19. What is the number of E’s room?


(a) 11 (b) 16 (c) 14
(d) 15 (e) None of these

20. Which of the following combination is not true?


(a) A- Green-Antariksh (b) E- Blue-Pragati (c) C- Blue-Ansal
(d) F-Yellow-Ansal (e) J- Green- Antariksh

Direction (21-25): Study the following information carefully and answer


the question given below-

There are seven boxes i.e. A, B, C, D, E, F and G which are kept one above the
other but not necessarily in the same order. They all contains different topic
books i.e. Puzzle, Machine Input (MI), English, Logical, Math’s, Computer,
and Ranking but not necessarily in the same order. All the box contains
different number of books. The total number of books in all the boxes is
192. No two boxes have same number of books.
Three boxes are kept between box F having odd number of books which is a
perfect cube and the box which contains computer book. Box C having 21
books kept immediately below the box having 34 books in it. Only one box
is kept between box A which contains Logical books and the box having
Computer books. Box G has 1 book less than box D. Box containing Machine

Adda247 Publications For any detail, mail us at


12
Publications@adda247.com
Cracker Book for Bank (IBPS | SBI | RRB PO | Clerk) Mains Exams

Input book kept immediately above the box containing English books,
which is 6 less than the number of logical Book. Box A contains even
number of books which is perfect square. Box F has more books than box C.
Box D is kept immediately above box A. Only two boxes are kept between
box D and box E which has total 34 books in it. More than three boxes are
kept between box containing Ranking book and the box containing Puzzle
book, which is kept above Ranking book. Neither box E nor box F contains
Puzzle book. The box containing puzzle books have total 15 books in it. Box
B has 15 books and is kept above box G. Box A has 9 more books than box F.

21. How many boxes are kept above the box B?


(a) One (b) Four (c) More than four
(d) Three (e) None

22. Which of the following box is kept at the bottom?


(a) C – Ranking (b) E – Computer (c) G – Math’s
(d) B – Puzzle (e) None of these

23. How many books does box A contains in it?


(a) 32 (b) 36 (c) 25
(d) 21 (e) None of these

24. Box F contains which of the following book?


(a) Math’s (b) Reasoning (c) Machine Input
(d) Computer (e) None of these

25. How many books does Box G contains?


(a) 21 (b) 29 (c) 30
(d) 27 (e) None of these

Direction (26-30): Study the following information carefully and answer


the question given below-
There are seven boxes i.e. A, B, C, D, E, F and G which are kept one above the
other. The boxes are of different colour i.e. Blue, white, Red, Black, Green,
Pink and Yellow but not necessarily in the same order. The boxes contains
different number of same items i.e. 16, 25, 9, 19, 10, 28, 32 but not
necessarily in the same order. The box are of different sizes. The total size
of all the boxes is 80 sq. cm.
Adda247 Publications For any detail, mail us at
13
Publications@adda247.com
Cracker Book for Bank (IBPS | SBI | RRB PO | Clerk) Mains Exams

Only two boxes are kept between box E, which is of Red colour and the box
which contains 28 items which is 15 sq.cm in length. Box B contains items
which is perfect square number but does not contain even number of items.
Only one box is kept between box F which is 12 sq.cm in size and the box
which is Red in colour. There are three boxes kept between box D which of
Green colour and the box F which is of Blue colour. Box A contains 4 items
less than box C. Only one box is kept between Box C and box D. Box D is kept
below box E but not kept at the bottom. Box C is of yellow colour and is kept
at the bottom. Only one box is kept between box C and box which is of 13
sq.cm in size. G is of Black in colour and is 14 sq.cm in size is Kept
immediately below the box containing 9 items which is 7 sq.cm in size. Box
F contains more item than box D but less items than box G. Box B is 1 Sq.cm
more in size than box C and is not of pink in colour.

26. Which of the following box is white in colour?


(a) Box A (b) Box D (c) Box B
(d) Box F (e) None of these

27. What is the total size of Box C, B and D?


(a) 32 Sq.cm (b) 31 Sq.cm (c) 36 Sq.cm
(d) 34 Sq.cm (e) None of these

28. Which of the following box is kept at the top?


(a) Box B (b) Box F (c) Box E
(d) Box G (e) None of these

29. How many items does box B contains?


(a) 16 (b) 9 (c) 28
(d) 25 (e) None of these

30. Which of the following combination of ‘Box-colour-no. of items’ is


correct?
(a) B- White- 25 (b) A- Pink- 28 (c) D- Green- 10
(d) F- Blue- 16 (e) All are correct
Adda247 Publications For any detail, mail us at
14
Publications@adda247.com
Cracker Book for Bank (IBPS | SBI | RRB PO | Clerk) Mains Exams

Directions (31-35): Study the following information carefully and answer


the question given below.

Six persons A, B, C, P, Q, R attend a seminar on three different months of the


same year on different dates i.e. 15th and 22nd. Months are January, April
and August. Each person attends only one seminar on each day.

B attends the seminar on 15th of the month, but not attends in the month
which has 30 days. Two persons attend seminar between B and B’ son. B’s
father attend seminar in January. Two persons attends seminar between B’s
wife and P, who is in the third generation of the family. B’s wife attends
seminar before B’s son-in-law. P is the daughter of A. Q attends seminar on
15th of the month. Two persons attend seminar between R and B’s son-in-
law. R has only one son.

31. How many persons attend seminar between Q’s wife and B’s wife?
(a) Three (b) Two (c) Four
(d) One (e) None of these

32. Who attends seminar on 22nd April?


(a) A’s daughter (b) R (c) B
(d) Q’s brother-in-law (e) None of these

33. R attends seminar on which of the following date?


(a) 15th April (b) 15th January (c) 22nd August
(d) None of these (e) 22nd January

34. How is Q related to A?


(a) Son-in-law (b) Daughter (c) Father
(d) Daughter-in-law (e) None of these

35. Four of the following five are alike in a certain way and hence they form
a group. Which one of the following does not belong to that group?
(a) B (b) C (c) A
(d) Q (e) R

Adda247 Publications For any detail, mail us at


15
Publications@adda247.com
Cracker Book for Bank (IBPS | SBI | RRB PO | Clerk) Mains Exams

Direction (36-40): Study the following information carefully and answer


the question below-
Seven persons A, B, C, D, E, F, G are going to participate in seven different
races all of which are of different milestones viz. 200, 350, 500, 800, 1000,
1200 and 1500. Each of them got different ranks in their respective races as
1st, 2nd, 3rd…and so on till 7th. Also each of them participated in the race in
different years viz. 1995, 1998, 2000, 2002, 2007, 2010, 2011. The one who
got 1st rank participated in 1200m race. C participated 2 years after A. D
got 4th rank. The milestone of F’s race was 4 times of the milestone of the
who participated in race in 2011. The one who participated first got 3rd
rank. There is a difference of one year between B and G’s participation year.
F got the lowest rank. Only two person got lower rank than G. A’s milestone
of race is twice of the one whose rank is just lower than him. The one who
got 6th rank participated in odd numbered year. The one whose rank is just
higher than G participated in an odd numbered year. The one whose rank is
last participated after the one whose rank is first. The one who participated
in 2007 does not have milestone which is a multiple of the one whose rank
is 3rd. B does not get higher rank than A.

36. Who among the following participated in 1995?


(a) B (b) G (c) A
(d) D (e) E

37. Who among the following got 6th rank?


(a) B (b) G (c) A
(d) D (e) E

38. What is the milestone of C?


(a) 350m (b) 1500m (c) 1000m
(d) 1200m (e) 500m

39. Who among the following participated second in the race?


(a) B (b) G (c) A
(d) D (e) E
Adda247 Publications For any detail, mail us at
16
Publications@adda247.com
Cracker Book for Bank (IBPS | SBI | RRB PO | Clerk) Mains Exams

40. What will be the difference between the milestones of the one who got
3rd rank and the one who got 6th rank?
(a) 300m (b) 700m (c) 1000m
(d) 200m (e) None of these

Directions (41-45): Study the following information carefully to answer


the given questions.
In Chennai, seven persons namely K, L, M, N, O, P and Q are living in an
apartment of a seven storey building. They are living on separate different
floors. The ground floor of the building is numbered 1, the floor above it 2
and so on until the topmost floor is numbered 7. Each person is of different
age. Also Each person likes different cars – BMW, Audi, Acura, Bugatti,
Buick, Desire and Safari, but not necessarily in the same order.
The person who likes Bugatti lives on floor numbered four. The one who
lives immediately above the one who likes Desire Car is 28 years old. P is 33
years old. K does not live on the lowermost floor. K lives on any odd
numbered floor below the one who likes Bugatti. Only two persons live
between K and the person who likes Safari. Only one person lives between L
and P. The difference between the ages of O and Q is 2 years. P lives on an
even numbered floor and does not like Bugatti. Only three persons live
between the persons who like BMW and Acura respectively. M is 9 years
older than K. The person who likes BMW lives on any floor above the L’s
floor. The person who likes BMW does not live on the topmost floor. The
one who lives on the middle floor of this building is 35 years old. Q lives on
an even numbered floor but neither immediately above nor immediately
below the floor of K. The one who likes BMW, is 30 years old but younger
than Q. M does not like BMW or Acura. Only two persons live between N
and the one who likes Bugatti. The person who likes Buick lives on the floor
immediately above the floor of the person who likes Desire. The sum of the
ages of those who live on 1st floor and 2nd floor is 64 years.

41. How many persons live between the floors on which Q and the one who
is 31 years old lives?
(a) Three (b) Two (c) Four
(d) Five (e) No one

Adda247 Publications For any detail, mail us at


17
Publications@adda247.com
Cracker Book for Bank (IBPS | SBI | RRB PO | Clerk) Mains Exams

42. Which of the following statements is/are true according to the given
information?
(a) O lives on floor numbered 5 and he does not like Bugatti
(b) K likes Buick and he does not live on floor numbered 4
(c) M likes Audi and he lives on the topmost floor
(d) Only two persons live between the floors of O and P
(e) All the statements are true.

43. Who among the following lives on the floor immediately above the floor
of the one who is 28 years old?
(a) L (b) P (c) Q
(d) M (e) No one

44. Who among the following lives exactly between the floors on which Q
and L live?
(a) P (b) O (c) N
(d) M (e) No one

45. Who among the following like Acura?


(a) K (b) M (c) L
(d) N (e) No one

Directions (46-50): Study the following information carefully and answer


the question given below:

Eight persons A, B, C, D, E, F, G, H work in different departments of a


company viz. Sales, IT, HR. Not more than three persons work in a single
department. Also each of them like different colours viz. Purple, Yellow,
White, Red, Orange, Pink, Green, Black. C likes green color and work in HR
department. B works in the same department with only one who likes
Purple color. Neither D nor E work in the same department as B. F likes
Orange color. Both A and H work in the same department. The one who
works in HR department like White color. Neither A nor D works in HR
department. The one who works in IT department like Pink color. The one
who works in the same department as G does not like Pink colour. H does
not like Pink color. B neither likes Red nor Black color. H does not like Red
color. D does not like Pink color.
Adda247 Publications For any detail, mail us at
18
Publications@adda247.com
Cracker Book for Bank (IBPS | SBI | RRB PO | Clerk) Mains Exams

46. G works in which of the following department?


(a) IT (b) HR (c) Sales
(d) Either (a) or (b) (e) None of these

47. Who among the following like Black color?


(a) H (b) E (c) D
(d) B (e) None of these

48. Who among the following works in HR department?


(a) H (b) A (c) F
(d) B (e) None of these

49. In which of the following department does exactly two persons are
working?
(a) IT (b) HR (c) Sales
(d) Either (a) or (b) (e) None of these

50. Who among the following likes Red color?


(a) H (b) A (c) D
(d) B (e) None of these

Direction (51-55): Study the following information carefully and answer


the question given below-
Eight persons i.e. A, B, C, D, E, F, G and H are sitting around a circular table
while having dinner. Some of them face inside and some of them face
outside the centre. Each of them is at equal distance from each other. B is
sitting third to the right of A. H sits third to the right of G. D is not the
immediate neighbour of A. H is the immediate neighbour of D. D sits second
to the right of B. E sits second to the right of F, who sits second to the left of
H. C and E faces same direction. The table is arranged in such a way that
two of the given eight persons are facing towards north. D and C faces
opposite direction. C sits second to the left of D. E faces H. After some time,
each of them started leaving the table one by one as one finished his dinner.
F finished first and left the table and walked in the same direction he is
facing to reach point M which is two meters away from his initial position.
After F, E got up from the table, turned 180 degrees and walked four meters
Adda247 Publications For any detail, mail us at
19
Publications@adda247.com
Cracker Book for Bank (IBPS | SBI | RRB PO | Clerk) Mains Exams

to reach point P which is five meters to the south of point Q. Point Q is in the
east of point M. Now, A left the table and walks for 7m in west to reach
point L. After A, H leaves the table and walks 4m in the direction opposite to
in which he was facing initially and reach point S which is 3m north of point
L. At this instant the remaining five persons are still sitting at the initial
positions around the circular table.

51. What is the shortest distance between point P and point S?


(a) 12 meters
(b) 13 meters
(c) 14 meters
(d) More than 16 meters
(e) None of these

52. What is the shortest distance between A and F’ initial position when
they all are sitting around the circular table?
(a) 3 meters
(b) 5 meters
(c) 7 meters
(d) Cannot be determined
(e) None of these

53. In which direction is point L with respect to the E’s Initial position at
the time of dinner?
(a) South
(b) Southwest
(c) Northwest
(d) West
(e) None of these
54. Who among the following is sitting third to the left of C at the instant
when they all are sitting around the circular table?
(a) A (b) E (c) B
(d) F (e) D

55. Who among the following is sitting farthest from point M at the instant
when they all were sitting around the circular table?
(a) A (b) B (c) C
(d) D (e) None of these
Adda247 Publications For any detail, mail us at
20
Publications@adda247.com
Cracker Book for Bank (IBPS | SBI | RRB PO | Clerk) Mains Exams

Direction (56-60): Read the following information carefully and answer


the questions given below:
Eight persons L, M, N, O, P, Q, R, S are working on an eight storey building
where ground floor is numbered as 1 and top floor is numbered as 8. Each
person works on a different floor and also ordered different snacks from
the restaurant viz. Aloo tikki, Masala Vada, Noodles, Pizza, Pakoda, Pav
bhaji, Sweet corn, Samosa. They also drink different varieties of drink from
CCD machine viz. Tea, Coffee, Lemon Tea, and Soup. 3 persons drink tea, 2
persons drink coffee and soup each and 1 person drinks Lemon tea.
There are three floors in between the one who ordered Masala Vada and Q
and both of them work on an odd numbered floor. L works immediate
above O. S drinks Tea. The one who ordered Pakoda also drinks Soup. Both
the persons, who drinks Coffee, works on two consecutive floors. N ordered
Sweet corn and lives below the one who ordered Samosa. P drinks neither
Tea nor Coffee and works on any floor below Q. The one who works on
lowermost floor ordered Pav Bhaji. Only the persons who work on even
number floor drinks Tea. The one who works on 4th floor ordered Pizza but
did not drink Tea. Neither O nor L ordered Pizza. M drinks Soup and both
the persons who work on a floor which is immediately above and
immediately below him drinks Tea. Samosa is ordered with Coffee but not
order by Q. Only one person works in between the floors of the one who
ordered Noodles and L.

56. Who among the following ordered Noodles?


(a) L (b) S (c) P
(d) R (e) Q

57. Which of the following drink is taken with Aloo Tikki?


(a) Lemon Tea
(b)Tea
(c) Either Coffee or Lemon Tea
(d) Soup
(e) Data inadequate
58. R ordered which snack?
(a) Pakoda (b) Pav Bhaji (c) Noodles
(d) Samosa (e) Pizza
Adda247 Publications For any detail, mail us at
21
Publications@adda247.com
Cracker Book for Bank (IBPS | SBI | RRB PO | Clerk) Mains Exams

59. On which of the following floor does O work?


(a) 7th (b) 4th (c) 3rd
(d) 5th (e) 2nd

60. How many persons lives between the one who ordered Pakoda and the
one who ordered Sweet corn?
(a) Five (b) Six (c) Two
(d) Three (e) None
Direction (61-65): Study the following information carefully and answer
the question below-
Six persons A, B, C, D, E and G who all belongs to a family having three
generation and all are sitting around two inscribed squares table such
that all the persons sit at the three corners of each outer and the inner
squares. Some of them face inside and some of them Face outside. F is
the daughter of G. Brother-in- law of G sits second to the right of B and
both faces opposite direction. B is the son-in-law of D and does not sits
in the same square with his daughter. E is the daughter of D, and is not
married to B. There is only one vacant seat in each square. The vacant
seats are not in front of each other. No one sits on the immediate right
of B. Daughter of B sits on the immediate left of G, who faces E. E does
not sit in the same square table with F. D is not the immediate
neighbour of G. Parent of E does not sit in the same square with his son-
in-law. B has no brother. F and A face same direction. F does not face
vacant seat. E and F faces same direction. B and F does not face each
other. A is married to E.

61. How is D related to F?


(a) Maternal Grand-father or Maternal Grandmother
(b) Grandmother (c) Aunt
(d) Father (e) None of these

62. How many persons faces inside?


(a) One (b) Three
(c) Two (d) Four
(e) Can’t be determine

Adda247 Publications For any detail, mail us at


22
Publications@adda247.com
Cracker Book for Bank (IBPS | SBI | RRB PO | Clerk) Mains Exams

63. How is G is related to D?


(a) Son-in-law (b) Son
(c) Daughter-in-law (d) Daughter
(e) None of these

64. Which among the following pair sits in the same square?
(a) B, D (b) A, F
(c) F, E (d) None of these
(e) G, D

65. Who among the following are the immediate neighbours of the vacant
seat?
(a) D, E (b) F, D
(c) A, E (d) E, F
(e) G, D

Directions (66-70): Study the following information carefully and answer


the questions below:

Ten ASEAN countries members (Brunei, Cambodia, Indonesia, Laos,


Malaysia, Myanmar, Philippines, Singapore, Thailand, Vietnam) and its six
FTA partners (Australia, New Zealand (NZ), India, China, Japan and Korea)
are living on different floors in a ten-storey hotel such that 6 of the floors
are doubly occupied. None of the two FTA’s or ASEAN members live on
same floor. Also, they are sitting around a circular table facing the centre in
the following manner as only all the ASEAN members are sitting around the
outer circular table.

Adda247 Publications For any detail, mail us at


23
Publications@adda247.com
Cracker Book for Bank (IBPS | SBI | RRB PO | Clerk) Mains Exams

The arrangement of the persons sitting in the inner circle is irrespective of


the positions of the persons sitting on the outer circle.
The one from the Laos lives on an odd numbered floor and lives just above
the one from India. None of the FTA lives on 10th and 9th floor. Two floors
are there between the one from Vietnam and Philippines. The one from
Australia sits immediate left to the one from New Zealand, who lives above
the one from china and there are three floors between the one from New
Zealand and the one from China. More than three floors are there between
the ones from Laos and Korea, who sits immediate left to the one from
Australia. Three persons sit between the ones from Philippines and
Indonesia (when counted from the left of the one from Philippines). No one
lives with the representatives of Philippines and Myanmar. The one from
Indonesia lives just below the one from Cambodia. Two floors are between
the ones from India and china. The one from the host country and the one
from Thailand live on even numbered floor together. More than two
persons between the ones from Myanmar and Brunei (when counted from
the left of Myanmar.) All the members in both the circles live exactly in the
same sequence as according to the floors they are living in. More than 1
person lives between the ones from Malaysia and Vietnam, who lives above
no one. The one from India does not live on top three floor. The one from
India does not sit next to Korea and the host country representative. The
ones from Malaysia and India live together on same floor. The one from
Myanmar does not lives on the top Floor. The one from Philippines sits
third to the left of the one who is from Vietnam. The one from Singapore
lives on a doubly occupied floor.

66. On which of the following floors does the representative of Singapore


lives?
(a) 4th (b) 2nd
(c) 3rd (d) 7th
(e) None of these

67. Who among the following sits 3rd to the right of the one who is from
Brunei?
(a) Vietnam (b) Laos
(c) Cambodia (d) Singapore
(e) none of these
Adda247 Publications For any detail, mail us at
24
Publications@adda247.com
Cracker Book for Bank (IBPS | SBI | RRB PO | Clerk) Mains Exams

68. Who among the following is the host country?


(a) Australia (b) Japan
(c) Cambodia (d) Singapore
(e) None of these

69. Four of the five are alike in a certain way, who among the following
does not belongs to this group?
(a) Australia (b) Japan
(c) Myanmar (d) India
(e) Brunei

70. Which of the following represents the incorrect combination of persons


living on the same floor?
(a) Australia -Indonesia
(b) Japan-Thailand
(c) Myanmar- New Zealand
(d) India-Malaysia
(e) Vietnam-Korea
Solutions

Solutions (1-5):
Step 1. From the information given in the question,
Only three cars are there on road Y. There are only three moving cars.
Distance between two cars on road X is a multiple of 5, while the distance
between two cars on road Y is a multiple of 3. There are three cars to the
right of A. C is 25 meters ahead of G. It means cars A, C and G lies are on
road X. B is to the left of G. It means car B is on road Y. Distance between E
and the car in front of it is two-third of the distance between car E and the
car behind it. The moving car on road Y is moving towards B. E is moving in
a direction opposite to G and C. It means car E is moving towards car B on
road Y. No Car is moving towards F. Car F is to the North-East of car D. Car D
is not anywhere behind car G. Distance between G and A is half of the
distance between G and D.
There can be two possible cases,
Adda247 Publications For any detail, mail us at
25
Publications@adda247.com
Cracker Book for Bank (IBPS | SBI | RRB PO | Clerk) Mains Exams

Case 1. When G is moving in the West direction.

Case 2. When G is moving in the East direction.

But it is given that car F is not behind car G. So, our case 1 will be
eliminated.
There will be two more possible cases when Car A is facing south direction
but both of the cases will be eliminated by the above given conditions.

Step 2. Proceeding with the remaining information,


Distance between B and F is 45 meters. So the distance between car E and B
will be 18 meters while the distance between E and F will be 27 meters.
Distance between G and A is half of the distance between G and D. Distance
between car C and D is a multiple of 3 but less than 28m. It means the
distance between car C and D is equal to 15 meters. Further the distance
between G and D is more than 20 and distance between C and D is multiple
of 3, so from this it is clear that D is ahead of Car C.

Adda247 Publications For any detail, mail us at


26
Publications@adda247.com
Cracker Book for Bank (IBPS | SBI | RRB PO | Clerk) Mains Exams

So, we have our final solution as,

1. (e); 2. (a); 3. (d); 4. (a); 5. (a);


Solutions (6-10):
The one who got first rank run complete 100m but rest all run different
distance but less than 100m in that time. The one who got II rank is only 5m
behind the border line. Means the one who got II rank runs 95m. The
difference between the distance covered by C and D is 5m. C covers less
distance than F. F runs less distance than E. Both A and B run less distance
than D. Means either F or D got second rank but None of the given person
runs 90m. C run 10 km more than G. So, clearly F got II rank.

Rank Persons Distance


I E 100m
II F 95m

The difference between the distance covered by C and G is same as the


distance covered by F and A. So, clearly A covers 85m. The difference
between the sum of the distance covered by A and D and sum of the
distance covered by E and F is 23. From this we get that D covers 87m.

Rank Persons Distance


I E 100m
II F 95m
D 87m
A 85m

Adda247 Publications For any detail, mail us at


27
Publications@adda247.com
Cracker Book for Bank (IBPS | SBI | RRB PO | Clerk) Mains Exams

The difference between the distance covered by C and D is 5m. C run 10 km


more than G. Nobody covers less distance than 75m. It means C runs 92m
and G runs 82m and also C & D got III and IV rank respectively. B covers 3m
less distance than G. So, the final arrangement is---

Rank Persons Distance


I E 100m
II F 95m
III C 92m
IV D 87m
V A 85m
VI G 82m
VII B 79m

6. (b); 7. (c); 8. (d);


9. (b); 10. (d);

Solutions (11-15):

Floor Players Earnings Boxer


18 T(+) three millions A
10 S(+/-) ten millions B
9 R(+) one million C/V
8 Q(-) three millions U
6 P(+) one million V/C
3 O(+) one million B
2 N(+/-) ten millions A
1 M(+) ten millions U

11. (d); 12. (d); 13. (d)


14. (d); 15. (e);
Adda247 Publications For any detail, mail us at
28
Publications@adda247.com
Cracker Book for Bank (IBPS | SBI | RRB PO | Clerk) Mains Exams

Solutions (16-20):

Room Members Colour Apartment


11 AGJ Green Antariksh
14 ECI Blue Pragati
15 BHK Red Parsavanath
16 DFL Yellow Ansal

16. (d); 17. (c); 18. (b);


19. (c); 20. (c);

Solutions (21-25):
Three boxes are kept between box F and the box which contains computer
book. Only one box is kept between box A which contains Logical books and
the box having Computer books. Box D is kept immediately above box A.

Case 1 Case 2
Box No. of Books Book Topic Box No. of Books Book Topic
F Computer
D D
A Logical A Logical

Computer F

Case 3
Box No. of Books Book Topic
F

Computer
D
A Logical

Adda247 Publications For any detail, mail us at


29
Publications@adda247.com
Cracker Book for Bank (IBPS | SBI | RRB PO | Clerk) Mains Exams

Only two boxes are kept between box D and box E which has total 34 books
in it. Box C having 21 books kept immediately below the box having 34
books in it.

Case 1a Case 1b Case 2


No. of Book No. of Book No. of Book
Box Box Box
Books Topic Books Topic Books Topic
F E 34 E 34
D C 21 C 21
A Logical F Computer
D D
E 34 Computer A Logical A Logical
C 21
Computer F

Case 3
Box No. of Books Book Topic
F

E 34
C 21
Computer
D
A Logical

More than three boxes are kept between box containing Ranking book and
the box containing Puzzle book, which is kept above Ranking book. Neither
box E nor box F contains Puzzle book. The box containing puzzle books
have total 15 books in it. Box B has 15 books and is kept above box G. Now,
it is clear that box B contain puzzle book and has 15 books. So, case 1a, 1b, 2
and case 3 is eliminated.
Adda247 Publications For any detail, mail us at
30
Publications@adda247.com
Cracker Book for Bank (IBPS | SBI | RRB PO | Clerk) Mains Exams

Case 1c
Box No. of Books Book Topic
B 15 Puzzle
F
D
A Logical
G
E 34 Computer
C 21 Ranking

Box G has 1 book less than box D has. Box containing Machine Input book
kept immediately above the box containing English books , which is 6 less
than the number of logical Book. So, case 1a and case 2 is eliminated. Box F
have odd number of books which is a perfect cube. Box A contains even
number of books which is perfect square. Box F has more books than box C.
Box A has 9 more books than box F.

Case 1c
Box No. of Books Book Topic
B 15 Puzzle
F 27 Machine Input
D 30 English
A 36 Logical
G 29 Math’s
E 34 Computer
C 21 Ranking

21. (e); 22. (a); 23. (b);


24. (c); 25. (b);

Adda247 Publications For any detail, mail us at


31
Publications@adda247.com
Cracker Book for Bank (IBPS | SBI | RRB PO | Clerk) Mains Exams

Solutions (26-30):
Only two boxes are kept between box E, which is of Red colour and the box
which contains 28 items which is 15 sq.cm in size. Only one box is kept
between box F which is 12 sq.cm in size and the box which is of Red in
colour.

CASE 1 CASE 2
No. of No. of
Box Sizes(Sq.cm) Colour Box Sizes(Sq.cm) Colour
Items Items
E Red 15 Sq.cm 28
F 12 Sq.cm
F 12 Sq.cm
15 Sq.cm 28 E Red

CASE 3 CASE 4
No. of No. of
Box Sizes(Sq.cm) Colour Box Sizes(Sq.cm) Colour
Items Items
F 12 Sq.cm 15 Sq.cm 28

E Red
E Red

15 Sq.cm 28 F 12 Sq.cm

There are three boxes kept between box D which of Green colour and the
box F which is of Blue colour. Box A contains 4 items less than box C i.e. Box
A contains 28 items and box C contains 32 item. Only one box is kept
between Box C and box D. Box D is kept below box E but not kept at the
bottom. So, case 1 and case 4 is eliminated

Adda247 Publications For any detail, mail us at


32
Publications@adda247.com
Cracker Book for Bank (IBPS | SBI | RRB PO | Clerk) Mains Exams

CASE 2
Box Sizes(Sq.cm) Colour No. of Items
A 15 Sq.cm 28
F 12 Sq.cm

E Red

D Green

CASE 3
Box Sizes(Sq.cm) Colour No. of Items
F 12 Sq.cm Blue

E Red

D Green
A 15 Sq.cm 28

Box C is of yellow colour and is kept at the bottom. Only one box is kept
between box C and D. Only one box is kept between box C and box which is
of 13 sq.cm in size. So, case-2 gets eliminated.

Box Sizes(Sq.cm) Colour No. of Items


F 12 Sq.cm Blue

E Red

D 13Sq.cm Green
A 15 Sq.cm 28
C Yellow 32
G is of Black in colour and is 14 sq.cm in size is Kept immediately below the
box containing 9 items which is 7 sq.cm in size. Box B contains items which
is perfect square number but does not contain even number of items. Box F
contains more item than box D but less items than box G. Box B is 1 Sq.cm
more in size than box C and is not of pink in colour. So, the final
arrangement is----

Adda247 Publications For any detail, mail us at


33
Publications@adda247.com
Cracker Book for Bank (IBPS | SBI | RRB PO | Clerk) Mains Exams

Box Sizes(Sq.cm) Colour No. of Items


F 12 Sq.cm Blue 16
B 10 Sq.cm White 25
E 7 Sq.cm Red 9
G 14 Sq.cm Black 19
D 13 Sq.cm Green 10
A 15 Sq.cm Pink 28
C 9 Sq.cm Yellow 32
Total 80 Sq.cm

26. (c); 27. (a); 28. (b);


29. (d); 30. (e);
Solutions (31-35):

Dates
Months 15th 22nd
January B R
April A C
August Q P

31. (b); 32. (d); 33. (e);


34. (a); 35. (c);
Adda247 Publications For any detail, mail us at
34
Publications@adda247.com
Cracker Book for Bank (IBPS | SBI | RRB PO | Clerk) Mains Exams

Solutions (36-40):
D got 4th rank. F got the lowest rank. Only two person got lower rank than G.
The milestone of F’s race was 4 times of the milestone of the who
participated in race in 2011. There is a difference of one year between B
and G’s participation year. So, from this we will get two possible cases---

Persons Rank Milestone Participation Year


A
B 200/ 2011/2010
C
D 4
E
F 7 800
G 5 200/ 2010/2011

Rank based comparison---


>>>D>G>>F

The one who participated first got 3rd rank means the one who participated
in 1995 got 3rd rank. A’s milestone of race is twice of the one whose rank is
just lower than him. C participated 2 years after A. So, it is clear that A got
2nd rank and his milestone of race is 1000. And also we get that the one who
participated in 1995 got 3rd rank and has milestone 500 of race. The one
who got 1st rank participated in 1200m race. B does not get higher rank
than A. So, we get that C got 1st rank and E got 3rd rank.

Persons Rank Milestone Participation Year


A 2 1000
B 200/ 2011/2010
C 1 1200
D 4
E 3 500 1995
F 7 800
G 5 200/ 2010/2011

Rank based comparison---


C> A > E > D > G > > F

Adda247 Publications For any detail, mail us at


35
Publications@adda247.com
Cracker Book for Bank (IBPS | SBI | RRB PO | Clerk) Mains Exams

Now, we get that B got 6th rank. The one who got 6th rank participated in
odd numbered year. So, B participated in 2011. The one whose rank is just
higher than G participated in an odd numbered year. So, D participated in
2007.

Persons Rank Milestone Participation


Year
A 2 1000
B 6 200 2011
C 1 1200
D 4 2007
E 3 500 1995
F 7 800
G 5 2010

Rank based comparison---


C> A > E > D > G >B > F
C participated 2 years after A. The one whose rank is last participated after
the one whose rank is first. So, clearly C participated in 2000 and A
participated in 1998. Further F participated in 2002. Rest, the one who
participated in 2007 does not have milestone which is a multiple of the one
whose rank is 3rd. So, D has milestone 350 and G has milestone 1500. So, the
final arrangement is---

Persons Rank Milestone Participation Year


A 2 1000 1998
B 6 200 2011
C 1 1200 2000
D 4 350 2007
E 3 500 1995
F 7 800 2002
G 5 1500 2010

Rank based comparison---


C> A > E > D > G >B > F

36. (e); 37. (a); 38. (d);


39. (c); 40. (a);
Adda247 Publications For any detail, mail us at
36
Publications@adda247.com
Cracker Book for Bank (IBPS | SBI | RRB PO | Clerk) Mains Exams

Solutions (41-45):

Floors Person Age Car


7 M 37 Audi
6 Q 32 Safari
5 O 30 BMW
4 L 35 Bugatti
3 K 28 Buick
2 P 33 Desire
1 N 31 Acura

41. (c); 42. (e); 43. (a);


44. (b); 45. (d);

Solutions (46-50):
C likes green color and work in HR department. B works in the same
department with only one who likes Purple color. Neither D nor E work in
the same department as B. F likes Orange color. Both A and H work in the
same department. From this it is clear that G works in same department
with only B and likes Purple color.

Persons Colour Department


A
B IT/Sales
C Green HR
D
E
F Orange
G Purple IT/Sales
H

The one who works in HR department like White color. Neither A nor D
works in HR department. So, from this we get that remaining E works in HR
department and likes White color as both A and H work in same
department.
Adda247 Publications For any detail, mail us at
37
Publications@adda247.com
Cracker Book for Bank (IBPS | SBI | RRB PO | Clerk) Mains Exams

Persons Colour Department


A
B IT/Sales
C Green HR
D
E White HR
F Orange
G Purple IT/Sales
H

The one who works in IT department like Pink color. The one who works in
the same department as G does not like Pink colour. H does not like Pink
color. D does not like Pink color. So, it is clear that A likes Pink color and
works in IT department with H and D. And rest B and G works in Sales
department.

Persons Colour Department


A Pink IT
B Sales
C Green HR
D IT
E White HR
F Orange
G Purple Sales
H IT

Not more than three persons work in a single department. So, rest F works
in HR department. B neither likes Red nor Black color. So, B likes Yellow
color. H does not like Red color. So, H likes Black color and D like Red color.

Adda247 Publications For any detail, mail us at


38
Publications@adda247.com
Cracker Book for Bank (IBPS | SBI | RRB PO | Clerk) Mains Exams

Persons Colour Department


A Pink IT
B Yellow Sales
C Green HR
D Red IT
E White HR
F Orange HR
G Purple Sales
H Black IT

46. (c); 47. (a); 48. (c);


49. (c); 50. (c);

Solutions (51-55):
B is sitting third to the right of A. D is not the immediate neighbour of A. D
sits second to the right of B.
H is the immediate neighbour of D. H sits third to the right of G.

E sits second to the right of F, who sits second to the left of H. So, case 1 and
case 3 are eliminated.
C and E faces same direction. D and C faces opposite direction. C sits second
to the left of D. E faces H. So, case 4 is eliminated

Adda247 Publications For any detail, mail us at


39
Publications@adda247.com
Cracker Book for Bank (IBPS | SBI | RRB PO | Clerk) Mains Exams

The table is arranged in such a way that two of the given eight persons are
facing towards north. So F and A are the only persons who faces opposite
direction and sits opposite to each other. After some time, each of them
started leaving the table one by one as one finished his dinner. F finished
first and left the table and walked in the same direction he is facing to reach
point M which is two meters away from his initial position. After F, E got up
from the table, turned 180 degrees and walked four meters to reach point P
which is five meters to the south of point Q. Point Q is in the east of point M.

Now, A left the table and walks for 7m in west reach point L. After, A H
leaves the table and walks 4m in the direction opposite to in which he was
facing initially and reach point S which is 3m north of point L. So, A walks in
the west direction.

51. (c); 52. (e); 53. (b);


54. (d); 55. (a);
Adda247 Publications For any detail, mail us at
40
Publications@adda247.com
Cracker Book for Bank (IBPS | SBI | RRB PO | Clerk) Mains Exams

Solutions (56-60):
There are three floors in between the one who ordered Masala Vada and Q
and both of them lives on an odd numbered floor. The one who works on
lowermost floor order Pav Bhaji. The one who works on 4th floor ordered
Pizza but did not drink Tea. Only the persons who works on even number
floor drinks Tea. Now, it is clear that the persons live on 2nd, 6th and 8th floor
drinks Tea. So, there can be three possible cases-

Case:1-
Floor Person Drinks Snacks
8 Tea
7 Masala Vada
6 Tea
5
4 Tea(×) Pizza
3 Q
2 Tea
1 Pav Bhaji

Case:2-
Floor Person Drinks Snacks
8 Tea
7 Q
6 Tea
5
4 Tea(×) Pizza
3 Masala Vada
2 Tea
1 Pav Bhaji

Adda247 Publications For any detail, mail us at


41
Publications@adda247.com
Cracker Book for Bank (IBPS | SBI | RRB PO | Clerk) Mains Exams

Case:3-
Floor Person Drinks Snacks
8 Tea
7
6 Tea
5 Masala Vada
4 Tea(×) Pizza
3
2 Tea
1 Q Pav Bhaji

Now, P neither drink Tea nor Coffee and works on any floor below Q. So,
Case: 3 will be eliminated as Q works on floor no.1. M drinks Soup and both
the persons who work on a floor which is immediately above and
immediately below him drinks Tea. From this Case:2 will be eliminated.
Now, L lives immediate above O. Neither O nor L ordered Pizza. So, only
possible condition will be as shown below-

Case:1-
Floor Person Drinks Snacks
8 Tea
7 M Soup Masala Vada
6 L Tea
5 O
4 Tea(×) Pizza
3 Q
2 Tea
1 P Pav Bhaji

Now, Samosa is ordered with Coffee but not order by Q. So, it will be the
person who works on 5th floor who ordered Samosa. Both the persons, who
drinks Coffee, works on two consecutive floors. N ordered Sweet corn and
lives below the one who ordered Samosa. So, N works on 2nd floor. S drinks
Tea so S works on 8th floor and remaining R will work on 4th floor. The one
who ordered Pakoda also drinks Soup.
Adda247 Publications For any detail, mail us at
42
Publications@adda247.com
Cracker Book for Bank (IBPS | SBI | RRB PO | Clerk) Mains Exams

Floor Person Drinks Snacks


8 S Tea
7 M Soup Masala Vada
6 L Tea
5 O Coffee Samosa
4 R Coffee Pizza
3 Q Soup Pakoda
2 N Tea Sweet corn
1 P Pav Bhaji

So, rest P will drink lemon tea. Only one person lives between the one who
ordered Noodles and L. So, the final arrangement will be-----

Floor Person Drinks Snacks


8 S Tea Noodles
7 M Soup Masala Vada
6 L Tea Aloo Tikki
5 O Coffee Samosa
4 R Coffee Pizza
3 Q Soup Pakoda
2 N Tea Sweet corn
1 P Lemon Tea Pav Bhaji

56. (b); 57. (b); 58. (e);


59. (d); 60. (e);

Solutions (61-65):
B is the son-in-law of D. E is the daughter of D, and is not married to B. F is
the daughter of G. Daughter of B sits on the immediate left of G who faces E.
B has no brother. Brother-in- law of G sits second to the right of B. A is
married to E. So, the blood relation tree

Adda247 Publications For any detail, mail us at


43
Publications@adda247.com
Cracker Book for Bank (IBPS | SBI | RRB PO | Clerk) Mains Exams

Brother-in- law of G sits second to the right of B and both faces opposite
direction. No one sits on the immediate right of B.

So, the blood relation tree

There is only one vacant seat in each square. The vacant seats are not in
front of each other. Daughter of B sits on the immediate left of G, who faces
E. E does not sit in the same square with F.

D is not the immediate neighbour of G. F and A face same direction. F does


not face vacant seat. So, case 2 and Case 4 is eliminated.

Adda247 Publications For any detail, mail us at


44
Publications@adda247.com
Cracker Book for Bank (IBPS | SBI | RRB PO | Clerk) Mains Exams

E and F faces same direction. B and F does not face each other. So, case 3
will be eliminated.

61. (a); 62. (e); 63. (d);


64. (e); 65. (e);

Solutions (66-70):
All the members in both the circles live exactly according to the sequence of
the floors they are living in. The one from Philippines sits third to the left of
the one who is from Vietnam. Two floors are there between the ones from
Vietnam and Philippines. The one from Australia sits immediate left to the
one from New Zealand, who lives above the one from china and there are
three floors between the one from New Zealand and the one from China.
The one from Korea sits immediate left to the from Australia. The one from
Vietnam lives above no one. Three persons sit between the ones from
Philippines and Indonesia, (when counted from the left of the one from
Philippines). The one from Indonesia lives just below the one from
Cambodia.

Adda247 Publications For any detail, mail us at


45
Publications@adda247.com
Cracker Book for Bank (IBPS | SBI | RRB PO | Clerk) Mains Exams

Floor ASEAN members FTA


10
9 Cambodia
8 Indonesia
7
6
5
4 Philippines
3
2
1 Vietnam

The one from India does not sit next to Korea and the host country
representative. None of the FTA lives on 10th and 9th floor. No one lives with
the representatives of Philippines and Myanmar. The one from the Laos was
allotted an odd numbered floor and lives just above the one from India.
More than 1 person lives between Malaysia and Vietnam. The one from
Malaysia and India live together on same floor. So, the one from Laos lives
on 7th floor and the one from India lives on 6th floor with the one from
Malaysia. ‘

Adda247 Publications For any detail, mail us at


46
Publications@adda247.com
Cracker Book for Bank (IBPS | SBI | RRB PO | Clerk) Mains Exams

Floor ASEAN FTA


members
10
9 Cambodia
8 Indonesia
7 Laos
6 Malaysia India
5
4 Philippines
3
2
1 Vietnam

Two floors are between the representatives of India and China, Since no
FTA lives on 9 & 10th floor. The one from china lives on 3rd floor. The one
from New Zealand, who lives above the one from china and there are three
floors between the one from New Zealand and the one from China The host
and Thailand live on even numbered floor together. No one lives with the
representatives of Philippines and Myanmar. More than two persons sits
between Myanmar and the one from Brunei(when counted from the left of
the one who is from Myanmar). . The one from Myanmar does not lives on
the top Floor. The one from Singapore lives on a doubly occupied floor.
The final arrangement is:

Adda247 Publications For any detail, mail us at


47
Publications@adda247.com
Cracker Book for Bank (IBPS | SBI | RRB PO | Clerk) Mains Exams

Floor ASEAN members FTA


10 Brunei
9 Cambodia
8 Indonesia Australia
7 Laos New Zealand
6 Malaysia India
5 Myanmar
4 Philippines
3 Singapore China
2 Thailand Japan
1 Vietnam Korea

66. (c); 67. (b); 68. (b);


69. (c); 70. (c);

 Key Points to Remember for New Pattern Questions

➢ Puzzles are not a hard nut to crack. These are given in


exams basically to judge your analytical, logical and
mental ability amalgamate with your intelligence.
➢ Try to solve this as a game that you are playing like a
Jigsaw puzzle. You just have to place all the objects on a
right place and you will find all your answers.
➢ There is modified concept of floor-based arrangement in
which flats are also incorporated in it. This will become
simple if we start following the instructions of puzzle for
the alignment of flats.
➢ The time-based puzzle which is also a new concept consist
of the consecutive timing which has no gap, is totally a
computation based puzzle. You have to adjust the timing
for which you have to start with the defined timing slot.
Adda247 Publications For any detail, mail us at
48
Publications@adda247.com
Cracker Book for Bank (IBPS | SBI | RRB PO | Clerk) Mains Exams

➢ The date and month puzzle is commonly seen topic now a


days. It required basic attention on even/odd date and
also number of days present in the given months. So, Just
go with these points and you will get your answer.
➢ Mathematical condition is a hot topic for puzzles in recent
exams. Most of the puzzles consist of these conditions so
you have to be strong enough in your mathematical
calculation.
➢ Box arrangement with different concepts as uncertain
number of boxes, different number of an item consist by
the box or the linear arrangement of boxes are recently
seen puzzles. These require a bit focus on the number of
boxes above or below the given one.

Adda247 Publications For any detail, mail us at


49
Publications@adda247.com
Cracker Book for Bank (IBPS | SBI | RRB PO | Clerk) Mains Exams

Adda247 Publications For any detail, mail us at


1
Publications@adda247.com
Cracker Book for Bank (IBPS | SBI | RRB PO | Clerk) Mains Exams

Chapter

3 Input-Output

BEST APPROACH TO SOLVE THE QUESTIONS

Recently many changes were seen in the machine Input questions asked in
various mains exams i.e. SBI Po/ IBPS Po/RRB Clerk/SBI Clerk/Insurance
etc. So, below there are some examples based on the new pattern Machine
Input asked in SBI Po Mains and IBPS Po mains 2017 which have been
explained with details solution to help you understand the approach of
these type of questions.

Example 1:
A word and number arrangement machine when given an input line of
words and numbers rearranges them following a particular rule in each
step. The following is an illustration of input and rearrangement. (IBPS Po
Mains 2017)

Input- njes glfu 53 18 31 44 wird dect


Step I: yktf 106 njes glfu 18 31 44 dect
Step II: plgu 88 yktf 106 glfu 18 31 dect
Step III: inhw 62 plgu 88 yktf 106 18 dect
Step IV: fgev 36 inhw 62 plgu 88 yktf 106

In this new pattern machine input question only one word and one number
is arranged in each step.

Let us understand the logic behind it:


We can see that in each step the words and the numbers both are arranged
from the left end.
For words- The word which has highest place value according to
alphabetical series is arranged first from left end and each letter of each
word is replaced by its second succeeding letter according to alphabetical
series and same will be followed in each step.
For numbers- Numbers are arranged in descending order from left end in
such a way that each number is multiplied by two.
Adda247 Publications For any detail, mail us at
2
Publications@adda247.com
Cracker Book for Bank (IBPS | SBI | RRB PO | Clerk) Mains Exams

Example 2:
Study the following diagram and convert it into other diagrams by
implementing the instructions which is given in each step to get next step.
(SBI Po Mains 2017)

Interchange the Alphabets to get step 1 as arrows mention in the above


figure.

For Step-2:
(i) If the alphabets contain one consonant and one vowel and the number
with them is greater than 3, then subtract 3 from the given number.
(ii) If the alphabets are two consonant and the number with them is
greater than 5, then change the letters with the previous letter in
alphabetical series.

Adda247 Publications For any detail, mail us at


3
Publications@adda247.com
Cracker Book for Bank (IBPS | SBI | RRB PO | Clerk) Mains Exams

For Step-3: step 3 is coded in some special pattern.

As, In the above question nothing has mentioned about the step 3. So, we
must understand the logic behind this.

For Step 3:
The elements arranged in the first and third column in such way that the
element in third row placed in first row and first row element is placed in
second row and second row element is placed in third row. The alphabet of
first row second column is replaced with the alphabet which is three places
after the alphabet in alphabetical series. The alphabet of third row second
column is replaced with the alphabet which is five places after the alphabet
in alphabetical series.

Adda247 Publications For any detail, mail us at


4
Publications@adda247.com
Cracker Book for Bank (IBPS | SBI | RRB PO | Clerk) Mains Exams

Practice Exercise Based on new Pattern

Directions (1-5): Study the following diagram and convert it into other
diagrams by implementing the instructions which is given in each step to
get next step.

For Step-1:
Add the number of the last alphabet which is in the alphabetical series with
the element.

For Step-2:
(i) If the alphabets contain one consonant and one vowel -
If the number with them is a whole square, then replace vowel with the
opposite letter in alphabetical series and add 2 in the given number.
If the number with them is not a whole square, then replace vowel with
the second previous letter in the alphabetical series.
(ii) If the alphabets are two consonants –
If the number with them is a whole square, then replace alphabets with
the opposite letter in alphabetical series and subtract 2 in the given
number.
If the number with them is not a whole square, then replace alphabets
with the second next letter in the alphabetical series.

Adda247 Publications For any detail, mail us at


5
Publications@adda247.com
Cracker Book for Bank (IBPS | SBI | RRB PO | Clerk) Mains Exams

For Step-3:
step 3 is coded in some special pattern.

As per the rules followed in the above step, find out the appropriate steps
for the given input.
And answer the following questions.

1. Which of the following element replaces the alphabets SN14 from step-
2 to in step 3?
(a) MJ10 (b) KD20 (c)T O9
(d) XW2 (e) None of these

2. Which of the following element replaces HY25 from step-1 to in step 2?


(a)FZ3 (b)SB23 (c)RT18
(d)GZ26 (e)None of these
Adda247 Publications For any detail, mail us at
6
Publications@adda247.com
Cracker Book for Bank (IBPS | SBI | RRB PO | Clerk) Mains Exams

3. Which of the following element is placed in 2nd column 3rd row in step
3?
(a) DM11 (b) XW2 (c) TC18
(d) RT18 (e) None of these
4. Which of the following is diagonally opposite to the element FZ3 in step
3?
(a) GZ26 (b)JA25 (c)TC18
(d)MJ10 (e)None of these

5. Which of the following replaces CD4 from step-1 to in step 2?


(a) XW2 (b) XN14 (c) MJ10
(d) GZ26 (e) None of these
Directions (6-10): Study the following information to answer the given
questions:
A word and number arrangement machine when given an input line of
words, rearranges them following a particular rule. The following is an
illustration of input and rearrangement.
Input: code word right you thing like
Step I: youv code word right thing like 211
Step II: worde youv code right thing like 211 44
Step III: thingh worde youv code right like 211 44 77
Step IV: rightu thingh worde youv code like 211 44 77 200
Step V: likef rightu thingh worde youv code 211 44 77 200 55
Step VI: codef likef rightu thingh worde youv 211 44 77 200 55 55
Step VI is the last step of the rearrangement.
As per the rules followed in the above steps, find out in each of the
following questions the appropriate steps for the given input.
Input: quite similar dull go test vice

6. How many steps would be needed to complete the arrangement?


(a) X (b) VIII (c) IX
(d) VI (e) None of these

7. What will the difference of the numbers which is third from the right
end in step III and 1st from the right end in step IV?
(a) 12 (b) 0 (c) 18
(d) 11 (e) None of these
Adda247 Publications For any detail, mail us at
7
Publications@adda247.com
Cracker Book for Bank (IBPS | SBI | RRB PO | Clerk) Mains Exams

8. Which of the following would be the cube root of the sum of the
numbers which is 2nd from right end in step IV and 2nd from right end in
Step VI?
(a) 9 (b) 8 (c) 13
(d) 7 (e) None of these

9. Which of the following element will be 6th to the left of 3rd from the
right end in step V?
(a) quitef (b) dull (c) similars
(d) 200 (e) None of these

10. In Step IV, which of the following word/number would be on 4th


position (from the left end)?
(a) vicef (b) 188 (c) 55
(d) quite (e) None of these

Directions (11-15): Study the following information carefully and answer


the given questions.
When a word and number arrangement machine is given an input line of
words and numbers, it arranges them following a particular rule. The
following is an illustration of Input and rearrangement.
Input: camps 59 to 91 concentration 48 including 85 Auschwitz 35.
Step I: Auschwitz 35 camps 59 to 91 concentration 48 including 85.
Step II: Auschwitz 35 camps 91 59 to concentration 48 including 85.
Step III: Auschwitz 35 camps 91 to 48 59 concentration including 85.
Step IV: Auschwitz 35 camps 91 to 48 concentration 85 59 including.
Step V: Auschwitz 35 camps 91 to 48 concentration 85 including 59.

And step V is the last step of the above input. As per the rules followed in
the above step, find out the appropriate step for the given output.
Input: 84 Warsaw has 72 14 already 49 come 83 under sharp 37 from
21.

11. Which element is exactly between the elements which are fourth from
left end and third from right end in step V?
(a) Sharp (b) 72 (c) Under
(d) 37 (e) None of these
Adda247 Publications For any detail, mail us at
8
Publications@adda247.com
Cracker Book for Bank (IBPS | SBI | RRB PO | Clerk) Mains Exams

12. Which step number would be the following output?


“already 21 Warsaw 14 has 72 under 37 sharp 83 from 84 come
49.”
(a) Step VII (b) Step IV (c) Step V
(d) Step VI (e) Step III

13. What is the position of ‘Under’ in the step IV?


(a) Fourth from the right end (b) Sixth from the left end
(c) None of these (d) Fourth from the left end
(e) Eighth from the right end

14. In step VI, what is the sum of 4th element from the left end and 7th
element from the right end?
(a) 56 (b) 51 (c) 48
(d) 54 (e) None of these
15. In which of the following step ’37 84 49’ found in the same order?
(a) Step II (b) Step VII (c) Step IV
(d) Step VI (e) None of these

Direction (16-20): A word and number arrangement machine when given


an input line of words and numbers rearranges them following a particular
rule in each step. The following is an illustration of input and
rearrangement.
Input- utys hltk 37 21 19 48 gmrd fexn
Step I: gdwo 100 utys hltk 37 21 48 gmrd
Step II: hlqe 100 gdwo 100 utys hltk 21 48
Step III: iksl 9 hlqe 100 gdwo 100 utys 48
Step IV: vsxt 144 iksl 9 hlqe 100 gdwo 100
Answer the following questions based on the following input: -
Input- qewb 23 plsf 17 15 cjrx 36 ytvd

16. What will be the difference of the number which is second from the left
end of step I and which is third from the right end of step III?
(a) 52 (b) 28 (c) 0
(d)91 (e)None of these

Adda247 Publications For any detail, mail us at


9
Publications@adda247.com
Cracker Book for Bank (IBPS | SBI | RRB PO | Clerk) Mains Exams

17. What will be the third step of the given input?


(a) Step III- rdvc 36 qkrg 25 diqy 64 32 ytvd
(b) Step III- qkrg 36 25 rdvc diqy 64 36 ytvd
(c) Step III- rdvc 36 qkrg 25 diqy 64 36 ytvd
(d) Step III- qkrg 36 rdvc 25 diqy 64 ytvd 36
(e) None of these

18. Which of the following word/number will be third to the left of sixth
from the left end in step II?
(a) 25 (b) rdvc (c) 64
(d) diqy (e)None of these

19. What will be the twice of the difference of sixth number from the left
end of step I and fifth number from right end of step IV?
(a) 34 (b) 42 (c) 28
(d) 44 (e) None of these

20. In step IV, what is the sum of 2nd element from the left end and 5th
element from the right end?
(a) 107 (b) 89 (c)117
(d) 106 (e) None of these

Directions (21-25): Study the following information carefully and answer


the given questions.
When a word and number arrangement machine is given an input line of
words and numbers, it arranges them following a particular rule. The
following is an illustration of Input and rearrangement.
Input: 18 quora 26 diagnose 89 maths 27 eat
Step I: fkcipqug quora 26 89 maths 27 eat 18
Step II: gcv fkcipqug quora 26 89 maths 18 27
Step III: ocvju gcv fkcipqug quora 89 18 27 26
Step IV: swqtc ocvju fkcipqug 18 27 26 89
And step IV is the last step of the above input. As per the rules followed in
the above step, find out the appropriate step for the given output.
INPUT: Queen 79 apple 38 vowel 19 jungle 26

Adda247 Publications For any detail, mail us at


10
Publications@adda247.com
Cracker Book for Bank (IBPS | SBI | RRB PO | Clerk) Mains Exams

21. Which element is exactly between the elements which are fourth from
left end and third from right end in step IV?
(a) 79 (b) 26 (c) 38
(d) 19 (e) None of these

22. Which step number would be the following output?


“swggp lwping crrng 79 vowel 19 38 26”
(a) Step III (b) Step IV (c) Step I
(d) Step II (e) None of the Above

23. What is the position of ‘vowel’ in the step II?


(a) Seventh from the right end (b) Sixth from the left end
(c) Fourth from the right end (d) Fourth from the left end
(e) None of these

24. In step II, what is the sum of 7th element and 6th element from left end?
(a) 56 (b) 54 (c) 48
(d) 45 (e) None of these

25. Which of the following would be at the third position from the left end
in step IV?
(a) swggp (b) lwping (c) crrng
(d) 19 (e) None of these

Direction (26-30): A word and number arrangement machine when given


an input line of words and numbers rearranges them following a particular
rule in each step. The following is an illustration of input and
rearrangement.
Input- juhs elok 65 17 32 15 pnir veox
Step I: ednc 6511 juhs elok 17 32 15 pnir
Step II: kmhi 325 ednc 6511 juhs elok 17 15
Step III: qtsh 178 kmhi 325 ednc 6511 elok 15
Step IV: donp 156 qtsh 178 kmhi 325 ednc 6511
Answer the following questions based on the following input: -
Input- wder 76 yqok 33 54 isxv 13 zcjf
Adda247 Publications For any detail, mail us at
11
Publications@adda247.com
Cracker Book for Bank (IBPS | SBI | RRB PO | Clerk) Mains Exams

26. What will be the difference of the number which is second from the left
end of step II and which is second from the left end of step III?
(a) 252 (b) 218 (c) 213
(d)191 (e)None of these
27. What will be the step II of the given input?
(a) Step II- bjnp 549 axqu 76 wder 33 isxv 13
(b) Step II- bjnp 549 axqu 7613 wder 33 isxv 13
(c) Step II- 549 bjnp axqu 7613 wder 33 isxv 13
(d) Step II- bjnp 546 axqu 7613 wder 33 isxv 13
(e) None of these
28. Which of the following word/number will be third to the left of sixth
from the left end in step III?
(a) 134 (b) bjnp (c) 336
(d) dwdi (e) None of these

29. What will be the twice of the difference of second number from the left
end of step IV and fifth number from right end of step IV?
(a) 384 (b) 424 (c) 828
(d) 404 (e) None of these
30. In which of the following step ’7613 isxv 13’ found in the same order?
(a) Step II (b) Step II (c) Step IV
(d) Step III (e) None of these
Directions (31-35): Study the following information to answer the given
questions.
A word arrangement machine when given an input line of words rearranges
them by following different operation in each step. The following is an
illustration of input and rearrangement.
Input: Duplicate YouTube Google Jaboong Myntra Strengths
Step I: 13*4 11*4 9%3 10*3 7%1 10*1
Step II: 3 4 1
Step III: es ea eg
Step IV: 6
Step IV is the last step of the arrangement of the above input as the
intended arrangement is obtained.
Now, answer the questions based on the following input.
Input: chamber satellite international domestic ambassador conference

Adda247 Publications For any detail, mail us at


12
Publications@adda247.com
Cracker Book for Bank (IBPS | SBI | RRB PO | Clerk) Mains Exams

31. Which of the following will be the sum of all the digits in step I?
(a) 58 (b) 43 (c) 81
(d) 62 (e) None of these

32. Which of the following is the final step?


(a) 7 (b) 9 (c) 8
(d) 6 (e) None of these

33. If there was one more word “transfer” placed right to conference in the
input then what will be step I ?
(a) 9*2 13*4 18*5 11%3 14%4 14%4 10%2
(b) 9*2 13*4 18*5 11%3 14%3 14%4 10*5
(c) 9*2 13*4 18*5 11%2 14%4 14%4 10%2
(d) 9*2 13*4 19*6 11%3 14%4 14%4 10%2
(e) None of these

34. Which of the following represents the sum of all the digits in step II?
(a) 14 (b) 10 (c) 2
(d) 13 (e) None of these

35. What is the total sum of the numbers before and after the symbol in
step I?
(a) 102 (b) 103 (c) 100
(d) 105 (e) None of these

Directions (36-40): Study the following information carefully and answer


the given questions.
A word and number arrangement machine when given an input line of
words and numbers rearranges them following a particular rule in each
step. The following is an illustration of input and rearrangement.
Input- Draft 95 alliance 67 ideological 58 complex 62
Step-I: revlmlhrdzm draft 95 alliance 58 complex 62 67
Step-II: revlmlhrdzm eszgu alliance 58 complex 62 67 95
Step-III: revlmlhrdzm eszgu dlnqmvy alliance 62 67 95 58
Step-IV: revlmlhrdzm eszgu dlnqmvy zmmrzodv 67 95 58 62
Input- tactics 89 constrained 73 macro 56 hardline 42

Adda247 Publications For any detail, mail us at


13
Publications@adda247.com
Cracker Book for Bank (IBPS | SBI | RRB PO | Clerk) Mains Exams

36. How many steps are required to rearrange the given input?
(a)III (b)VI (c)V
(d)IV (e)None of these

37. What will be the second step of the given input?


(a) Step II- uzdurdt nzdsl constrained 56 hardline 73 42 89
(b) Step II- uzdurdt nzdsl constrained 42 hardline 56 73 89
(c) Step II- uzdurdt nzdsl constrained 56 hardline 42 73 89
(d) Step II- uzdurdt nzdsl constrained 73 56 hardline 42 89
(e) None of these

38. Which of the following word/number will be at fifth position from right
end in III step?
(a) 73 (b) nzdsl (c) 56
(d) constrained (e)None of these

39. What is the addition of 2nd and 5th element from the right end in step II?
(a)130 (b)129 (c)128
(d)121 (e)None of these

40. Which step gives following output?


“uzdurdt nzdsl izsemrov constrained 56 73 89 42”
(a) Step-I (b) Step-II (c) Step-III
(d) Step-IV (e)None of these

Adda247 Publications For any detail, mail us at


14
Publications@adda247.com
Cracker Book for Bank (IBPS | SBI | RRB PO | Clerk) Mains Exams

Solutions

Directions (1-5):

The alphabets are arranged according to the directions are given for step 1.

The alphabets are arranged according to the directions are given for step 2.

For Step-3:
(i) The elements arranged in a serial way in first and third row as element
of third column is placed in first column, element of first column is
placed in second column and element of second column is placed in
third column.
(ii) In the second row the elements are arranged in the given arrangement-

Adda247 Publications For any detail, mail us at


15
Publications@adda247.com
Cracker Book for Bank (IBPS | SBI | RRB PO | Clerk) Mains Exams

The Consonant are replaced with the letter in the alphabetical series with
the next letter of the given letter in alphabetical series. There is a
subtraction of 5 from the given number.

1. (c) 2. (b) 3. (a)


4. (a) 5. (a)

Directions (6-10):
Logic: - There are six words in the input. In each of the step the words are
arranged in reverse alphabetical order such that next letter of the last
letter of that word is also placed with it. Also, in each step a number is
placed at the right end which is the place value of the last letter of the
word which is arranged, with the last digit of that number (place value)
is repeated once in it. For example, Vice (place value of e = 5) so 55 is
placed at the rightmost end.

Input: quite similar dull go test vice


Step I: vicef quite similar dull go test 55
Step II: testu vicef quite similar dull go 55 200
Step III: similars testu vicef quite dull go 55 200 188
Step IV: quitef similars testu vicef dull go 55 200 188 55
Step V: gop quitef similars testu vicef dull 55 200 188 55 155
Step VI: dullm gop quitef similars testu vicef 55 200 188 55 155 122
6. (d); 7. (b); 8. (d);
9. (c); 10. (a);
Adda247 Publications For any detail, mail us at
16
Publications@adda247.com
Cracker Book for Bank (IBPS | SBI | RRB PO | Clerk) Mains Exams

Direction (11-15):
Students let us understand the Logic behind this Question and let’s
understand how to solve it. When we see each step, then we can find that
The machine rearranges one word and one number in each step
simultaneously, words and numbers both are arranged from left end.

(i) Words are arranged in decreasing alphabetical order according to the


last letter of the word and same will be followed in further steps.
(ii) Numbers are arranged in increasing order, according to addition of
their digits. (For example: 19 = 9+1=10).

INPUT: 84 Warsaw has 72 14 already 49 come 83 under sharp 37 from


21.
Step I: already 21 84 Warsaw has 72 14 49 come 83 under sharp 37
from.
Step II: already 21 Warsaw 14 84 has 72 49 come 83 under sharp 37
from.
Step III: already 21 Warsaw 14 has 72 84 49 come 83 under sharp 37
from.
Step IV: already 21 Warsaw 14 has 72 under 37 84 49 come 83 sharp from.
Step V: already 21 Warsaw 14 has 72 under 37 sharp 83 84 49 come from.
Step VI: already 21 Warsaw 14 has 72 under 37 sharp 83 from 84 49 come.
Step VII: already 21 Warsaw 14 has 72 under 37 sharp 83 from 84 come 49.

11. (d); 12. (a); 13. (e);


14. (b); 15. (c);

Direction (16-20):
In the given Input-Output one word and one number is being arranged
simultaneously in each step.
For Words- In first step- The word which comes first according to
alphabetical series will be arranged from left end such that its first and last
letter of the word will be replaced with its next letter and rest of the letters
are replaced with their preceding letter. Then the next word according to
alphabetical order will be arranged in second step from the left end and so
on………..

Adda247 Publications For any detail, mail us at


17
Publications@adda247.com
Cracker Book for Bank (IBPS | SBI | RRB PO | Clerk) Mains Exams

For numbers- All the numbers are arranged with the one word
simultaneously in each step, such that first the prime numbers are arranged
in ascending order and then non-prime numbers will be arranged. Each of
the number will be arranged by following logic as number will be replaced
with the square of the addition of the digits of that number after
arrangement.
Input- qewb 23 plsf 17 15 cjrx 36 ytvd
Step I: diqy 64 qewb 23 plsf 15 36 ytvd
Step II: qkrg 25 diqy 64 qewb 15 36 ytvd
Step III: rdvc 36 qkrg 25 diqy 64 36 ytvd
Step IV: zsue 81 rdvc 36 qkrg 25 diqy 64

16. (c); 17. (c); 18. (d);


19. (b); 20. (c);

Direction (21-25):
Students let us understand the Logic behind this Question and let’s
understand how to solve it. When we see each step, then we can find that
The machine rearranges one word and one number in each step
simultaneously, words are arranged from left end and numbers are
arranged from right end.

(i) Words are arranged in increasing alphabetical order with each letter of
word is replaced by its 2nd succeeding letter according to alphabetical
series and same will be followed in further steps.
(ii) Numbers are arranged in decreasing order, according to difference of
their digits. (For example: 19 = 9-1=8).

INPUT: Queen 79 apple 38 vowel 19 jungle 26


Step I: crrng queen 79 38 vowel jungle 26 19
Step II: lwping crrng queen 79 vowel 26 19 38
Step III: swggp lwping crrng 79 vowel 19 38 26
Step IV: xqygn swggp lwping crrng 19 38 26 79

21. (d); 22. (a); 23. (c);


24. (d); 25. (b);
Adda247 Publications For any detail, mail us at
18
Publications@adda247.com
Cracker Book for Bank (IBPS | SBI | RRB PO | Clerk) Mains Exams

Direction (26-30):
For words- All the words are arranged in descending order in alphabetical
series. The logic of the arrangement of words is that the word which is last
according to the alphabetical series is arranged first from the left end in
step I, then the word which is second last according to the alphabetical
order is arranged from the left end in step II and so on….Also in the words
each of the consonant is replaced by its opposite letter and each vowel is
replaced by its previous letter while arrangement.
For Numbers-Only One number is arranged along with one word in each
step. The logic of the arrangement of numbers is that the numbers are
arranged in decreasing order as the highest number is arranged in step I
with the word then second highest number is arranged in step II with the
next word and so on... Each number is arranged along with one word in
such a way that the sum of the digits of that number is placed next to it.
Input- wder 76 yqok 33 54 isxv 13 zcjf
Step I: axqu 7613 wder yqok 33 54 isxv 13
Step II: bjnp 549 axqu 7613 wder 33 isxv 13
Step III: dwdi 336 bjnp 549 axqu 7613 isxv 13
Step IV: hhce 134 dwdi 336 bjnp 549 axqu 7613

26. (c); 27. (b); 28. (b);


29. (d); 30. (d);

Directions (31-35):
In the given machine input, different operation are performed in different
steps:
In step I-

(a) the number before the symbol- Total number of letters in the word +
number of vowels in the word.
(b) the number after the symbol- Total number of vowels in the word.
(c) Symbol- if the resultant of number in {(a) +(b)} is even then % and if it
is odd then ‘*’.
Adda247 Publications For any detail, mail us at
19
Publications@adda247.com
Cracker Book for Bank (IBPS | SBI | RRB PO | Clerk) Mains Exams

In step II. The difference between the first numbers of the words according
to the following pattern.

Step III. The last letters of the words combinedly used in the step II.

Step IV. Adding the place value of all the letters obtained in step III and
then adding the digits obtained after the sum .
Es Ea Eg ---- 5+19+5+1+5+7=42=6

INPUT: chamber satellite international domestic ambassador conference


Step I: 9*2 13*4 19*6 11%3 14%4 14%4
Step II: 5 1 8
Step III: re er lc
Step IV: 7
31. (a); 32. (a); 33. (d);
34. (a); 35. (b);

Directions (36-40):
In this new pattern Input-Output question only one word and one number
is arranged in each step. Let us understand the logic behind it- In each step
the words are arranged from the left end such that in 1st step word are
arranged in extreme end than 2nd word are arranged in 2nd left end and so
on… while the numbers are arranged from the right end.

Adda247 Publications For any detail, mail us at


20
Publications@adda247.com
Cracker Book for Bank (IBPS | SBI | RRB PO | Clerk) Mains Exams

For words- words are arranged according to reverse alphabetical order


(English dictionary) from left end with each vowel of word is replaced by its
opposite letter according to alphabetical series while each consonant of
word is replaced by its succeeding letter according to alphabetical series
and same will be followed in further steps.
For numbers- Numbers are arranged in ascending order from right end in
such a way that first all odd numbers are arranged after that all even
numbers are arranged.
Input- tactics 89 constrained 73 macro 56 hardline 42
Step-I: uzdurdt 89 constrained macro 56 hardline 42 73
Step-II: uzdurdt nzdsl constrained 56 hardline 42 73 89
Step-III: uzdurdt nzdsl izsemrov constrained 56 73 89 42
Step-IV: uzdurdt nzdsl izsemrov dlotuszrove 73 89 42 56
36. (d); 37. (c); 38. (d);
39. (b); 40. (c);



Key Points to Remember for New Pattern Questions
In your daily life you are required to put some efforts, then some
processing has to be done so that output will get received.
Similarly, In Input-Output, Input that has been given, processed
by a particular logic within some steps and then finally you will
get the output.
➢ The machine input-output is keep on changing recently, as it
includes the logic related to place values of the letters of the
word or it may some numbers and there will be some
mathematical operations applied on it.
➢ In some recent exam the new pattern that has been seen
includes a diagram having some elements. Some operations are
applied on it in a stepwise manner and then the diagram
changed accordingly.
➢ In some of the input-output questions, now a days there can be
seen some different logics which includes omission and addition
of letters in the words.

Adda247 Publications For any detail, mail us at


21
Publications@adda247.com
Cracker Book for Bank (IBPS | SBI | RRB PO | Clerk) Mains Exams

1 Adda247 Publications For any detail, mail us at


Publications@adda247.com
Cracker Book for Bank (IBPS | SBI | RRB PO | Clerk) Mains Exams

Chapter

4 Data Sufficiency

BEST APPROACH TO SOLVE THE QUESTIONS

Data Sufficiency, a topic that has emerged as mostly based on seating and
puzzle arrangement recently. It requires attention while solving as each and
every statement contains some direct and indirect information in it. So, you
are required to solve every statement individually and also by combining it
with other one.

Example-1:
Each of the questions below consists of a questions and two statements
numbered I and II given below it. You have to decide whether the data
provided in the statements are sufficient to answer the question. Read both
the statements and find;
(IBPS PO Mains 2017)
(a) if the data in statement I alone are sufficient to answer the question,
while the data in statement II alone are not sufficient to answer the
question.
(b) if the data in statement II alone are sufficient to answer the question,
while the data in statement I alone are not sufficient to answer the
question.
(c) if the data either in statement I alone or in statement II alone are
sufficient to answer the question.
(d) if the data even in both statements I and II together are not sufficient to
answer the question.
(e) if the data in both statements I and II together are necessary to answer
the question.

2 Adda247 Publications For any detail, mail us at


Publications@adda247.com
Cracker Book for Bank (IBPS | SBI | RRB PO | Clerk) Mains Exams

1. Five friends Q, R, S, T, V are married on different dates 12, 19, 21, 28, 31
but not necessarily in the same order in the month of either January or
September. T is married on which of the following date?
(I) Both T and Q are not married on even date of the month. T is
married before Q. Two persons are married in between T and V.
(II) No body is married after R. S is married on a date in between T and
Q. S is married on an even date. More than three persons are married
in the same month.

2. Six boxes A, B, C, D, E, F of different colours are placed one above


another. Also each box have different number of toffees. Only two boxes
are placed in between B and Green box. No box is placed above B. Box D
is placed immediately above Blue box. Only Red box is placed in between
Green box and A. Only one box is placed between Red and Blue box. Only
one box is placed in between D and E. Only one box is placed between
Orange box and C. How many number of toffees does Blue box have?
(I) Box E has more number of toffees than 8 while box C has more
number of toffees than 20. Box D has 21 toffees. The box which has
lowest and 2nd lowest number of toffees has 10 and 12 toffees
respectively. Box A, C, D and F has odd number of toffees.
(II) A has more number of toffees than B but not more than D. The
difference in the number of toffees in box F and E is 7. The box
which has highest number of toffees has 8 more toffees than box F.
Total number of toffees in box B and A is 31.

Explanation (The Approach)


1. (e); From statement II- S can be married on either 28 January or 12
September.
From statement I and II- When S is married on 28 January

When S is married on 12 September

3 Adda247 Publications For any detail, mail us at


Publications@adda247.com
Cracker Book for Bank (IBPS | SBI | RRB PO | Clerk) Mains Exams

But it is given in statement II that More than three persons are


married in the same month. So, we will get that S is married on 12
September and final arrangement from both statement I and II-

2. (e);
Boxes Colours
B Orange
D -
C Blue
E Green
F Red
A -
So, it is clear that, Box C is of Blue color.
From Statement I-

From Statement II-


Total Number of toffees in box B+A= 31
Difference in Number of toffees in box F-E=7
Number of toffees in box F+8= Box contains highest number of
toffees
Now combining both Statement I and II-

So, Box C of Blue color contain 25 toffees.

Example-2:

Study the following information and answer the questions given below:
(IBPS Clerk Mains 2017)
1. Four friends are sitting around a circular table named M, N, O and P. Are
they facing to the center of table? If-
I. N is sitting second to the right of P. P is facing centre. O is sitting
immediate right of N and P.
4 Adda247 Publications For any detail, mail us at
Publications@adda247.com
Cracker Book for Bank (IBPS | SBI | RRB PO | Clerk) Mains Exams

II. M is sitting immediate left of N. O is not sitting immediate left of M. O


is sitting immediate right of P.
III. P is sitting immediate of M and O. N is sitting immediate left of M. O
is sitting immediate left of P.
(a) If the data in statement I and II are sufficient to answer the question,
while the data in statement III are not sufficient to answer the
question.
(b) If the data in statement II and III are sufficient to answer the
question, while the data in statement I is not sufficient to answer the
question.
(c) If the data in statement I alone or in the statement II alone or in the
statement III alone is sufficient to answer the question.
(d) If the data in all the statement I, II and III are necessary to answer
the question.
(e) If the data in statement I and III are sufficient to answer the
question, while the data in statement II is not sufficient to answer
the question.

2. Six people A, B, C, D, E and F are living in six floor building each live in
one of apartment of building. In building there are from I to VI floor.
Ground floor is given number I, then II and so on. Who is living on
Ground floor?
I. In which floor C and B are living there is only one floor between
them. A is living in even number floor.
II. E is not living in even number floor. B is living in even number floor.
B is not living in top floor.
III. D is living in odd number floor. In which floor D and A are living,
there are two between them. E is living immediate next to C from
down.
(a) If the data in statement I and II are sufficient to answer the question,
while the data in statement III are not sufficient to answer the
question.
5 Adda247 Publications For any detail, mail us at
Publications@adda247.com
Cracker Book for Bank (IBPS | SBI | RRB PO | Clerk) Mains Exams

(b) If the data in statement I and III are sufficient to answer the
question, while the data in statement II is not sufficient to answer
the question.
(c) If the data in statement II and III are sufficient to answer the
question, while the data in statement I is not sufficient to answer the
question.
(d) If the data in all the statement I, II and III are necessary to answer
the question
(e) If the data in statement I alone or in the statement II alone or in the
statement III alone is sufficient to answer the question.

3. Who is youngest among six family members G, H, I, J, K and L? (Each


member is of different ages.)
I. G is the daughter-in-law of J, K is grandson of L, who is the father of
H.
II. G is not youngest while L is the eldest.
III. H is the father of I, who is grandson of L. J is the wife of K’s son and
I’s father
(a) If the data in statement I and II are sufficient to answer the question,
while the data in statement III are not sufficient to answer the
question.
(b) If the data in statement I alone or in the statement II alone or in the
statement III alone is sufficient to answer the question.
(c) If the data in statement I and III are sufficient to answer the
question, while the data in statement II is not sufficient to answer
the question.
(d) If the data in statement II and III are sufficient to answer the
question, while the data in statement I is not sufficient to answer the
question.
(e) If the data in all the statement I, II and III are necessary to answer
the questio

6 Adda247 Publications For any detail, mail us at


Publications@adda247.com
Cracker Book for Bank (IBPS | SBI | RRB PO | Clerk) Mains Exams

Explanation (The Approach):


1. (c); I.

II.

III.

I, II or III statement alone are sufficient to conclude that M, N, O and


P are all not facing to centre of table.
7 Adda247 Publications For any detail, mail us at
Publications@adda247.com
Cracker Book for Bank (IBPS | SBI | RRB PO | Clerk) Mains Exams

2. (c); From II and III-


6. A
5. E
4. C
3. D
2. B
1. F
It is clear that F is living on the Ground floor

3. (d);

From statements II and III, I is the youngest because G is not


youngest.

Example-3:
In each of the following questions, a question is followed by three
statements numbered I, II and III. Read all the statements to find the answer
to given question and then answer accordingly that which statement/s can
give the answer alone/together.
(SBI PO Mains 2017)
1. What is the direction of point U with respect to point X?
Statement I: Point R is 7 m to the North of point Q. Point P is 8 m to
the West of point Q. Point R is 6 m to the West of point U.
Statement II: Point B is 9 m to the North of point A. Point P is 5 m to the
North of point Z. Point Z is 4 m to the West of point A.
Statement III: Point C is 7 m to the East of point A. Point X is 2 m to the
East of point F. Point F is 3 m to the North of point C.
(a) Both I and III
(b) Both II and III
(c) All I, II and III
(d) II and either I or III
(e) Even I, II and III together are also not sufficient
8 Adda247 Publications For any detail, mail us at
Publications@adda247.com
Cracker Book for Bank (IBPS | SBI | RRB PO | Clerk) Mains Exams

2. What does the code ‘bp’ stand for in the given code language?
Statement I: In the language, ‘black white red’ is coded as ‘df dc or’
and ‘green blue grey’ is coded as ‘st hn wo’
Statement II: In the language, ‘blue pink brown’ is coded as ‘er bp hn’
and ‘pink blue white’ is coded as ‘hn or bp’
Statement III: In the language, ‘green violet orange’ is coded as ‘pa wo
kl’ and ‘yellow pink brown’ is coded as ‘bp bi er’
(a) Both II and III
(b) I and either II or III
(c) II and either I or III
(d) Both I and III
(e) All I, II and III

Explanation (The Approach):


1. (c); From all the statements

2. (c); From I and II, we get- ‘hn’ stands for ‘Blue’ So,
The code ‘bp’ stands for—Pink
From II and III, we get—
The code ‘bp’ stands for—Pink

9 Adda247 Publications For any detail, mail us at


Publications@adda247.com
Cracker Book for Bank (IBPS | SBI | RRB PO | Clerk) Mains Exams

Practice Exercise Based on new


Pattern
Directions (1): Each of the questions below, consist of a question and three
statements numbered I, II and III. You have to decide whether the data
provided in the statements are sufficient to answer the question. Read the
three statements and Give answer
(a) If the data in statement I and II together are sufficient to answer the
question, while the data in statement III are not required to answer the
question.
(b) If the data in statement I and III together are sufficient to answer the
question, while the data in statement II are not required to answer the
question.
(c) If the data in statement II and III are sufficient to answer the question,
while the data in statement I are not required to answer the question.
(d) If the data in all three statements I, II and III together are necessary to
answer the question.
(e) If the data in all the statements, I, II and III even together are not
sufficient to answer the question.

1. Six employees P, Q, R, S, T, U attend office on six different days starting


from Monday to Saturday and each of them have their office on
different floors viz. 1st, 4th, 5th, 6th, 8th, 9th. The one who has office on 1st
floor attend office on which of the following day?
(I) Only two persons attend office in between R and S. The one who
attend office on Wednesday has office on 8th floor. Q has office on
an even numbered floor. P attend office immediately before T. R
attend office before S and has office on an odd numbered floor.
(II) No one attend office after U. Only three persons attend office in
between the one who has office on 1st floor and the one who has
office on 5th floor. The one who attend office immediately before T
has office 1st floor.
(III) R has office neither on 1st nor on 5th floor. Q attend office
immediately after S. The one who attend office on Thursday does
not has office on 4th floor.
10 Adda247 Publications For any detail, mail us at
Publications@adda247.com
Cracker Book for Bank (IBPS | SBI | RRB PO | Clerk) Mains Exams

Directions (2-3): Each of the questions below consists of a question and


two statements numbered I and II given below it. You have to decide
whether the data provided in the statement are sufficient to answer the
question. Read both the statements and Give answer:
(a) If the data in statement I alone are sufficient to answer the question,
while the data in statement II alone are not sufficient to answer the
question.
(b) If the data in statement II alone are sufficient to answer the question,
while the data in statement I alone are not sufficient to answer the
question.
(c) If the data either in statement I alone or in statement II alone are
sufficient to answer the question.
(d) If the data even in both statements I and II together are not sufficient to
answer the question.
(e) If the data in both statements I and II together are necessary to answer
the question.

2. If given Output: bold 71 keen 212 tough 353 unit 634 then which of
the following will the input?
I. If step II of the given input is---
Step II: bold 71 keen 212 tough 63 35 unit
II. If Step III of the given input is---
Step III: bold 71 keen 212 tough 353 63 unit

3. Six persons A, B, C, D, E, F born in six different months of same year.


Only one person was born in one month. C was born in which of the
following month?
I. E was born in a month having least number of days. A was born in a
month having 30 days. A was not born in April, June and November.
C was born in on one of the month between D and A.
II. No person was born after B. C was born in a month having 31 days.
F was born in a month immediately after A. There is gap of 4
months in between the months in which F and D were born.
11 Adda247 Publications For any detail, mail us at
Publications@adda247.com
Cracker Book for Bank (IBPS | SBI | RRB PO | Clerk) Mains Exams

Directions (4-5): Each of the questions below, consist of a question and


three statements numbered I, II and III. You have to decide whether the
data provided in the statements are sufficient to answer the question. Read
the three statements and Give answer
(a) If the data in statement I and II together are sufficient to answer the
question, while the data in statement III are not required to answer the
question.
(b) If the data in statement I and III together are sufficient to answer the
question, while the data in statement II are not required to answer the
question.
(c) If the data in statement II and III are sufficient to answer the question,
while the data in statement I are not required to answer the question.
(d) If the data in all three statements I, II and III together are necessary to
answer the question.
(e) If the data in all the statements, I, II and III even together are not
sufficient to answer the question.

4. Six persons A, B, C, D, E, F are sitting around a circular table facing


towards the center. Each of them like different color Blue, White, Pink,
Purple, Black, Red. The one who is sitting second to the left of C like
which of the following color?
I. F sits opposite to E. The one who likes Blue color sits second to the
right of E. The one who likes Purple is an immediate neighbour of
the one who likes Blue color. B does not like Purple color. C is an
immediate neighbour of A.
II. C sits second to the left of E and likes Black color. The one who likes
Pink color sit between the one who likes Red color and the one who
likes White color. F sits to the immediate right of D.
III. Only D sits between F and B. B likes White color and sits opposite
to C.

5. Ten persons P, Q, R, S, T, U, V, W, X, Y are living in a five storey building


such as ground floor is numbered as 1, above it is floor 2 then top floor
is numbered as 5. Each of the floor has 2 flats in it as flat-1 and flat-2.
Flat-1 of floor-2 is immediately above flat-1 of floor-1 and immediately
below flat-1 of floor-3 and so on. In the same way flat-2 of floor-2 is
immediately above flat-2 of floor-1 and immediately below flat-2 of
floor-3 and so on. Who among the following lives in flat-2 of floor-4?
12 Adda247 Publications For any detail, mail us at
Publications@adda247.com
Cracker Book for Bank (IBPS | SBI | RRB PO | Clerk) Mains Exams

I. Q lives on floor-2 and T lives to the west of Q. There is two floors


gap between Q and V.
II. X lives to the east of W. V does not live in the same flat number as
W. There is a two floors gap between S and U.
III. P lives to the west of S but does not live on ground floor. Y lives
above R. Y and R live in same flat number.

Directions (6-8): Each of the questions below, consist of a question and


three statements numbered I, II and III. You have to decide whether the
data provided in the statements are sufficient to answer the question. Read
the three statements and Give answer
(a) If the data in statement I and II together are sufficient to answer the
question, while the data in statement III are not required to answer the
question.
(b) If the data in statement I and III together are sufficient to answer the
question, while the data in statement II are not required to answer the
question.
(c) If the data in statement II and III are sufficient to answer the question,
while the data in statement I are not required to answer the question.
(d) If the data in all three statements I, II and III together are necessary to
answer the question.
(e) If the data in all the statements, I, II and III even together are not
sufficient to answer the question.

6. Some persons are sitting in a row such that all are facing in the north
direction. Who among the following sits third to the left of M (less than
15 people sits in the row)?
I. M sits fourth to the right of A. Only two people sits between A and
K. O sits on the sixth right of N
II. Only 2 person sits between N and D. M sits at the end of the row.
III. D sits eighth to the left of K. B sits third to the right of O.
13 Adda247 Publications For any detail, mail us at
Publications@adda247.com
Cracker Book for Bank (IBPS | SBI | RRB PO | Clerk) Mains Exams

7. In the question given below two conclusions followed by three


statements. You have to choose the correct set of statements that
logically satisfies the given conclusions
Conclusions:
Some Rhombus is not Square.
No Ring are Rhombus.
Statements:
I. All Ring are Square. No Rhombus is rectangle.
II. All ring are square. Some square are Rhombus.
III. Only Rectangle are square. Some ring are circle.
8. Some students are sitting in a row such that all are facing in the same
direction. How many persons sits in the row.
I. A sits fifth to the right of G, who sits seventh to the right of M. Two
persons sits between D and A. M does not sits at the end of the row.
A, sits at the end of the row.
II. D sits sixth to the right of B. B sits third to the right of M.
III. A sits seventh to the right of M. M sits at the end of the row. Three
persons sits between O and A, who does not sit fifth place from the
end of the row.

Directions (9-10): Each of the questions below consists of a question and


two statements numbered I and II given below it. You have to decide
whether the data provided in the statements are sufficient to answer the
question.
(a) If statements I alone is sufficient to answer the question, but statement
II alone is not sufficient to answer the question.
(b) If statement II alone is sufficient to answer the question, but statement I
alone is not sufficient to answer the question.
(c) If statement either I or II is sufficient to answer the question.
(d) If both the statements I and II taken together are not sufficient to
answer the questions.
(e) If both the statements I and II taken together are sufficient to answer
the questions.
14 Adda247 Publications For any detail, mail us at
Publications@adda247.com
Cracker Book for Bank (IBPS | SBI | RRB PO | Clerk) Mains Exams

9. In a class of certain number of students, all of them are sitting in a row


according to their rank. J’s rank is 10th from the left end. What will be
L’s rank from the left end?
(I) K’s rank is 5th from the right end and he is 4th from the right of H.
Only three persons sit between H and D. D is 7th from the left end.
(II) O is exactly in between J and L. O is third to the left of H.

10. Six persons P, Q, R, S, T, U are sitting around a circular table. Some of


them are facing the center while some are facing away from the center.
How many persons are facing inside?
(I) P sits third to the right of Q, who is facing outside. Both R and U are
immediate neighbour of each other. S is not an immediate
neighbour of Q.
(II) U sits second to the left of Q, who is sitting third to the left of P. R
sits second to the right of T.

Directions (11-12): Each of the questions below consists of three


statements numbered I, II and III given below it. You have to decide
whether the data provided in the statements are sufficient to answer the
question.

11. Five persons P, Q, R, S and T are standing in a row facing north in


increasing order of their heights from left to right. That means the
shortest one stands at the leftmost end while the tallest one stands at
the rightmost end. Who among the following is the fourth tallest?
I. Q and S are shorter than R. S height is 172 cm. Only two persons
are standing to the right of S.
II. Q’s height is 178 cm. T is not an immediate neighbour of R.
III. Q is an immediate neighbour of T and S. More than two persons are
standing to the left of P.
(a) Statement (I) and Statement (II) together are sufficient.
(b) Statement (II) and Statement (III) together are sufficient.
(c) Statement (I) and Statement (III) together are sufficient.
(d) All the statement (I), statement (II) and statement (III) together are
sufficient.
(e) Either two of the given statements are sufficient.

15 Adda247 Publications For any detail, mail us at


Publications@adda247.com
Cracker Book for Bank (IBPS | SBI | RRB PO | Clerk) Mains Exams

12. A, B, C, D and E are five friends among the 15 persons who participated
in quiz. Each of the 15 participants obtained different marks (a whole
number). One of the participant among the given five friends obtained
11 marks. Who obtained 11 marks?
I. B obtained 4 marks. Least marks obtained by a person is 2. Third
lowest scorer obtained either 5 or 6 marks.
II. Marks of C lies somewhere between D and A. There are only five
participants whose marks lies between E and D. A obtained 3 more
marks than B.
III. D scored 5 more marks than A. E scored 18 marks. Neither A nor B
score more marks than D. There are only two candidates whose
marks lies between C and A.
(a) Statement (I) and Statement (II) together are sufficient.
(b) Any two statements together are sufficient.
(c) Either statement (I) and statement (II) together or statement (III)
are statement
(d) All the statement (I), statement (II) and statement (III) together are
sufficient.
(e) None is sufficient.

Directions (13-15): Each of the questions below consists of three


statements numbered I, II and III given below it. You have to decide
whether the data provided in the statements are sufficient to answer the
question.
Given answer:
(a) If the data in statement I alone are sufficient to answer the question,
while the data in statement II alone are not sufficient to answer the
question.
(b) If the data in statement II alone are sufficient to answer the question,
while the data in statement I alone are not sufficient to answer the
question.
(c) If the data either in statement I alone or in statement II alone are
sufficient to answer the question.
(d) If the data even in both statements I and II together are not sufficient to
answer the question.
(e) If the data in both statement I and II together are necessary to answer
the question.
16 Adda247 Publications For any detail, mail us at
Publications@adda247.com
Cracker Book for Bank (IBPS | SBI | RRB PO | Clerk) Mains Exams

13. Six persons P, Q, R, S, T and U are sitting around a circular table, are
they all facing the center?
I. P sits second to the left of U. S sits second to the right of U. T does
not sit opposite to S who sit second to the left of P. Both T and R are
immediate neighbour of P. U is facing the center. R sits second to
the right of T. Q sits second to the right of R. Both Q and S are facing
same direction as P.
II. Q is second to the left of T. Only S is between Q and T. R is to
immediate left of U. Q sits opposite to P. P is second to the right of
S. T is second to the right of Q and second to the left of R. Both Q
and P faces each other.

14. Five boxes D, V, F, N, K are placed in a row from west to east direction
according to their weight in an increasing order. And also each of them
is of different color.
The box which is second lightest is of which color?
(I) F placed at one of the extreme end. Purple box is heavier than Red
box. Neither box K nor V is the heaviest. Box D is not of Blue and
Red color. Pink box is not heavier than Red box.
(II) Only one box is placed in between box K and V. Box D is heavier
than Pink box. Box V is not placed adjacent to F. Box N is of Black
color. Blue box is lighter than N. Neither K nor F is of Blue color.
Blue box is heavier than Purple box.
15. Five persons K, L, M, N, O lives on five different floors such as bottom
floor is numbered as 1 and top floor is numbered as 5. Also each of
them born in different months of a year but no two person born in two
consecutive months. The one who lives on fourth floor born in which of
the following month?
(I) Three person lives between L and O. O is older than M and born in
a month having 31 days. M lives on third floor and born in April. L
is born in one of the month after July but in a month having 30
days.
(II) Only one person lives between K and N and K is younger than N. O
does not live above M. N is born before L. K was not born in last
month of the year and is younger than L. N was not born in a month
of 30 days.

17 Adda247 Publications For any detail, mail us at


Publications@adda247.com
Cracker Book for Bank (IBPS | SBI | RRB PO | Clerk) Mains Exams

Directions (16-20): Each of the questions given below consists of a


question and two statements numbered I and II. You have to decide
whether the data provided in the statements are sufficient to answer the
question.
(a) If statement I alone is sufficient to answer the question, but statement
II by itself is not sufficient to answer the question.
(b) If statement II by itself is sufficient to answer the question, but
statement I alone is not sufficient to answer the question.
(c) If statement either I or II is sufficient to answer the question.
(d) If both the statements I and II taken together are not sufficient to
answer the question.
(e) If both the statements I and II taken together are sufficient to answer
the question.

16. Conclusion: Some Milk are Coffee. Some Coffee are not Cold-drinks.
Which of the following set of statement is required that logically
satisfies given conclusions (Given statements to be true even if they
seem to be at variance from commonly known facts).
Statement I: Some Milk are Drink. Some Coffee are Tea.
Statement II: All drink are Coffee. No Tea is Cold-drink.

17. Six persons A, B, C, D, E and F are sitting around a circular table. Are all
facing towards the center?
Statement I: E sits second to the left of A, who is facing towards the
center. C sits opposite to D. F sits to the immediate right of A. A sit
second left of C and C does not sit between A and E. F sits second to
right of D. F sits third left of E.
Statement II: F sits second left of B. E sits second right of C and is not
an immediate neighbour of F. D sits third left of C. B is facing towards
the center.

18. What is code of ‘Allegation’?


Statement I: In certain code language, ‘Speech audience popularity’ is
coded as- ‘kl da bp’ and ‘Conference debate report’ is coded as- ‘nu ga
tr’
Statement II: In certain code language, ‘Deliver patience Speech’ is
coded as- ‘bt hd kl’ and ‘Allegation conference popularity’ is coded as-
‘wa ga bp’.
18 Adda247 Publications For any detail, mail us at
Publications@adda247.com
Cracker Book for Bank (IBPS | SBI | RRB PO | Clerk) Mains Exams

19. Five persons P, Q, R, S, T are sitting in a row facing north direction. Who
among the following sits at extreme left end?
Statement I: Q is third to the left of P. T is not an immediate neighbour
of P. S sits second to the right of R.
Statement II: T sits second to the left of P. More than two persons sit
between Q and S. Q sits to the left of R.

20. Who among the following is the father of K?


I. L is the father of R. R is the brother-in-law of D. D is the wife of K. K
is the son of M, who is the wife of L.
II. F is the son of K. F is married to G, who is the daughter-in-law of D.
F is the grandson of M.

Directions (21-23): Each of the questions below, consist of a question and


three statements numbered I, II and III. You have to decide whether the
data provided in the statements are sufficient to answer the question. Read
the three statements and Give answer
(a) If the data in statement I and II together are sufficient to answer the
question, while the data in statement III are not required to answer the
question.
(b) If the data in statement I and III together are sufficient to answer the
question, while the data in statement II are not required to answer the
question.
(c) If the data in statement II and III are sufficient to answer the question,
while the data in statement I are not required to answer the question.
(d) If the data in all three statements I, II and III together are necessary to
answer the question.
(e) If the data in all the statements, I, II and III even together are not
sufficient to answer the question.
21. Some persons are sitting in a row such that all are facing in the north
direction. Who among the following sits fourth to the left of D?
I. M sits third to the right of D. Only five persons sit between M and A.
More than two person sits between M and N.
II. D sits second to the left of M, who sits sixth to the left of N. A sits third
to the right of N.
III. More than six person sits between R and D. B sits exactly between N
and A. B sits fifth to the right of R. Only one person sits between N and
A.
19 Adda247 Publications For any detail, mail us at
Publications@adda247.com
Cracker Book for Bank (IBPS | SBI | RRB PO | Clerk) Mains Exams

22. In the question given below two conclusions followed by three sets of
statement. You have to choose the correct set of statement that logically
satisfies the given conclusions
Conclusion: I. Some hands are not Socks.
II. No legs are socks.
I. All legs are Hands. Some Shoes are legs. No Legs are fingers.
II. Some watches are arm. All arms are fingers. All legs are Hands.
III. No Socks are Shoes. All legs are Shoes. Some Hands are finger.

23. Eight persons i.e. A, B, C, D, E, F, G and H are sitting around a circular


table such that some of them face inside and some face outside. How
many persons face outside?
I. A sits second to the right of E and second to the right of D. G and D
faces opposite direction. C and G are the immediate neighbour of E.
F does not face inside.
II. Not more than two persons sitting adjacent to each other faces
same direction. H sits second to the left of C, who sits third to the
left of B. H and A face same direction. C and G face opposite
direction.
III. A sits third to the left G, who sits second to the right of C. E is not
the immediate neighbor of G.

Directions (24-25): Each of the questions below, consist of a question and


three statements numbered I, II and III. You have to decide whether the
data provided in the statements are sufficient to answer the question. Read
the three statements and Give answer
(a) If the data in statement I and II together are sufficient to answer the
question, while the data in statement III are not required to answer the
question.
(b) If the data in statement I and III together are sufficient to answer the
question, while the data in statement II are not required to answer the
question.
(c) If the data in statement II and III are sufficient to answer the question,
while the data in statement I are not required to answer the question.

20 Adda247 Publications For any detail, mail us at


Publications@adda247.com
Cracker Book for Bank (IBPS | SBI | RRB PO | Clerk) Mains Exams

(d) If the data in all three statements I, II and III together are necessary to
answer the question.
(e) If the data in all the statements, I, II and III even together are not
sufficient to answer the question.

24. Six lectures on different subjects i.e. Physics, Chemistry, Math’s,


Biology, Economics and English delivered by different professor i.e. A,
B, C, D, E and F on the different days of the week starting from Monday
to Saturday but not necessarily in the same order. Who among the
following professor delivered the lecture on Wednesday and on which
subject?
I. More than two professors delivered the lecture before the one who
deliver the lecture of English. Three professors deliver the lecture
between B and C, who delivered the lecture before B. C delivered
the lecture of Biology. Only one professor delivered the lecture
between B and the one who deliver the lecture of physics.
II. Two professor deliver the lecture between C and the one who
deliver the lecture of English. Only two lectures were delivered
between Physics and Math’s. Two professors deliver the lecture
between D and E. B does not delivered the lecture of economics. A
delivered the lecture before F.
III. Only one professor deliver the lecture between F and the one who
deliver the lecture of Math’s. B delivered the lecture before the one
who delivered the lecture on Math’s. D deliver the lecture
immediately after A, who deliver the lecture on Economics.

25. There are eight persons i.e. A, B, C, D, E, F, G and H are sitting in row
such that some are facing towards north and some are facing towards
south direction. How many persons faces north direction?
I. Only one person sits on the right of A. Only three persons sits
between C and A. A and C faces opposite direction. B sits second to
the left of C. There are two vacant seats and none of the vacant
seats are at the end of the row. G and B are not the immediate
neighbour of the vacant seats.

21 Adda247 Publications For any detail, mail us at


Publications@adda247.com
Cracker Book for Bank (IBPS | SBI | RRB PO | Clerk) Mains Exams

II. D sits third to the left of G, who faces same direction as A. G does
not sits at the end of the row. H and B face opposite direction. E
faces North. H does not face north direction.
III. Immediate neighbours of G face opposite direction. Four persons
sits between the two vacant seats.
E sits third to the left of F. F does not sits at the end of the row. H
and D faces opposite direction.

Directions (26-30): Each of the questions below consists of a question and


two statements numbered I and II given below it. You have to decide
whether the data provided in the statement are sufficient to answer the
question. Read both the statements and Give answer:
(a) If the data in statement I alone are sufficient to answer the question,
while the data in statement II alone are not sufficient to answer the
question.
(b) If the data in statement II alone are sufficient to answer the question,
while the data in statement I alone are not sufficient to answer the
question.
(c) If the data either in statement I alone or in statement II alone are
sufficient to answer the question.
(d) If the data even in both statements I and II together are not sufficient to
answer the question.
(e) If the data in both statements I and II together are necessary to answer
the question.

26. In a certain code language ‘fi ku le ja’ means ‘Indian Cricket Premier
League’. Then what is the code of ‘league’? If,
I. In the given code language ‘Premier league shifted venue’ is written
as ‘le hi nu ku’.
II. In the given code language ‘league venue cricket pune’ is written as
‘nu le fi un’.

27. How is N related to D?


I. N is the only child of R. D is father of M, who is brother-in-law of S.
P is the brother of M. M is married to R. P is unmarried.

22 Adda247 Publications For any detail, mail us at


Publications@adda247.com
Cracker Book for Bank (IBPS | SBI | RRB PO | Clerk) Mains Exams

II. D is married to A. P is the son of N. Q is brother of M. N is married


to M. Q is the only male child of A. S is the sister of N. A is a male
28. On which of the following day of the week (starting from Monday and
ends on Sunday) match of CSK is scheduled?
I. Dhoni captain of CSK completely remembers that match is
scheduled after Wednesday but not on Sunday.
II. Jadeja correctly remembers that the match of CSK is scheduled
before Friday but after Monday.

29. Some persons i.e. P, Q, A, C, M, D and S are sitting in a row such that all
are facing in the north direction, who among the following sits on the
right end of the row?
I. Q sits third to the left of D and both does not sit at the end of the
row. S sits fourth to the right of A and one of them sits at the end of
the row. C sit third to the right of P. M is not the immediate
neighbour of D.
II. D sits second to right of Q. One of the immediate neighbour of D sits
fifth to the right of M. S sits second to the right of A. Neither P nor C
sits at the end of the row. P and C are not the immediate
neighbours of each other.

30. How many people are sitting around the circle (all are facing inside)?
I. Z is sitting third to the right of C, who is sitting third to the right of
A. Three persons are sitting in between F and Z.
II. Two persons are sit in between A and F who is sitting second to the
right of Z. C is sitting third right of A and Three person sit between
Z and C.

Directions (31-34): Each of the questions below consists of a question and


two statements numbered I and II given below it. You have to decide
whether the data provided in the statements are sufficient to answer the
question. Read both the statements. Give answer-
(a) If the data in statement I alone are sufficient to answer the question,
while the data in statement II alone are not sufficient to answer the
question.

23 Adda247 Publications For any detail, mail us at


Publications@adda247.com
Cracker Book for Bank (IBPS | SBI | RRB PO | Clerk) Mains Exams

(b) If the data in statement II alone are sufficient to answer the question,
while the data in statement I alone are not sufficient to answer the
question.
(c) If the data either in statement I alone or in statement II alone are
sufficient to answer the question.
(d) If the data in both statements I and II together are not sufficient to
answer the question.
(e) If the data in both statements I and II together are necessary to answer
the question.
31. A, B, C, D, E and F are sitting in a circle, some of them facing towards the
centre while some are facing away from the center of the circle. How
many persons are facing inside?
I F is on the immediate left of E. Only D is between B and E. C is
immediate right of F. C is second to the left of B. A does not faces
same direction as E. D and C faces opposite direction of A.
II. A is facing D. Only C is between A and B. Only F is between E and A.
E sits second to the right of B. Both F and C faces same direction but
opposite to B. F does not sits to the immediate left E.

32. Amongst P, Q, R, S and T all of them are of different price. Also they are
placed one above another. Which book is placed at the top?
I. Price of the book R is more than the price of book P. Only two
books are placed between book R and P. Price of the book S is
second costliest among all and is placed above T.
II. Price of the book Q is more than only P and T and costliest book is
not placed at top. The third costliest book is exactly in the middle.

33. Six people viz. I, J, K, L, M and N lives in a Building on different floors


from top to bottom (such as ground floor numbered as 1 and top is
numbered as 6). Each of them likes different colours Black, White, Blue,
Pink, Purple, Grey. Which color is liked by the one who lives on 4th
floor?
(I) Only one person lives between L and M. J lives above I who likes
Black color. There is a gap of three floors between J and L and both
of them lives on odd number of floor. The one who lives on top
floor likes Purple color.
(II) K likes White color. The one who likes Grey color lives above I. J
does not like Grey color.
24 Adda247 Publications For any detail, mail us at
Publications@adda247.com
Cracker Book for Bank (IBPS | SBI | RRB PO | Clerk) Mains Exams

34. Six people viz. D, F, L, M, Q, R are sitting in a row some of them are
facing north while some of them are facing south direction. Who sits to
the immediate right of D?
(I) M sits third to right of R.Q sits second to the left of M. R is not an
immediate neighbour of Q. D is not an immediate neighbor of Q.
(II) L sits third to the left of Q who is facing north direction. Both the
immediate neighbours of M faces same direction as M.

Directions (35): Each of the questions below consists of a question and


two statements numbered I and II given below it. You have to decide
whether the data provided in the statement are sufficient to answer the
question. Read both the statements and Give answer:
(a) If the data in statement I alone are sufficient to answer the question,
while the data in statement II alone are not sufficient to answer the
question.
(b) If the data in statement II alone are sufficient to answer the question,
while the data in statement I alone are not sufficient to answer the
question.
(c) If the data either in statement I alone or in statement II alone are
sufficient to answer the question.
(d) If the data even in both statements I and II together are not sufficient to
answer the question.
(e) If the data in both statements I and II together are necessary to answer
the question.

35. Five friends J, K, L, M, N born on different dates 8, 13, 19, 24, 31 but not
necessarily in the same order in the month of March and June in the
same year. Only one person born on one of the given date. M is born on
which of the following date?
I. Both M and J are not born on even date of the month. M is born
before J. Two persons are born in between M and N.
II. Nobody is born after K. L is born on a date in between M and J. L is
born on an even date. M does not born on 13 and N does not born
on 19.
25 Adda247 Publications For any detail, mail us at
Publications@adda247.com
Cracker Book for Bank (IBPS | SBI | RRB PO | Clerk) Mains Exams

Directions (36-37): Each of the questions below, consist of a question and


three statements numbered I, II and III. You have to decide whether the
data provided in the statements are sufficient to answer the question. Read
the three statements and Give answer
(a) If the data in statement I and II together are sufficient to answer the
question, while the data in statement III are not required to answer the
question.
(b) If the data in statement I and III together are sufficient to answer the
question, while the data in statement II are not required to answer the
question.
(c) If the data in statement II and III are sufficient to answer the question,
while the data in statement I are not required to answer the question.
(d) If the data in all three statements I, II and III together are necessary to
answer the question.
(e) If the data in all the statements, I, II and III even together are not
sufficient to answer the question.

36. Seven persons i.e. A, B, C, D, E, F and G are sitting around a circular


table. Are all the persons facing towards the centre of the table?
I. A sits third to the right of C, who sits second to the left of D. Only two
person sits between G and A. B is not the immediate neighbour of C. C
faces inside.
II. D sits third to the left of B. and second to the right of E. C sits third to
the right of F, who is not the immediate neighbour of E.
III. F sits third to the right of B. A and G faces same direction. F sits second
to the left of A. E sits third to the right of F, and on the immediate left of
B. E and B faces same direction. B is not an immediate neighbour of A.

37. There are 9 members i.e. A, B, D, G, H, R, Z, I and K in a family having


three generation. How is R related to Z?
I. Z has three children. C is the unmarried brother of G. B is the sister-in-
law of D. There are three married couple in the family.
II. D is the daughter in law of A, who is the father in law of I. K is the
cousin of R.
III. Z has only one daughter. I is the brother-in-law of C. B is the mother of
K.

26 Adda247 Publications For any detail, mail us at


Publications@adda247.com
Cracker Book for Bank (IBPS | SBI | RRB PO | Clerk) Mains Exams

Directions (38-40): Each of the questions below consists of a question and


two statements numbered I and II given below it. You have to decide whether
the data provided in the statements are sufficient to answer the question.
Read both the statements and give answer.
(a) If the data in statement I alone are sufficient to answer the question, while
the data in statement II alone are not sufficient to answer the question.
(b) If the data in statement II alone are sufficient to answer the question, while
the data in statement I alone are not sufficient to answer the question.
(c) If the data either in statement I alone or in statement II alone are sufficient
to answer the question.
(d) If the data in both statements I and II together are not sufficient to answer
the question.
(e) If the data in both statements I and II together are necessary to answer
the question.
38. Eight people viz. G, H, I, J, K, L, M and N lives in a Building on different
floors from top to bottom(such as ground floor numbered as 1 and top
is numbered as 8) where I lives on floor number 6. Who among the
following lives on 4th floor?
(I) Only one person lives between L and M. J lives above I. There is a
gap of three floors between J and L and both of them lives on odd
number of floor.
(II) Three persons live between K and H. N lives just above H who lives
on even numbered floor.

39. Seven people viz. A, D, F, L, M, Q, R are sitting in a row some of them are
facing north while some of them are facing south direction. R sits at an
extreme end of the row. F is not facing south direction. D sits third to
the left of F. Who sits to the immediate right of F?
(I) M sits third to right of R.Q sits second to the left of M.D is not an
immediate neighbor of Q.
(II) A sits third to the left of Q. F is an immediate neighbor of M.F sits
second to the right of L.

40. In a certain code language -‘benefits life in loss’ is coded as ‘hlt sa rtv
mlp’ then what will be the code of ‘Benefits’?
(I) The code of ‘live life trouble benefits’ is ‘mlp hlt ngi snk’,
(II) The code of ‘live happy feeling at’ is ‘ngi riy nop hus’.
27 Adda247 Publications For any detail, mail us at
Publications@adda247.com
Cracker Book for Bank (IBPS | SBI | RRB PO | Clerk) Mains Exams

Solutions

1. (a); From I

Case-1 Case-2
Days Person Floors Person Floors
Monday R
Tuesday P/ R
Wednesday T/ 8th P 8th
Thursday S T
Friday P/ S
Saturday T/

From II,
Case-1 Case-2
Days Person Floors Person Floors
Monday 1st
Tuesday T 1st
Wednesday T
Thursday
Friday 5th
Saturday U U 5th

Now, from I, II-

Days Person Floors


Monday R 9th
Tuesday P 1st
Wednesday T 8th
Thursday S
Friday Q
Saturday U 5th
So, clearly P has office on 1st floor attend office on Tuesday.

28 Adda247 Publications For any detail, mail us at


Publications@adda247.com
Cracker Book for Bank (IBPS | SBI | RRB PO | Clerk) Mains Exams

2. (d); Even by combining both I and II we cannot find the input of the
output given in the question.

3. (d); From I,
Person Month
E February
A September

From II,
Person Month
B December
C (31)

From I and II,


Persons Months
A September
B December
D May
C July/August
E February
F October

So, even by combining both I and II we cannot find that C was born
in which of the following month.
4. (a); From I, From II,

29 Adda247 Publications For any detail, mail us at


Publications@adda247.com
Cracker Book for Bank (IBPS | SBI | RRB PO | Clerk) Mains Exams

From III,

From I and II,

So, we get that the one who is sitting second to the left of C is D and
like Blue color.

5. (d); From I,
Floors Flat-1 Flat-2
5 V/ V/
4
3
2 T Q
1

30 Adda247 Publications For any detail, mail us at


Publications@adda247.com
Cracker Book for Bank (IBPS | SBI | RRB PO | Clerk) Mains Exams

From I, II and III-

Floors Flat-1 Flat-2


5 Y V
4 P S
3 W X
2 T Q
1 R U

6. (d); The data in all three statements I, II and III together are necessary
to answer the question.
From statement I and III M sits at the end of the row and fourth to
the right of A. B sits third to the right of O. less than 15 people sits
in the row. Only two people sits between A and K and D sits eighth
to the left of K so K sits third to the right of A because if K sits third
to the left of A and we placed D then there will be sixteen persons
in the row which can’t be possible. Now by using statement II that
Only 2 person sits between N and D. M sits at the end of the row.
We get our final answer B sits third to the left of M.

7. (b); Using statement I and III we can find the definite relation between
Rhombus and Square and between Ring and Rhombus which is not
possible if we use statement II.

31 Adda247 Publications For any detail, mail us at


Publications@adda247.com
Cracker Book for Bank (IBPS | SBI | RRB PO | Clerk) Mains Exams

8. (e); From the statements, I, II

As it is given that M does not sits at the end of the but it is also not
mention in any statement that how many persons sits on the left of
M. Hence, it cannot get confirmed how many person sits in the row.
From statement II and III,

From these we cannot conclude that O sitting at the end of the row
or not.
9. (e); From I,

From I and II,

10. (d); From I,

From II,

32 Adda247 Publications For any detail, mail us at


Publications@adda247.com
Cracker Book for Bank (IBPS | SBI | RRB PO | Clerk) Mains Exams

Even by combining both the statements I and II together we get that


both the are not sufficient to answer the questions.

11. (c); From (I) and (III), The arrangement of person facing north in
increasing order of their heights from left to right is--------------
T< Q < S < P/R < P/R

12. (d); Using the information given in statement (I), (II) and (III),
From (I) B obtained 4 marks. Least marks obtained by a person is
2. Third lowest scorer obtained either 5 or 6 marks. It means B is
the second least scorer.
From (II) A obtained 3 more marks than B. It means A obtained
seven marks he could be either the fourth or the fifth lowest scorer.
Marks of C lies somewhere between D and A.
From (III) D scored 5 more marks than A. It means D scored 12
marks. There are only two candidates whose marks lies between C
and A. E scored 18 marks.

Rank Person Marks


1 2
2 B 4
3 5/6
4 A 7
5
6
7 C
8 D 12
9
10
11
12
13
14
15

33 Adda247 Publications For any detail, mail us at


Publications@adda247.com
Cracker Book for Bank (IBPS | SBI | RRB PO | Clerk) Mains Exams

Or

Rank Person Marks


1 2
2 B 4
3 5
4 6
5 A 7
6
7
8 C
9 D 12
10
11
12
13
14
15

And as we know that One of the participant among the given five
friends obtained 11 marks it means C must have scored 11 marks.

13. (c);
From II, From II,

34 Adda247 Publications For any detail, mail us at


Publications@adda247.com
Cracker Book for Bank (IBPS | SBI | RRB PO | Clerk) Mains Exams

14. (e); From I,

From II,

Purple < Blue < Black (N) and Pink < D


From both I and II,
We get that F is the lightest box and is of Pink color and K is second
lightest and is of Red color.

15. (d); From I,

And L can be born either in September or November. And O is born


in January.
From II,
K<L<N
From both I and II,
Even by combining both the statements we only know that K is
born in November and N is born in July but we cannot find that
either K or N live on 4th floor, so both together are not sufficient.

16. (e); By using both statement I and II-

35 Adda247 Publications For any detail, mail us at


Publications@adda247.com
Cracker Book for Bank (IBPS | SBI | RRB PO | Clerk) Mains Exams

17. (d); Even by combining both statement I and II the direction of facing of
F cannot be determined. So, both the statements I and II taken
together are not sufficient to answer the question.

18. (e); By using both statement I and II the code of Allegation will be- wa.

19. (c); From either I or II we will get that Q sits at extreme left end of the
row.

20. (a); Statement I alone is sufficient to answer the question.

But from Statement II we cannot find that M is either father or


mother of K. So, statement II by itself is not sufficient to answer the
question.
36 Adda247 Publications For any detail, mail us at
Publications@adda247.com
Cracker Book for Bank (IBPS | SBI | RRB PO | Clerk) Mains Exams

21. (b); The statement I and III together are sufficient to answer the
question;
Step 1: M sits third to the right of D. Only five persons sits between
M and A. More than two person sits between M and N. Only one
person sits between N and A.

Step 2: B sits exactly between N and A. B sits fifth to the right of R.


More than six person sits between R and D. So, case 1 Case 2 and
case 4 gets eliminated.

Hence, B sits fourth to the left of D.

22. (c); Combining both the statements II and III we get,

I. From the venn diagram it is clear that all legs are hand, all legs
are shoes and no shoes is socks. Hence, we can conclude that
some hands are not socks.
II. From the venn diagram it is clear that all legs are shoes and no
shoes are socks. Hence, we can conclude that no legs are socks.

37 Adda247 Publications For any detail, mail us at


Publications@adda247.com
Cracker Book for Bank (IBPS | SBI | RRB PO | Clerk) Mains Exams

23. (a); Using statements, I and II.


A sit second to the right of E and second to the right of D. C and G
are the immediate neighbour of E. H sits second to the left of C, who
sits third to the left of B.

Now, not more than two persons sitting adjacent to each other
faces same direction. F does not face inside. H and A face same
direction. C and G face opposite direction. So, case 1 gets
eliminated.

Therefore, five person faces outside.


24. (b); From statement I and III
More than two professors delivered the lecture before the one who
deliver the lecture of English. Three professors deliver the lecture
between B and C, who delivered the lecture before B. C delivered
the lecture of Biology. B delivered the lecture before the one who
delivered the lecture on Math’s. Only one professor delivered the
lecture between B and the one who deliver the lecture of physics.
38 Adda247 Publications For any detail, mail us at
Publications@adda247.com
Cracker Book for Bank (IBPS | SBI | RRB PO | Clerk) Mains Exams

Only one professor deliver the lecture between F and the one who
deliver the lecture of Math’s. D deliver the lecture immediately
after A, who deliver the lecture on Economics. So, the final
arrangement is--

Days Professor Subject


Monday C Biology
Tuesday A Economics
Wednesday D Physics
Thursday F English / Chemistry
Friday B Chemistry / English
Saturday E Math’s

25. (d); Using the given informations from I, II and III,


Four persons sits between the two vacant seats. Only one person
sits on the right of A. There are two vacant seats and none of the
vacant seats are at the end of the row. G and B are not the
immediate neighbour of the vacant seats. B sits second to the left of
C. A and C faces opposite direction.

39 Adda247 Publications For any detail, mail us at


Publications@adda247.com
Cracker Book for Bank (IBPS | SBI | RRB PO | Clerk) Mains Exams

D sits third to the left of G, who faces same direction as A. G does


not sits at the end of the row. Immediate neighbours of G face
opposite direction. E sits third to the left of F. F does not sits at the
end of the row. H and B face opposite direction. E faces North. H
and D faces opposite direction.

H does not face north direction. So, from this case-1 will be
eliminated. And the final arrangement is-----

26. (e); Using Statement I and II together,

Words Codes
Indian ja
Cricket fi
League le
Venue nu
Premier ku
Shifted hi
pune un

The code for League is ‘le’.

40 Adda247 Publications For any detail, mail us at


Publications@adda247.com
Cracker Book for Bank (IBPS | SBI | RRB PO | Clerk) Mains Exams

27. (b); From statement I, we can’t determine the gender of N.

From the statement II,

N is the son-in-law of D.

28. (e); From statement I, Dhoni remembers that the match is scheduled on
either Thursday, Friday or on Saturday.
From Statement II, Jadeja remembers that the match is scheduled
either on Tuesday, Wednesday or on Thursday.
So, using both the statement CSK match is scheduled on Thursday.

29. (c); From Statement 1:


Step 1: From the given statement I, Q sits third to the left of D and
both does not sit at the end of the row. S sits fourth to the right of A
and of them sits at the end of the row. There will be two possible
cases

41 Adda247 Publications For any detail, mail us at


Publications@adda247.com
Cracker Book for Bank (IBPS | SBI | RRB PO | Clerk) Mains Exams

Step 2: Now, C sit third to the right of P. M is not the immediate


neighbour of D. So, case 1 will be eliminated and we get our final
answer,

From Statement II,


Step 1: As it is given that, D sits second to right of Q. One of the
immediate neighbour of D sits fifth to the right of M. So, there will
be two possible cases,

Step 2: Now, S sits second to the right of A. Neither P nor C sits at


the end of the row. Neither P nor C are the immediate neighbours
of each other. So, case 2 will be eliminated and we get our final
answer,

30. (b); From statement II,


Step 1: Using the given conditions, two persons are sit in between
A and F who is sitting second to the right of Z. There will be two
possible cases

42 Adda247 Publications For any detail, mail us at


Publications@adda247.com
Cracker Book for Bank (IBPS | SBI | RRB PO | Clerk) Mains Exams

Step 2: Now, C is sitting third right of A, So, case 1 will be


eliminated. Continuing with step 2, Three person sit between Z and
C. Hence, we get our final answer

31. (c); From I, we get two possible case-

From both of the above cases we get that three persons are facing
inside.
From II,

From both of the above cases we get that four persons are facing
inside.
So, either I or II is sufficient to answer the question.
43 Adda247 Publications For any detail, mail us at
Publications@adda247.com
Cracker Book for Bank (IBPS | SBI | RRB PO | Clerk) Mains Exams

32. (d); From I,


R>P and > S> > >

Books
R/P

R/P

From II, Q> P, T


Books

From I and II,


R>S>Q>P/T>P/T

As, even by combining both we can say either book S or P is placed


at top. So, data in both statements I and II together are not
sufficient to answer the question.

33. (e); From I,


Floors Persons Colours
6 Purple
5 J
4 I/ Black/
3 M
2 I/ Black/
1 L
44 Adda247 Publications For any detail, mail us at
Publications@adda247.com
Cracker Book for Bank (IBPS | SBI | RRB PO | Clerk) Mains Exams

From II, K likes White color. The one who likes Grey color lives
above I.

From I and II,

Floors Persons Colours


6 Purple
5 J Grey/
4 K White
3 M Grey/
2 I Black
1 L

So, From both I and II we get that K lives on 4th floor and likes
White colour.

34. (d); From I,

From II,

From I and II,

Even by combining I and II we cannot get the direction of D, so data


in both statements I and II together are not sufficient to answer the
question.
45 Adda247 Publications For any detail, mail us at
Publications@adda247.com
Cracker Book for Bank (IBPS | SBI | RRB PO | Clerk) Mains Exams

35. (e); From statement II- L can be born on either 24 March or 8 June.
From statement I and II- When L is born on 24 March

When L is born on 8 June

But it is given in statement II that M does not born on 13 and N


does not born on 19 So, final arrangement from both statement I
and II-

36. (b); A sits third to the right of C, who sits second to the left of D. Only
two person sits between G and A. B is not the immediate neighbour
of C. F sits third to the right of B. C faces inside. B is not an
immediate neighbour of A.

A and G faces same direction. F sits second to the left of A. E sits


third to the right of F, and on the immediate left of B. E and B faces
same direction.

46 Adda247 Publications For any detail, mail us at


Publications@adda247.com
Cracker Book for Bank (IBPS | SBI | RRB PO | Clerk) Mains Exams

37. (e);

38. (b); Only statement II is sufficient to answer the question as from II


statement it is clear that H lives on 4th floor. Statement I is not
required to answer the question.
Floors Persons
8 K
7
6 I
5 N
4 H
3
2
1

39. (a); From Only statement I there will be two possible cases-----
Case 1-

Case 2-

From both possible cases it is clear that Q sits to the immediate


right of F. So only Statement I alone is sufficient to answer the
questions but Statement II alone is not sufficient to answer the
question.

47 Adda247 Publications For any detail, mail us at


Publications@adda247.com
Cracker Book for Bank (IBPS | SBI | RRB PO | Clerk) Mains Exams

40. (d); Statement I and II together are not sufficient to answer the
question as by combining both I and II together we get the code of
Benefit can be either mlp or hlt.

 Key Points to Remember for New Pattern Questions


➢ Today’s era is a race in which everybody wants to put
minimum efforts to get maximum output, Data sufficiency
is similar concept in which you have to find the minimum
information which is required to get the answer of your
question.
➢ The topic of data sufficiency is bit molded as seen in the
recent exams as most of its part is either in the form of a
puzzle or seating arrangement.
➢ While solving the data sufficiency questions based on
puzzles you have to draw separate arrangement from
every statement as you have to find the answer of your
question even if you are not be able to draw the complete
arrangement from that particular statement.
➢ If there are two possible solutions occur from one
statement or from all, then it will be an insufficient
information as we are not be able to get a single solution.
➢ Now a days the questions of data sufficiency as seen in the
exam are bit tricky and you have to be smart enough to
extract the required information from it.

48 Adda247 Publications For any detail, mail us at


Publications@adda247.com
Cracker Book for Bank (IBPS | SBI | RRB PO | Clerk) Mains Exams

Adda247 Publications For any detail, mail us at


1
Publications@adda247.com
Cracker Book for Bank (IBPS | SBI | RRB PO | Clerk) Mains Exams

Chapter

5 Coding-Decoding

BEST APPROACH TO SOLVE THE QUESTIONS

Coding decoding is the topic which shows continuous changes in it. In each
and every exam this topic has gone through various changes. So, to get
updated we are providing you some questions based on recent exams. By
solving these you may get to know about the latest pattern and you will be
able to prepare for the upcoming exams.

Example-1:
Directions (1-5): Study the information and answer the following
questions: (IBPS PO Mains-2016)
In a certain code language
"fresh mind happy life " is coded as " H#14 M@17 K#33 U@17 "
"kashi city of temple" is coded as " S#20 G@28 L@21 O@25"
"Sarnath belongs to varanasi" is coded as"G#27 T#21 G@35 H@31"
"One airport in banaras " is coded as "M#20 I#21 R@23 Z#21 "

1. What is the code for ‘Excellence’ in the given code language?


(a) X@10 (b) X#10 (c) E@10
(d) E#10 (e )None of these

2. What is the code for ‘University' in the given code language?


(a) G#46 (b) T@46 (c) N@46
(d) G@46 (e) None of these

3. What may be the possible code for ‘One army ’ in the given code
language?
(a) M#20 M@26 (b) M#20 T@26 (c) N@26 M#20
(d) M#20 N@26 (e) None of these

Adda247 Publications For any detail, mail us at


2
Publications@adda247.com
Cracker Book for Bank (IBPS | SBI | RRB PO | Clerk) Mains Exams

4. What may be the possible code for ‘Veracity’ in the given code
language?
(a) T@47 (b) G@47 (c) E@47
(d) G#47 (e) None of these

5. What is the code for ‘Ghats in Kashi' in the given code language?
(a) R@21 T#20 G#26 (b) G#26 R@23 S#20 (c) G@26 S#20 R@21
(d) G@26 S#20 R#21 (e) None of these

Explanations (The Approach):


These are the latest pattern of coding-decoding questions. In these
questions we are applying following concept:-

1. (a); X@10 2. (d); G@46 3. (d); M#20 N@26


4. (b); G@47 5. (b); G#26 R@23 S#20
Adda247 Publications For any detail, mail us at
3
Publications@adda247.com
Cracker Book for Bank (IBPS | SBI | RRB PO | Clerk) Mains Exams

Example-2:
Direction (1-5): Study the following information and answer the given
questions. (IBPS PO Mains-2017)
In alphabetical series A-Z each letter except vowels is assigned a different
number from 1-8 (for ex- B is coded as 1, C-2………..K-8)and again those
numbers get repeated(for ex- L-1, M-2……..so on).
Also each vowel is assigned a different symbol viz. #, $, %, @, &.
For example-
In coded language-
“She is girl” is coded as - 76% #7 5#61
“What did you like” is coded as - 26$8 3#3 4@& 1#8%
“It is Opinion” is coded as - #8 #7 @4#3#@3

(i) If both first and last letter of a word is vowel then the codes of both the
vowels are interchanged.
(ii) If first letter of a word is vowel and last letter is consonant then both
are to be coded as *.
(If the word does not satisfy the conditions given above then the letters
of that word are to be coded as per the directions given above)

1. What can be the code of ‘Nothing perfect’?


(a) 3@86#53 4%64%28 (b) 3@68#35 4%64%28
(c) 3@86#35 4%64%38 (d) 3@86#35 4%64%28
(e) None of these

2. What can be the code of ‘Exam was easy’?


(a) *3$* 2$7 *$7* (b) *3$* 2$7 %$74 (c) %3$2 2$7 *$7*
(d) *3$* 2$7 $74% (e) None of these

3. What can be the code of ‘Create style’?


(a) 2#6$8% 7841% (b) 26%$8% 7814% (c) 26%$8% 7841%
(d) 26%$6% 7841% (e) None of these
4. What can be the code of ‘Strength of god’?
(a) 768%3586 4@ 5@3 (b) 786%3586 ** 5@3(c) 786%3586 @* 5@3
(d) 786%3856 4@ 5@3(e) None of these

Explanations (The Approach):


Adda247 Publications For any detail, mail us at
4
Publications@adda247.com
Cracker Book for Bank (IBPS | SBI | RRB PO | Clerk) Mains Exams

Direction (1-4): In this new pattern coding decoding each letter, except
vowel, is assigned a number from 1-8 So, B-1, C-2, D-3, F-4, G-5, H-6, J-7, K-
8, L-1, M-2, N-3, P-4, Q-5, R-6, S-7, T-8, V-1, W-2, X-3, Y-4, Z-5.
Each vowel is assigned a different symbol as-%, #, $, @, &. So, for vowels
the symbols are - A-$, E-%, I-#, O- @, U-&.

1. (d);

2. (a); The code will be- *3$* 2$7 *$7*

3. (c);

4. (b); The code will be- ‘786%3586 ** 5@3’

Example 3:
Directions (1-3): Study the information carefully answers the questions
given below. (SBI PO Mains-2017)
@ means either hour hand or minute hand is at 8
# means either hour hand or minute hand is at 5
$ means either hour hand or minute hand is at 4
% means either hour hand or minute hand is at 12
& means either hour hand or minute hand is at 2
£ means either hour hand or minute hand is at 3
Note: if two symbols are given than by default first symbol is consider as
hour hand and second one is consider as minute hand. And all time are
consider at PM.

1. If A takes 25 min to reach railway station and his train is scheduled at


#& then at what time should he leave to reach the station 5 minute
earlier?
(a) $% (b) $& (c) &S
(d) $@ (e) £$
Adda247 Publications For any detail, mail us at
5
Publications@adda247.com
Cracker Book for Bank (IBPS | SBI | RRB PO | Clerk) Mains Exams

2. If a train departed from a station at &£ and it takes 2 hours to reach the
destination then when it will reach to the destination?
(a) $£ (b) $% (c) #$
(d) $# (e) £$

3. A person has to catch a train that is scheduled to depart at ‘@%’. It


takes the person 4 hours and 15 minutes to reach the railway station
from his home. At what time should he leave from his home for the
railway station to arrive at the station at least 25 minutes before the
departure of the train?
(a) %@ (b) £$ (c) %+
(d) +@ (e) None of these

Explanations (The Approach):


Directions (1-3):
1. (d); train is scheduled at= 5 hour 10 minute = #&
Time should he leave to reach the station 5 minute earlier= 5 hour
10 minute-(25+5) minute= 4 hour 40 minute

2. (a); 4 hour 15 minute

3. (b); Scheduled time of departure of train= @%= 8:00 PM


Time for travel+ early arrival= 4 hour 15 minutes + 25 minutes = 4
hour 40 minutes
8:00 PM- 4 hour 40 minutes= 3:20 PM= £$

Adda247 Publications For any detail, mail us at


6
Publications@adda247.com
Cracker Book for Bank (IBPS | SBI | RRB PO | Clerk) Mains Exams

Practice Exercise Based on new Pattern

Direction (1-3): Study the following information carefully and answer the
given questions:

In alphabetical series A-Z each letter except vowels is assigned a different


number from 1-5 (for ex- B is coded as 1, C-2………..G-5)and again those
numbers get repeated(for ex- H-1, J-2……..so on).
Also each vowel is assigned a different symbol viz. #, $, %, @, &.
In coded language-
“Solar Power Energy” is coded as – 5&4@4 2&3#4 #1#455
“Need to Change” is coded as – 1##3 1& 21@15#
“Less Economic Revenue” is coded as – 4#55 #2&1&5$2 4#2#1%#
Besides the above example, following operations are to be applied for
coding the words given in the questions below.

(i) If both first and last letter of a word is consonant then the codes of both
the consonant are interchanged.
(ii) If first letter of a word is vowel and last letter is consonant then both
are to be coded as *.
(If the word does not satisfy the conditions given above then the letters
of that word are to be coded as per the directions given above)

1. What can be the code of ‘Nuclear Bomb’?


(a) 4%24#@1 15&1 (b) 4%42#@1 1&51 (c) 4%24#@1 1&51
(d) 4%24@#1 1&51 (e) None of these

2. What can be the code of ‘War and Peace’?


(a) 3@4 *1* 2#@2# (b) 4@3 *1* 2#@2# (c) 4@3 @13 2#@2#
(d) 4@3 *1* 2#2@# (e) None of these

3. What can be the code of ‘Own life’?


(a) &3* 4$4# (b)*3* 44$# (c) &33 4$4#
(d)*3* 4$4# (e)None of these
Adda247 Publications For any detail, mail us at
7
Publications@adda247.com
Cracker Book for Bank (IBPS | SBI | RRB PO | Clerk) Mains Exams

Directions (4-5): Study the following information carefully to answer the


given questions. In a certain code language:-
“mount serious challenge has ended” is written as ‘ nu re ch is de’.
“inform wants credible alternative has ” is written as ‘ di lt wu is fu’.
“Has Serious Challenge Credible” is written as ‘re ch is di’.
“Credible alternative mount inform” is written as ‘ nu lt di fu’.

4. What is the code for ‘alternative’?


(a) fu (b) di (c) lt
(d) re (e) Either (a) and (c)

5. Which of the following words are coded as ‘Serious challenge has


ended’ ?
(a) de ch is wu (b) wu re ch is (c) re is de ch
(d) re is de di (e) None of these

Directions: (6-10): Study the information carefully and answer the


questions given below.
In a certain code language,
“country first casino resorts” is coded as “GL18 ER15 FZ9 GV12”
“policies may deterrent investors” is coded as “HL11 CZ22 IV11 IM10”
“filing process for small businesses” is coded as “FR1 GI12 CL15 EN7 JF9”

6. Which of the following could be the code for “Sanctuary”?


(a) IZ16 (b) DZ18 (c) HZ16
(d) EA18 (e) None of these

7. Which of the following words could be coded as ‘EL13’?


(a) Mountain (b) Courier (c) Power
(d) Banker (e) Police

8. Which of the following could be the code for “annual turnover”?


(a) FM6 HU10 (b) FM6 HF10 (c) KP13 LN13
(d) KP13 NL13 (e) None of these

9. Which of the following words could be coded as ‘GV12’?


(a) Beast (b) Lucky (c) Roller
(d) Returns (e) Ready
Adda247 Publications For any detail, mail us at
8
Publications@adda247.com
Cracker Book for Bank (IBPS | SBI | RRB PO | Clerk) Mains Exams

10. Which of the following could be the code for “Dairy Milk”?
(a) EZ10 DR10 (b) EZ20 DR9
(c) EZ10 DR17 (d) CL10 DR7
(e) EZ20 DR7

Directions (11-13): Study the following information carefully to answer


the given questions.
In a certain code language: -
“tell tale heart” is written as ‘V5M V5F I5U’
“raven black cat” is written as ‘S5O C1L D1U’
“premature burial” is written as ‘Q21F F7M’
“pit and pendulum” is written as ‘Q9U B1E F9N’

11. In the given coding language, which of the following will be the code for
“raven and cats”?
(a) S50 B1E D1U (b) Q9U B1E D1U
(c) C1L B1E D1U (d) S50 Z5T B1E
(e) None of these

12. In the given coding language, which of the following will be the code for
“dark cave”?
(a) D5R V5F (b) D6N Z5L (c) Z5L V5F
(d) Z4L V4L (e) None of these

13. Which of the following words could be coded as “T15E O9U”?


(a) Death Magnetic
(b) Sword Fish
(c) Dark Knight
(d) Sword Night
(e) Last Knight

Directions (14-15): Study the information and answer the following


questions:
In a certain code language
“Words Meaning Enhanced” is coded as “6&14 4@4 1&5”.
“Vowels Support Education” is coded as “3#12 1#18 9%15”
“Sometime Mentioned Tongue” is coded as “14$13 9$5 15&21”.
“Device Made Function” is coded as “8&15 1&3 8#4”.
Adda247 Publications For any detail, mail us at
9
Publications@adda247.com
Cracker Book for Bank (IBPS | SBI | RRB PO | Clerk) Mains Exams

14. What is the code for ‘supported'?


(a) 15#5 (b) 15&5 (c) 14$5
(d) 14&5 (e) None of these

15. What is the code for ‘Baking Powder'?


(a) 5#14 2#5 (b) 2#13 5#5
(c) 5#13 2%5 (d) 5&13 2#5
(e) None of these

Directions (16-20): These questions are based on the following


informations.
In a certain code:
‘severe cash crunch again’ is coded as ‘M*Z S@X V#H S#X’.
‘Finance Ministry has emphasis’ is coded as ‘B$N H&S H$V V©U’.
‘Reserve Bank India has claimed’ is coded as ‘Z*R V©I H&S W©X P@Y’.
16. What will be the code for ‘Emphasis’?
(a) V©U (b) H&S (c) B$N
(d) H$V (e) None of these

17. What will be the code for ‘Bankers’?


(a) Y©H (b) Y&B (c) H©Y
(d) B$Y (e) M35 C12

18. Which of the following will be the code for ‘Renews’?


(a) H#I (b) H&I (c) I#H
(d) I$O (e) None of these

19. Which of the following can be coded as ‘L*A’?


(a) Jaxes (b) Zumbo (c) Judge
(d) April (e) None of these

20. What could be the code for ‘Court has judges’?


(a) G*X H$S Q#H (b) G*X H$S H#Q (c) G@X H$S H#Q
(d) G*X H$S H@Q (e) G*X H&S H#Q
Adda247 Publications For any detail, mail us at
10
Publications@adda247.com
Cracker Book for Bank (IBPS | SBI | RRB PO | Clerk) Mains Exams

Direction (21-25): Study the following information and answer the given
questions:
In alphabetical series each consonant is assigned a different number from
1-7 (for ex- B is coded as 1, C-2……….J-7)and again those numbers get
repeated(for ex- K-1, L-2…….so on).
Besides the above information, following operations are to be applied for
coding the words given in the questions below.
Each letters of the given questions will be coded as per the given
conditions:
I. Vowels appearing before ‘M’ in the Alphabetical series will be coded as
‘**’.
II. Vowels appearing after ‘M’ in the alphabetical series will be coded as
‘$$’.
III. Number immediately preceded by vowel will be coded as ‘#1’.
IV. Number immediately followed by vowel will be coded as ‘@#’.

21. What will possibly be the code for ‘NORMAL’?


(a) ‘#1$$@##2**@# (b) ‘#1$$@##1**@# (c) ‘#3$$@##1**@#
(d) ‘#1$#@##1**@# (e) None of these

22. What will possibly be the code for ‘EMBARKS’?


(a) **@##1**@#12 (b) **@##1**@#31 (c) **@##1**@#41
(d) **@##1**@#11 (e) None of these

23. What will possibly be the code for ‘SMITTLE’?


(a) 1#1**@#2#1** (b) 1#1**@#2#11* (c) 1#1**@#2#1*1
(d) 2#1**@#2#1** (e) None of these

24. What will possibly be the code of ‘ANNUAL’?


(a) **@##1$$**@# (b) *1@##1$$**@# (c) **##@1$$**@#
(d) *2@##1$$**@# (e) None of these

25. What is the code for ‘PROM’?


(a) 5#1$$@# (b) 2#1$$@# (c) 4#1$$@#
(d) 6#1$$@# (e) None of these

Adda247 Publications For any detail, mail us at


11
Publications@adda247.com
Cracker Book for Bank (IBPS | SBI | RRB PO | Clerk) Mains Exams

Directions (26-30): Study the information and answer the following


questions:
In a certain code language
“Seemed peer attend” is coded as “18BV 20BZ 19CW”.
“Arrive Assessing file” is coded as “22BZ 19DZ 12RV”
“Double systems possible” is coded as “19BY 21LY 25CV”.

26. What is the code for ‘support'?


(a) 22DL (b) 21CL (c) 21BL
(d) 22BL (e) None of these

27. What is the code for ‘asking'?


(a) 19ZH (b) 20ZH (c) 21XH
(d) 19HX (e) 19HZ

28. What is the code for ‘Announced'?


(a) 21CZ (b) 21ZC (c) 23BZ
(d) 22CZ (e) None of these

29. What is the code for ‘Less’?


(a) 20BZ (b) 19BV (c) 21CV
(d) 19BZ (e) None of these

30. What is the code for ‘peer loom'?


(a) 15CO 18BC (b) 15BO 18BV
(c) 15BO 18CV (d) 18CO 15BV
(e) None of these

Direction (31-35): Study the following information and answer the given
questions:
In alphabetical series A-Z each letter except vowels is assigned a different
number from 1-6 (for ex- B is coded as 1, C-2………..H-6)and again those
numbers get repeated(for ex- J-1, K-2……..so on).
Also each vowel is coded with different letters viz. g, w, q, c, k.
In coded language-

Adda247 Publications For any detail, mail us at


12
Publications@adda247.com
Cracker Book for Bank (IBPS | SBI | RRB PO | Clerk) Mains Exams

“Election Live” is coded as – g3g24kq5 3k5g


“start new” is coded as – 34c24 5g6
“under process” is coded as – w53g2 62q2g33
Besides the above example, following operations are to be applied for
coding the words given in the questions below.

(i) If first letter is consonant and last letter is vowel then the codes of both
of them will be interchanged.
(ii) If both first and last letter are vowel then they are to be coded as $.
(iii) If first letter of a word is vowel and last letter is consonant then both
are to be coded as @.
(iv) If both first and last letter are consonant then they are to be coded as &.
(If the word does not satisfy the conditions given above then the letters
of that word are to be coded as per the directions given above)

31. What can be the code of ‘global’?


(a) 53q1c& (b) 53q1c3 (c) &q31c&
(d) &3q1c& (e) None of these

32. What can be the code of ‘Title’?


(a) gk434 (b) gk344 (c) 4k43g
(d) kg434 (e) None of these

33. What can be the code of ‘Adviser’?


(a) c35k3g2 (b) c35k3g@
(c) @35k3g@ (d) c53k3g2
(e)None of these

34. What can be the code of ‘Insurance’?


(a) k53w2c52c (b) $53w25c2$
(c) $35w2c52$ (d) $53w2c52$
(e)None of these

35. What can be the code of ‘Marine’?


(a) gc2sk4 (b) gc2k54 (c) 4c2k54
(d) 4c2k5g (e)None of these

Adda247 Publications For any detail, mail us at


13
Publications@adda247.com
Cracker Book for Bank (IBPS | SBI | RRB PO | Clerk) Mains Exams

Directions (36-38): Answer these questions based on the following


information.
In a certain code:
“ review time slot” is coded as - “e%5 o#11 a@27”
“moment question answer” is coded as - “g#29 c&18 q#19”
“tension paper mailing” is coded as - “f%10 b#3 f%15”
“ design layout home” is coded as - “j#6 h%11 c#14”

36. What is the code for “purchase railway”?


(a) k%20 g%1 (b) b#13 h@35 (c) b@13 u%12
(d) k%20 g%2 (e) none of these

37. What is the code for “crosscheck”?


(a) g#21 (b) h#21 (c) p%16
(d) b@14 (e) None of these

38. Which among the following may be coded as “q#9”?


(a) assets (b) recover (c) sector
(d) connect (e) none of these

Directions (39-40): Triangle represents (1) and circle represents (0). If


triangle appears in unit's place, then its value is 1. If it appears in 10's place
its value is doubled to 2 like that it continues. Questions based on this
For example:

39. How will you represent ‘11’ in this code language?


(a) (b)
(c) (d)
(e)
40. What will be the code for ?
(a) 19 (b) 23 (c) 22
(d) 27 (e) 25
Adda247 Publications For any detail, mail us at
14
Publications@adda247.com
Cracker Book for Bank (IBPS | SBI | RRB PO | Clerk) Mains Exams

Solutions

Solutins (1-3): In this new pattern coding decoding each letter, except
vowel, is assigned a number from 1-5 So, B-1, C-2, D-3, F-4, G-5, H-1, J-2, K-
3, L-4, M-5, N-1, P-2, Q-3, R-4, S-5, T-1, V-2, W-3, X-4, Y-5, Z-1.
Each vowel is assigned a different symbol as-%, #, $, @, &. So, for vowels
the symbols are - A-@, E-#, I-$, O- &, U-%.

1. (c); Nuclear Bomb – Condition (i) applied- 4%24#@1 1&51

2. (b); War – Condition (i) applied-4@3


And- Condition (ii) applied- *1*
Peace- No condition applied- 2#@2#

3. (d); Own- Condition (ii) applied- *3*


Life- No Condition applied- 4$4#

Solutions (4-5):
WORDS CODES
Mount nu
Has is
Serious/Challenge re/ch
Credible di
Alternative Lt/fu
Wants Wu
Inform Fu/lt
Ended de

4. (e); 5. (c);

Solutions (6-10): This coding decoding question is based on the latest


pattern, following logic are applied in this question to decode the code.
(i) The first letter of the code represents an alphabet which has the same
place value as the number of letters in the given word.
(ii) The second letter of the code is the reverse of the second letter of the
word(A-Z, B-Y…..)
Adda247 Publications For any detail, mail us at
15
Publications@adda247.com
Cracker Book for Bank (IBPS | SBI | RRB PO | Clerk) Mains Exams

(iii) The number at the end of the code is the difference between the place
value of the last letter of the word and the total number of letters in the
word.

6. (a); 7. (c); 8. (b);


9. (d); 10. (e);

Solutions (11-13): This is question of Coding-Decoding based on new


pattern. In these questions, following logic is applied to decode the code:-
If the total number of letters in the word is even,
1st letter of the code:- Reverse(A-Z, B-Y….) of the greatest(according to the
alphabetical series) vowel in the word.
2nd letter of the code:- Total number of letters in the word + 1
3rd letter of the code:- Next letter(according to the alphabetical series) of
the last letter of the word.

If the total number of letters in the word is odd,


1st letter of the code:- Next letter(according to the alphabetical series) of the
first letter of the word.
2nd letter of the code:- Rank/Place value of the greatest vowel in the word.
3rd letter of the code:- Next letter(according to the alphabetical series) of
the last letter of the word.

Adda247 Publications For any detail, mail us at


16
Publications@adda247.com
Cracker Book for Bank (IBPS | SBI | RRB PO | Clerk) Mains Exams

11. (d); 12. (c); 13. (d);

Solutions (14-15): Let us understand the logic behind the given coding
decoding:
For the I digit of the code – I Number in the code will be the difference of
the of place value of the first letter and last letter present in the word.
For the symbol
For different number of vowels present in the word, the code will be
accordingly assigned to it.

No. of vowels in the word Code


1 @
2 #
3 &
4 $
5 %

For the last digit of the code – The place value of the second last letter
present in the word.

14. (b); 15. (a);

Solutions (16-20): The given words are coded as per following pattern:
(i) First letter of the code represents the opposite letter of the last letter of
the given word.
For example. Again- ‘N’=M
(ii) Last letter of the code represents the opposite letter of the first letter of
the given word.
Adda247 Publications For any detail, mail us at
17
Publications@adda247.com
Cracker Book for Bank (IBPS | SBI | RRB PO | Clerk) Mains Exams

For example. Again- ‘A’=Z


(iii) The symbol of the code is depending on the total number of letters in
the given word.
No. of letters - Symbol
3-&
4-@
5-*
6-#
7–©
8-$
For ex. Again – The code is ‘M*Z’.

16. (d); 17. (c); 18. (a);


19. (b); 20. (e);

Solutions (21-25):
Logic: The different number codes for all the consonant as per the given
condition are,

Step 1: The consonants of the word ‘NORMAL’ are to be coded as the


number allotted to the consonant:

Step 2: The numbers immediately preceded and followed by the vowels are
to be coded as per the given conditions;
So, the code for consonant for word ‘NORMAL’ is coded as ‘4O73A2’,
numbers 4 and 7 is immediately followed and preceded
respectively by ‘O’ so, ‘4’ is coded as ‘#1’ and ‘7’ is coded as ‘@#’.
Similarly, ‘3’ and ‘2’ is immediately followed and preceded
respectively by ‘A’ so, ‘3’ is coded as ‘#1’ and ‘2’ is coded as ‘@#’.

Adda247 Publications For any detail, mail us at


18
Publications@adda247.com
Cracker Book for Bank (IBPS | SBI | RRB PO | Clerk) Mains Exams

Step 3: Now the vowels are to be coded as per the given conditions, as ‘O’
comes after ‘M’ in the alphabetical series so ‘O’ is coded as ‘$$’ and
‘A’ comes before ‘M’ in the alphabetical series so, ‘A’ is to be coded
as ‘**’.

So, the final code for the word ‘NORMAL’ is ‘#1$$@##1**@#’.

21. (b); Therefore, the code for the word ‘NORMAL’ is ‘#1$$@##1**@#’.

22. (d) Step 1: The consonants of the word ‘EMBARKS’ are to be coded as
the number allotted to the consonant:

Step 2: The numbers immediately preceded and followed by the


vowels are to be coded as per the given conditions;
So, the code for consonant for word ‘EMBARKS’ is coded as
‘E31A711’, number ‘3’ is followed by vowel so the code for ‘3’ is
‘@#’ and numbers ‘1’ and ‘7’ is immediately preceded and followed
respectively by ‘A’ so, ‘1’ is coded as ‘#1’ and ‘7’ is coded as ‘@#’.
But the numeric code of ‘K’ and ‘S’ is ‘1’ is neither followed by nor
preceded by any vowel. Hence, there code will remain the same.

Adda247 Publications For any detail, mail us at


19
Publications@adda247.com
Cracker Book for Bank (IBPS | SBI | RRB PO | Clerk) Mains Exams

Step 3: Now the vowels are to be coded as per the given conditions,
as ‘E’ comes before ‘M’ in the alphabetical series so ‘E’ is coded as ‘**’
and ‘A’ comes before ‘M’ in the alphabetical series so, ‘A’ is to be
coded as ‘**’.

So, the final code for the word ‘EMBARKS’ is ‘**@##1**@#11’.

23. (a); The code for ‘’SMITTLE’ is ‘1#1**@#2#1**’.

24. (a); The code for ‘ANNUAL’ is ‘**@##1$$**@#’.

25. (a);

Solutions (26-30):
Let us understand the logic behind the given coding decoding:
For the digit of the code – Number in the code will be the place value of the
highest place value of letter present in the word.
For the first letter of the code-

Adda247 Publications For any detail, mail us at


20
Publications@adda247.com
Cracker Book for Bank (IBPS | SBI | RRB PO | Clerk) Mains Exams

Case-1
If the given word has some common letter then the code will be according
to the given order:

No. of common letter in the word Code


2 B
3 C
4 D

For ex: Arrive -In this word two common letters ‘r’ and in Assessing has
four common letters ‘s’. So the first letter of code for Arrive is ‘B’
and for Assessing is ‘D’.

Case II
If the given word has no common letter then first letter of the code will be
coded as opposite letter of the second letter of the word.
For the last letter of the code – The opposite letter of the smallest place
value letter present in the word according to the English alphabet.

26. (c); 27. (e); 28. (a);


29. (b); 30. (b);

Direction (31-35):
In this new pattern coding decoding each letter, except vowel, is assigned a
number from 1-6 So, B-1, C-2, D-3, F-4, G-5, H-6, J-1, K-2, L-3, M-4, N-5, P-6,
Q-1, R-2, S-3, T-4, V-5, W-6, X-1, Y-2, Z-3.
Also each vowel is assigned different letters. So, for vowels the letters are -
A-c, E-g, I-k, O-q, U-w.

31. (d); global- Condition (iv) applied- &3q1c&

32. (a); Title- Condition (i) applied- gk434

33. (c); Adviser- Condition (iii) applied- @35k3g@

34. (d); Insurance- Condition (ii) applied $53w2c52$


Adda247 Publications For any detail, mail us at
21
Publications@adda247.com
Cracker Book for Bank (IBPS | SBI | RRB PO | Clerk) Mains Exams

35. (b); Marine- Condition (i) applied- gc2k54

Solutions (36-38):

36. (a); “purchase railway” ------ “k%20 g%1”

37. (b); “crosscheck” ------ “h#21”

38. (d); “q#9” --- “connect”

Solutions (39-40):
39. (e); As given in the Directions
If triangle appears in unit's place then its value is 1. If it appears in
10's place its value is doubled to 2 like that it continues.
Then to represent the 11 by 2^3 0^2 2^1 2^0
= 8+0+2+1 = 11

Adda247 Publications For any detail, mail us at


22
Publications@adda247.com
Cracker Book for Bank (IBPS | SBI | RRB PO | Clerk) Mains Exams

40. (b); As given in the Directions If triangle appears in unit's place then its
value is 1. If it appears in 10's place its value is doubled to 2 like
that it continues
= 2^4 0^3 2^2 2^1 2^0
=16+0+4+2+1 = 23

Key Points to Remember for New Pattern Questions


➢ To expect the unexpected shows a thoroughly modern
intellect, it completely goes with the coding decoding now-
a-days.
➢ The coding decoding has changed completely as its pattern
has gone through lots of changes and the logic behind it, is
always a mystery to go around with. Just go through the
given pattern completely otherwise there will be a chance of
ambiguity in finding the logic.
➢ Recent exam consist of different logics as you may see that
the logic varies as per the vowels or consonants present in
the word.
➢ Sometimes the logic goes around the first or last letter of the
word so try to initiate accordingly.
➢ It is not necessary that all the words present in the word
may follow same logic as it may varies according to the
number of letters present in word as if it is odd one then
there may be a different logic for it as it is for the even one.

Adda247 Publications For any detail, mail us at


23
Publications@adda247.com
Cracker Book for Bank (IBPS | SBI | RRB PO | Clerk) Mains Exams

1 Adda247 Publications For any detail, mail us at


Publications@adda247.com
Cracker Book for Bank (IBPS | SBI | RRB PO | Clerk) Mains Exams

Chapter

6 Syllogism

BEST APPROACH TO SOLVE THE QUESTIONS


Every day changing pattern of examination makes us realize that any
topic has its own importance and you may see some new type of questions
introduced in it at any time, the same goes with the syllogs. We have seen
some changes that has already happed as reverse syllogism, does not follow
and so on. And in future exams also we may see some changes as some
coded syllogism or any thing new. Here are some glimpse of the already
seen syllogism question in the given below example.

Example-1:
In each of the questions below are given three statements followed by three
conclusions number I, II and III. You have to take the given statements to be
true even if they seem to be at variance from commonly known facts. Read
the entire conclusion and then decide which of the given conclusions
logically follows from the given statements disregarding commonly known
facts. (IBPS Clerk Mains 2017)

1. Statements: All pigs are Zebra.


All Zebra are Rabbits.
No Ant is are Pig.
Conclusions: I. Some rabbit is Ant.
II. Some Zebra are Ant is a possibility.
III. Some Ant are Pig.
(a) Only I follows (b) Only II follows (c) Only III follows
(d) All follow (e) None of these

2. Statements: Some bat are pen.


Some pen are Shirt.
No shirt is Coat.
Conclusions: I. Some Coat is Pen.
II. Some Coat is shirt is a possibility.
III. All pen can be Coat
(a) None follows (b) only II follows (c) only III follows
(d) only I follows (e) only either II or III follow
2 Adda247 Publications For any detail, mail us at
Publications@adda247.com
Cracker Book for Bank (IBPS | SBI | RRB PO | Clerk) Mains Exams

3. Statements: All Men are Rock.


Some men are pearl.
No Rock is Angel.
Conclusions: I. Some men are not Angel.
II. All Pearl can be Angel is a possibility.
III. Some Angel can be Pearl is a possibility.
(a) only I and II follow (b) only I and III follow
(c) only II and III follow (d) All I, II and III follow
(e) None of these

1. (b);

2. (a);

3. (b);

Example-2:
In each question below are given three statements followed by four
conclusions numbered I, II, III and IV. You have to take the given statements
to be true even if they seem to be at variance with commonly known facts.
Read all the conclusions and then decide which of the given conclusions
logically follows from the given statements, disregarding commonly known
facts. (RBI Grade-B 2016)

3 Adda247 Publications For any detail, mail us at


Publications@adda247.com
Cracker Book for Bank (IBPS | SBI | RRB PO | Clerk) Mains Exams

1. Statements: A. No spoons are plates


B. All plates are bowls
C. All bowls are utensils
Conclusions: I. No spoons are bowls
II. No spoons are utensils
III. Some bowls are not plates
IV. Some utensils are not plates
(a) Only I follow (b) Only III follows (c) Only IV follows
(d) None follows (e) None of these

2. Statements: A. Some roses are leaves


B. Some leaves are plants
C. Some plants are flowers
Conclusions: I. No roses are flowers
II. No plants are roses
III. Some leaves are flowers
IV. Some plants are roses
(a) None follows (b) Only I and II follow
(c) Only II and III follow (d) All follow (e) None of these

3. Statements: A. Some sweets are chocolates


B. All chocolates are fruits
C. Some fruits are vegetables
Conclusions: I. Some sweets are vegetables
II. Some chocolates are vegetables
III. Some fruits are not sweets
IV. Some fruits are not vegetables
(a) Only I follow (b) Only II follows (c) Only III follows
(d) None follows (e) None of these

4 Adda247 Publications For any detail, mail us at


Publications@adda247.com
Cracker Book for Bank (IBPS | SBI | RRB PO | Clerk) Mains Exams

4. Statements: A. No reynolds are jetters


B. No jetters are finegrips
C. All finegrips are pens
Conclusions: I. No jetters are reynolds
II. Some jetters are not finegrips
III. Some finegrips are pens
IV. Some pens are finegrips
(a) All follows
(b) Only I, II and III follow
(c) Only II, III and IV follow
(d) Only I, II and IV follow
(e) None of these

5. Statements: A. Some officers are gentlemans


B. All graduates are officers
C. All intelligents are graduates
Conclusions: I. Some gentlemans are officers
II. All intelligents are officers
III. Some officers are not intelligents
IV. All officers are intelligents
(a) Only I, II and III follow (b) Only I, II and IV follow
(c) Only I and II follow (d) Only I and III follow
(e) None of these

1. (d);

5 Adda247 Publications For any detail, mail us at


Publications@adda247.com
Cracker Book for Bank (IBPS | SBI | RRB PO | Clerk) Mains Exams

2. (e);

3. (d);

4. (a);

5. (e);

6 Adda247 Publications For any detail, mail us at


Publications@adda247.com
Cracker Book for Bank (IBPS | SBI | RRB PO | Clerk) Mains Exams

Practice Exercise Based on new Pattern

Direction (1-3): In the following questions, the symbols #, @, $ and © are


used with the following meanings as illustrated below. Study the following
information and answer the given questions. In each of the questions given
below statements are followed by some conclusions. You have to take the
given statements to be true even if they seem to be at variance from
commonly known facts. Read all the conclusions and then decide which of
the given conclusions logically does not follows from the given statements
disregarding commonly known facts.
P#Q – All P is Q.
P@Q - Some Q is P.
P©Q – No P is Q.
P$Q – Some P is not Q.

1. Statements: Rat @ Cat # Dog © Fish @ Goat $ Hen


Conclusions: (a) Rat @ Dog (b) Goat @ Fish
(c) Cat © Fish (d) Rat $ Fish
(e) Fish $ Hen
2. Statements: Air @ Car # Drive $ Exit © Ride @ Air
(a) Air $ Exit (b) Drive @ Air
(c) Car @ Ride (d) Car @ Drive
(e) Exit $ Ride

3. Statements: Rubber © Pencil # Eraser @ Scale # Copy $ Book


Conclusions: (a) Scale @ Eraser (b) Eraser @ Copy
(c) Eraser @ Pencil (d) Eraser $ Rubber
(e) Rubber $ Book

Directions (4-5): In each group of questions below are two conclusions


followed by five set of statements. You have to choose the correct set of
statements that logically satisfies given conclusions. Given statements to be
true even if they seem to be at variance from commonly known facts.

7 Adda247 Publications For any detail, mail us at


Publications@adda247.com
Cracker Book for Bank (IBPS | SBI | RRB PO | Clerk) Mains Exams

4. Conclusion: At least some Caps being Pens is a possibility.


Some Copies are Caps.
Statements:
Statements-1: All Caps are Copies. All Caps are Papers. No Cap is a Pen.
Statements-2: All Caps are Copies. All Copies are Papers. No Paper is a
Pen.
Statements-3: Some Caps are Copies. Some Caps are Papers. No Cap is
a Pen.
Statements-4: Some Caps are Copies. All Copies are Papers. No Cap is a
Pen.
Statements-5: All Caps are Copies. Some Caps are Papers. No Paper is a
Pen.
(a) Only Statements – 1
(b) Both Statements – 2 and 4
(c) Only Statements – 3
(d) Both Statements – 3 and 5
(e) Only Statements – 5

5. Conclusion: Some Reds being Black is a possibility


Some Blue being Red is a possibility
Statements:
Statements-1: All Black are Pink. Some Red are Pink. No Blue is Red.
Statements-2: All Black are Pink. No Red is Pink. All Blue are Black.
Statements-3: Some Black are Pink. Some Red are Pink. No Blue is Red.
Statements-4: Some Black are Pink. Some Red are Pink. No Pink is
Blue.
Statements-5: Some Black are Pink. Some Red are Pink. All Blues are
Black.
(a) Only Statements – 1
(b) Both Statements – 2 and 3
(c) Only Statements – 4
(d) Both Statements – 4 and 5
(e) Only Statements – 5
8 Adda247 Publications For any detail, mail us at
Publications@adda247.com
Cracker Book for Bank (IBPS | SBI | RRB PO | Clerk) Mains Exams

Directions (6-8): In each of the questions below. Some statements are


given followed by conclusions/group of conclusions. You have to assume all
the statements to be true even if they seem to be at variance from the
commonly known facts and then decide which of the given conclusions
logically does not follows from the information given in the statements.
6. Statements: Some air is ball No ball is cat
Some cat is dog All dog is egg.
Conclusions: (a) some dogs are not ball.
(b) some egg are not ball
(c)All air is egg is a possibility.
(d) All egg can be ball.
(e) some air is dog is a possibility.

7. Statements: All watch is clock All clock is table


No watch is pen No clock is chair.
Conclusions: (a) Some table are watch
(b) Some table can be chair.
(c) No watch is chair
(d) No pen is chair.
(e) some table is not pen.

8. Statements: All plastic is rubber Some plastic is Teflon.


Some Teflon is cotton.
All cotton is woolen.
Conclusions: (a) some woolen is Teflon.
(b) No plastic is woolen is a possibility.
(c) All Teflon can be woolen.
(d) some Teflon is rubber.
(e) All rubber is plastic.

9 Adda247 Publications For any detail, mail us at


Publications@adda247.com
Cracker Book for Bank (IBPS | SBI | RRB PO | Clerk) Mains Exams

Directions (9-10): In each group of questions below are two conclusions


followed by five set of statements. You have to choose the correct set of
statements that logically satisfies given conclusions. Given statements to be
true even if they seem to be at variance from commonly known facts.

9. Conclusion-Some Students are Teachers. Some Principal are not


Students.
(a) Statement- No Study is School. All Teachers is Students. Some
Teachers is Study. Some School are Principal.
(b) Statement- No School is Study. Some Teachers is Students. Some
Teachers is Study. Some School are Principal.
(c) Statement- All Students is Teachers. Some Teachers is School. Some
Study are Principal. Some School is Study.
(d) Statement- All Students is Teachers. Some Teachers is Study. No
Students is School. Some School are Principal.
(e) Statement- Some Teachers is Students. No Teachers is School.
Some School is Study. Some School are Principal.

10. Conclusion: Some Enjoy is not Comedy. Some Fun are Circus.
(a) Statement: Some Joke is Enjoy. No Enjoy is Fun. Some Fun is Circus.
Some Joke are Comedy.
(b) Statement - All Fun is Circus. No Circus is Comedy. Some Joke are
Comedy. Some Joke is Enjoy.
(c) Statement - All Comedy is Circus. Some Joke are Circus. All Joke is
Fun. No Enjoy is Fun.
(d) Statement - No Circus is Joke. Some Joke are Fun. Some Fun is
Circus. Some Enjoy are Comedy.
(e) Statement -Some Joke is Enjoy. Some Enjoy are Circus. All Circus
are Fun. All Comedy are not fun.

Directions (11-13): This question consists of four statements followed by


five conclusions. Consider the given statements to be true even if they seem
to be at variance with commonly known facts. Read all the conclusions and
then decide which of the given conclusions does not logically follow from
the given statements using all statements together.

10 Adda247 Publications For any detail, mail us at


Publications@adda247.com
Cracker Book for Bank (IBPS | SBI | RRB PO | Clerk) Mains Exams

11. Statements: Some Pens are Papers.


All copies are Pencil
No Papers is a pencil
All pencil is Inks.
Conclusions: (a) Some Copies are Inks.
(b) All Inks being Pens is a possibility.
(c) At least some Pens are not Copies.
(d) All Papers can be Inks.
(e) Some Pens are Copies.

12. Statements: Some Book are Note.


All Book are Stationery.
All Note are Stationery.
No Book are Test.
Conclusions: (a) Some Stationery are not Test.
(b) All Test being Stationery is a possibility.
(c) No Test is Stationery.
(d) No Test being Stationery is a possibility.
(e) Some Note are not Test.
13. Statements: All blades are sharp.
Some blades are blunt.
No blunt is a sword.
Some sword are sharp.
Conclusions: (a) Some sharp are not blunt
(b) All blunt being sharp is a possibility.
(c) some blades are not sword.
(d) All sword can be blade.
(e) All sharp can be blunt.
Directions (14-15): In each of the questions below are given three
statements followed by three conclusions number I, II and III. You have to
take the given statements to be true even if they seem to be at variance
from commonly known facts. Read the entire conclusion and then decide
which of the given conclusions logically follows from the given statements
disregarding commonly known facts.

11 Adda247 Publications For any detail, mail us at


Publications@adda247.com
Cracker Book for Bank (IBPS | SBI | RRB PO | Clerk) Mains Exams

14. Statements: All PO are graduate.


All clerk are PO.
Some clerk are intermediate.
Conclusions: I. Some clerk are graduate.
II. Some intermediate are graduate is a possibility.
III. All intermediate can be graduate
(a) only I and II follow (b) only I and III follow
(c) only II and III follows (d) All I, II and III follows
(e) None of these

15. Statements: Some Hotels are 5-Star.


No 5-Star are Cheap.
Some Cheap are luxurious.
Conclusions: I. Some luxurious are hotel.
II. No hotel are luxurious.
III. All hotel are luxurious.
(a) None follows (b) only II follows
(c) only III follows (d) only I follows
(e) only either I or II follow

Direction (16-18): In the following questions, the symbols *, +, - and / are


used with the following meanings as illustrated below. Study the following
information and answer the given questions. In each of the questions given
below statements are followed by some conclusions. You have to take the
given statements to be true even if they seem to be at variance from
commonly known facts. Read all the conclusions and then decide which of
the given conclusions logically does not follows from the given statements
disregarding commonly known facts.
A*B – All A is B.
A+B - Some B is A.
A/B – No B is A.
A-B – Some A is not B.
?- possibility case
Note: if ? is placed after any of the symbols mentioned above then it will be
considered as possibility case of the symbol. E.g. A+?B means some B being
A is a possibility.

12 Adda247 Publications For any detail, mail us at


Publications@adda247.com
Cracker Book for Bank (IBPS | SBI | RRB PO | Clerk) Mains Exams

16. Statements: mall+ plaza* market- complex


Conclusions: (a) mall- complex
(b) mall+ market
(c) market+ plaza
(d) complex+? market
(e) plaza-? complex

17. Statements: road/traffic/light*crossing


Conclusions: (a) road+? light
(b) crossing- light
(c) traffic*? crossing
(d) road+? crossing
(e) crossing-traffic

18. Statements: pen+ eraser* paper; pen*pencil


Conclusions: (a) paper+? pencil
(b) paper+ pencil
(c) pencil*? paper
(d) pencil+ eraser
(e) paper+ pen

Directions (19-20): In each group of questions below are two conclusions


followed by five set of statements. You have to choose the correct set of
statements that logically satisfies given conclusions. Given statements to be
true even if they seem to be at variance from commonly known facts.

19. Conclusion-Some Light are Dark. Some Dull are not Light.
Statement I- No Bright is Glow. All Dark is Light. Some Dark is
Bright. Some Glow are Dull.
Statement II- No Glow is Bright. All Dark is Light. Some Dark is Glow.
Some Light are Dull.
Statement III- All Light is Dark. Some Dark is Glow. Some Bright are
Dull. Some Glow is Bright.
Statement IV- All Light is Dark. Some Dark is Bright. No Light is Glow.
Some Glow are Dull.
Statement V- Some Dark is Light. No Dark is Glow. Some Glow is
Bright. Some Glow are Dull.
13 Adda247 Publications For any detail, mail us at
Publications@adda247.com
Cracker Book for Bank (IBPS | SBI | RRB PO | Clerk) Mains Exams

20. Conclusion-Some Above not being Upper is a possibility. Some Below


are Odd.
Statement I- Some Above is Below. No Below is Even. Some Even is
Odd. Some Above are Upper.
Statement II- All Even is Odd. No Odd is Upper. Some Above are
Upper. Some Above is Below.
Statement III- All Upper is Odd. Some Above are Odd. All Above is
Even. No Below is Even.
Statement IV- No Odd is Above. Some Above are Even. Some Even is
Odd. Some Below are Upper.
Statement V- Some Above is Below. Some Below are Odd. All Odd are
Even. All Upper are not even.

Direction (21-22): In each group of questions below are some conclusions


followed by five set of statements. You have to choose the correct set of
statements that logically satisfies given conclusions. Given statements to be
true even if they seem to be at variance from commonly known facts.

21. Conclusion: Some Science are not English. Some Reasoning can be
Computer.
Statements:
(a) All Computer are Math. No Math is reasoning. Some Reasoning are
English. Some Science are Computer.
(b) Some Math are English. No English is Computer. No Computer is
Reasoning. Some Reasoning is Science.
(c) No Computer is Science. Some Science are English. No English is
Reasoning. Some Reasoning is Math.
(d) Some Computer are Science. All Science is Math. No Math are
English. Some English are Reasoning.
(e) All English are Math. No Math is Reasoning. Some Reasoning are
Computer. Some Science are Computer.

14 Adda247 Publications For any detail, mail us at


Publications@adda247.com
Cracker Book for Bank (IBPS | SBI | RRB PO | Clerk) Mains Exams

22. Conclusion: Some Physics can never be Geography. Some Biology are
not Geography.
Statements:
(a) All Biology are Geography. No Geography is History. Some History
are Physics. Some Chemistry are Biology.
(b) Some Biology are Chemistry. No Geography is History. Some
History are Physics. All Physics are Biology.
(c) No Biology is Chemistry. Some Chemistry are Physics. No Physics
is History. Some History is Geography.
(d) Some Biology are Geography. All Geography is History. No History
are Physics. Some Chemistry are Biology.
(e) Some Geography are Physics. No Physics is Biology. No Biology is
History. Some History is Chemistry.

Directions (23-25): In each question below are given some statements


followed by some conclusions numbered I, II, III and IV. You have to take
the given statements to be true even if they seem to be at variance with
commonly known facts. Read all the conclusions and then decide which of
the given conclusions definitely does not logically follow from the given
statements, disregarding commonly known facts. Give answer

23. Statements: Some Pepsi are Cocacola.


No Cocacola is a Mirinda.
All Mirinda are Dew.
Conclusions: I. Some Cocacola not being Mirinda is a possibility.
II. Some Dew are not Cocacola.
III. All Pepsi are Mirinda.
IV. Some Pepsi can be Dew.
(a) Only I & II do not follow
(b) Only III & IV do not follow
(c) Only III does not follow
(d) All follow except IV
(e) None of these
15 Adda247 Publications For any detail, mail us at
Publications@adda247.com
Cracker Book for Bank (IBPS | SBI | RRB PO | Clerk) Mains Exams

24. Statements: All Puma are Reebok.


All Adidas are Reebok.
Some Adidas are Woodland.
Conclusions: I. No Puma is Adidas.
II. Some Reebok being Woodland is a possibility.
III. Some Reebok are Puma.
IV. Some Reebok are Adidas.
(a) Only I & III do not follow
(b) Only II does not follow
(c) All follow except III
(d) Only II & I do not follow
(e) All follow except I
25. Statements: Some tibbet are China
All china are nepal.
No nepal is bhutan.
Conclusions: I. Some bhutans are not tibbet.
II. All tibbet being nepal is a possibility.
III. At least some bhutans are nepal.
IV. All china can be nepal.
(a) II & IV do not follow
(b) I & IV do not follow
(c) I & III do not follow
(d) II & III do not follow
(e) None of these

Directions (26-28): In each of the questions below are given some


statements followed by some conclusions. You have to take the given
statements to be true even if they seem to be at variance from commonly
known facts. Read the entire conclusion and then decide which of the given
conclusions logically follows from the given statements disregarding
commonly known facts.

26. Statements: No Boys are Girls.


All Boys are Graduate
All Girls are graduate
Some Girls are IAS
16 Adda247 Publications For any detail, mail us at
Publications@adda247.com
Cracker Book for Bank (IBPS | SBI | RRB PO | Clerk) Mains Exams

Conclusions: I. Some Boys are not IAS.


II. All IAS can be Boys.
III.All Graduate being IAS ia a possibility.
(a) Only I follows
(b) Only II follows
(c) Only III follows
(d) All follow
(e) None of these

27. Statements: All milk is water.


Some milk is Tea.
No Coffee is Tea.
Conclusions: I. Some milk is Coffee.
II. No milk is Coffee
III. All milk being coffee is a possibility.
(a) None follows
(b) only II follows
(c) only III and either I or II follow
(d) only I follows
(e) only either I or II follow

28. Statements: Some Pant are Shirt.


Some Shirt are Suit.
No Suit is Button.
Conclusions: I. Some shirt is not Button.
II. Some Pant is Button.
III. Some Buttton is not Shirt
(a) Only I follows (b) Only II follows (c) Only III follows
(d) All follow (e) None of these

17 Adda247 Publications For any detail, mail us at


Publications@adda247.com
Cracker Book for Bank (IBPS | SBI | RRB PO | Clerk) Mains Exams

Directions (29-30): In each group of questions below are two conclusions


followed by five set of statements. You have to choose the correct set of
statements that logically satisfies given conclusions. Assume the given
statements to be true even if they seem to be at variance from commonly
known facts. Mark your answer as,

29. Conclusion: I. All mango can be papaya.


II. Some grapes are papaya.
(a) All grapes are guavas. Some mangoes are grapes. Some guavas are
papaya.
(b) Some grapes are guavas. No mangoes are grapes. Some guavas are
papaya.
(c) Some guavas are papaya. No grapes are guavas. No mangoes are
grapes.
(d) No mangoes are grapes. All guavas are papaya. Some grapes are
guavas.
(e) No grapes are guavas. No mangoes are grapes. No guavas are
papaya.

30. Conclusions: All cups being plate is a possibility. Some glasses can be
cups.
Statements:
(a) Some plates are Steels. All Steels are glasses. All cups are plate.
(b) Some plates are Steels. Some glasses are Steels. No plate is a cup.
(c) Some glasses are Steels. Some plates are Steels. Some plates are not
cups.
(d) All glasses are plates. Some Steels are glasses. No cup is a glass.
(e) None of these

Directions (31-35): Some statements are given followed by some


conclusions. You have to consider the statements to be true even if they
seem to be at variance from commonly known facts. You have to decide
which of the following conclusions follow from the given statements:

31. Statements: All bags are books.


Some books are pencils.
No pencils are boxes.
18 Adda247 Publications For any detail, mail us at
Publications@adda247.com
Cracker Book for Bank (IBPS | SBI | RRB PO | Clerk) Mains Exams

Conclusion: (i) Some books are not boxes


(ii) Some books being boxes is a possibility
(iii) Some bags are not boxes
(a) Only (i) follow (b) Only (ii) and (iii) follow
(c) Only (i) and (ii) follow (d) All follow
(e) None of these

32. Statements: Some dolls are barbie.


Some barbie are famous.
All famous are player.
No famous is actor.
Some actors are barbie.
Conclusion: (i) Some barbie are not actors
(ii) Some barbie are players
(iii) Some players are famous
(a) Only (i) follow (b) Only (ii) and (iii) follow
(c) Only (i) and (ii) follow (d) All follow
(e) None of these

33. Statements: All cats are dogs.


Some dogs are not cows.
No cow is a rat.
Some rats are dogs.
No rat is horse.
Conclusion: (i) All cats are horses.
(ii) Some dogs being cows is a possibility.
(iii) All cats being cows is a possibility.
(a) Only (i) follow (b) Only (ii) and (iii) follow
(c) Only (i) and (ii) follow (d) All follow
(e) None of these

34. Statements: All good are bad.


All bad are best.
All best are worst.
Some worst is fair.
No best is normal.
Some normal are worst.
19 Adda247 Publications For any detail, mail us at
Publications@adda247.com
Cracker Book for Bank (IBPS | SBI | RRB PO | Clerk) Mains Exams

Conclusion: (i) All fair being normal is a possibility


(ii) Some best are fair.
(iii) Some normal being good is a possibility
(a) Only (i) follow (b) Only (ii) and (iii) follow
(c) Only (i) and (iii) follow (d) All follow
(e) None of these

35. Statements: All eyes are legs.


No eyes are nose.
All noses are hands.
Some hands are legs.
No legs are ears.
Conclusion: (i) Some hands are ears
(ii) Some hands are not nose
(iii) Some hands are not ears
(a) Only (i) follow
(b) Either (i) or (ii) and (iii) follow
(c) Either (i) or (iii) and (ii) follow
(d) All follow
(e) None of these

Directions (36-38): In each group of questions below are two conclusions


followed by five set of statements. You have to choose the correct set of
statements that logically satisfies given conclusions.
Given statements to be true even if they seem to be at variance from
commonly known facts.

36. Conclusions: All summer being autumn is a possibility.


Some Autumn are Winters is a possibility
Statements:
Statements I: All summer are winters. No winter is Spring. All Springs
are Autumn.
Statements II: No Winter is summer. No summer is Autumn. No
Autumn is spring.
Statements III: All summer are winters. No summer is Spring. No winter
is autumn.
20 Adda247 Publications For any detail, mail us at
Publications@adda247.com
Cracker Book for Bank (IBPS | SBI | RRB PO | Clerk) Mains Exams

Statements IV: All Winters are summer. No winter is Spring. No summer


is Autumn.
Statements V: All summer are winters. No winter is Autumn. All
Springs are Autumn.
(a) Only Statements I (b) Only Statements II
(c) Only Statements III (d) Only Statements IV
(e) Only Statements V

37. Conclusions: At least some samsung being motorola is a possibility.


Some apple are samsung.
Statements:
Statements I: All Samsung are Apple. All samsung are lenovo. No
samsung is a motorola.
Statements II: All Samsung are Apple. All Apple are lenovo. No lenovo
is a motorola.
Statements III: Some Samsung are Apple. Some samsung are lenovo. No
samsung is a motorola.
Statements IV: Some Samsung are Apple. All Apple are lenovo. No
samsung is a motorola.
Statements V: All Samsung are Apple. Some samsung are lenovo. No
lenovo is a motorola.
(a) Only Statements I (b) Only Statements II
(c) Only Statements III (d) Only Statements IV
(e) Only Statements V

38. Conclusions: At least some purse are bag.


Some moneys being bag is a possibility
Statements:
Statements I: All bag are purse. Some purse are wallet. No money is
bag.
Statements II: No bag is purse. Some purse are moneys. No money is
wallet.
Statements III: All bag are purse. Some purse are moneys. No money is
wallet.

21 Adda247 Publications For any detail, mail us at


Publications@adda247.com
Cracker Book for Bank (IBPS | SBI | RRB PO | Clerk) Mains Exams

Statements IV: No bag is purse. Some purse are moneys. No money is


wallet.
Statements V: All bag are purse. Some purse are wallet. No money is
bag.
(a) Only Statements I (b) Only Statements II
(c) Only Statements III (d) Only Statements IV
(e) Only Statements V

Directions (39-40): In the questions below are given two conclusions


followed by four set of statements. You have to choose the correct set of
statements that logically satisfies given conclusions. Assume the given
statements to be true even if they seem to be at variance from commonly
known facts.

39. Conclusions: Some Purse is wallet. Some money is not purse.


Statements:
(a) Some ATM is wallet. No wallet is money. All money is coin. No coin
is purse.
(b) All ATM is wallet. Some wallet is money. No money is coin. Some
coin is purse.
(c) Some ATM is purse. All purse is money. No money is wallet. All
wallet is coin.
(d) All ATM is money. Some money is coin. No coin is purse. Some
wallet is purse.
(e) None of these.

40. Conclusions: Some wall is paint. Some fan is not paint.


Statements:
(a) No window is fan. All wall is fan. No floor is paint. Some paint is
brush.
(b) All window is fan. No window is brush. Some brush is wall. No wall
is paint.
(c) Some window is brush. All brush is floor. No floor is paint. All paint
is fan.

22 Adda247 Publications For any detail, mail us at


Publications@adda247.com
Cracker Book for Bank (IBPS | SBI | RRB PO | Clerk) Mains Exams

(d) All wall is window. Some paint is wall. No paint is brush. All fan is
brush.
(e) None of these.

Directions (41-45): Each question consists of three or four statements


followed by some conclusions. Consider the given statement to be true even
if they seem to be at variance with commonly known facts. Read all the
conclusions and then decide which of the given conclusions logically follow
from the given statements using all statements together. Mark your answer
as-

41. Statements: Some telephones are mobiles


All tablets are laptops
No mobiles are tablets
Some palmtops are tablets
Conclusions:
I. Some laptops are mobiles is a possibility
II. All telephones can be laptops
III. Some palmtops are laptops is a possibility
(a) Only I follow
(b) Only II and III follow
(c) Only I and II follow
(d) Only II follow
(e) None of these

42. Statements: Some bottels are mugs


All mugs are tups
No tups are balty
Conclusions:
I. No balty is mug is a possibility
II. Some baltys are tups
III. Some bottels are not balty
(a) Only II and III follow
(b) Only I follow
(c) None follow
(d) Only I and III follow
(e) Only III follow
23 Adda247 Publications For any detail, mail us at
Publications@adda247.com
Cracker Book for Bank (IBPS | SBI | RRB PO | Clerk) Mains Exams

43. Statements: No mangoes are banana


No bananas are apple
All apples are litchi
No litchis are sweet
Conclusions:
I. No apple are litchi
II. No bananas are sweet
III. All apples can be mango
(a) All follow
(b) Only I and II follow
(c) Only I follow
(d) Only III follow
(e) Only II and III follow

44. Statements: Some tables are chairs


All beds are sofas
No chairs are beds
All bed-sheets are Tables
Conclusions:
I. All bed-sheets can be sofas
II. Some chairs are bad-sheets
III. All sofas can be chairs
IV. Some bed-sheets are tables
(a) All follow
(b) Only IV follows
(c) Only I and IV follow
(d) Only II and III follow
(e) None follow

45. Statements: All books are pens


Some pens are pencils
No pencils are dots
Some dots are pens
Conclusions: I. Some pens are not pencils
II. Some books are dots

24 Adda247 Publications For any detail, mail us at


Publications@adda247.com
Cracker Book for Bank (IBPS | SBI | RRB PO | Clerk) Mains Exams

III. Some pens are not dots


IV. No dots are books
(a) Only III follow
(b) Only I and III follow
(c) Only II follow
(d) Only II and IV follow
(e) None of these

Directions (46-50): In each of the questions below are given three


statements followed by three Conclusions numbered I, II and III. You have
to take the given statements to be true even if they seem to be at variance
from commonly known facts. Read all the Conclusions and then decide
which of the given Conclusions logically follows from the given statements
disregarding commonly known facts.
46. Statements: All flowers are toys.
Some toys are trees.
Some angels are trees.
Conclusions:
I. Some angels are toys.
II. Some trees are flowers.
III. Some flowers are angels.
(a) Only I follows
(b) Only II follows
(c) Only I and III follow
(d) Only III follows
(e) None follows
47. Statements: Some pigeons are dogs.
All dogs are cats.
All pigeons are horses.
Conclusions: I. Some horses are dogs.
II. Some cats are pigeons.
III. Some horses are cats.
(a) All follows
(b) Only II and III follow
(c) Only I and III follow
(d) Only I and II follow
(e) None of these
25 Adda247 Publications For any detail, mail us at
Publications@adda247.com
Cracker Book for Bank (IBPS | SBI | RRB PO | Clerk) Mains Exams

48. Statements: All tables are streets.


All streets are bottles.
All parrots are bottles.
Conclusions: I. Some streets are parrots.
II. Some bottles are tables.
III. All tables are bottles.
(a) Only I follows
(b) Only II follows
(c) Only III follows
(d) Only II and III follow
(e) None of these

49. Statements: Some cartoons are boys.


All boys are windows.
All jokers are windows.
Conclusions: I. All jokers are boys.
II. Some cartoons are windows.
III. Some jokers are cartoons.
(a) Only I follows
(b) Only II follows
(c) Only III follows
(d) Only I and II follow
(e) None of these

50. Statements: Some saints are balls.


All balls are bats.
Some tigers are balls.
Colnclusions: I. Some bats are tigers.
II. Some saints are bats.
III. All bats are balls.
(a) Only I and III follow
(b) Only II follows
(c) Only I and II follow
(d) Only III follows
(e) None of these

26 Adda247 Publications For any detail, mail us at


Publications@adda247.com
Cracker Book for Bank (IBPS | SBI | RRB PO | Clerk) Mains Exams

Solutions

Direction (1-3):
1. (e);

2. (c);

3. (e);

Direction (4-5):
4. (e);

27 Adda247 Publications For any detail, mail us at


Publications@adda247.com
Cracker Book for Bank (IBPS | SBI | RRB PO | Clerk) Mains Exams

5. (d);

Direction (6-8)
6. (d);

Among all the conclusion, conclusion (d) does not follows as it is


clear from the venn diagram some part of egg is cat and no cat is
ball. Therefore, all egg cannot be ball.

7. (d);

Among all the conclusion, conclusion (d) does not follows as


there is no direct relation between the elements pen and chair.
Therefore, we cannot conclude that no pen is chair.

28 Adda247 Publications For any detail, mail us at


Publications@adda247.com
Cracker Book for Bank (IBPS | SBI | RRB PO | Clerk) Mains Exams

8. (e);

Among all the conclusions, conclusion (e) does not follows as


some rubber is definitely plastic but nothing can be said about
all rubber being plastic.

Direction (9-10):
9. (d);

10. (e);

Directions (11-13):
11. (e);

12. (c);

29 Adda247 Publications For any detail, mail us at


Publications@adda247.com
Cracker Book for Bank (IBPS | SBI | RRB PO | Clerk) Mains Exams

13. (e);

Directions (14-15):
14. (b);

15. (e);

Directions (16-18):
16. (a);

Conclusion (a) does not follows as there is no direct relation


between the elements mall and complex. Hence, we cannot
conclude that some mall are not complex.

17. (b);

30 Adda247 Publications For any detail, mail us at


Publications@adda247.com
Cracker Book for Bank (IBPS | SBI | RRB PO | Clerk) Mains Exams

Conclusion (b) does not follows as from the venn diagram it is


clear some light is crossing but nothing can be said about some
crossing not being light. Hence, some crossing is not light does
not follow.

18. (a);

Conclusion (a) does not follows as from the venn diagram it is


clear that definitely some paper is pencil. Hence, possibility case
will not hold true.
Directions (19-20):
19. (d);

20. (e);

Direction (21-22):
21. (d);

31 Adda247 Publications For any detail, mail us at


Publications@adda247.com
Cracker Book for Bank (IBPS | SBI | RRB PO | Clerk) Mains Exams

22. (b);

23. (e);

24. (d);

25. (e);

26. (c)

27. (e)

28. (a);

32 Adda247 Publications For any detail, mail us at


Publications@adda247.com
Cracker Book for Bank (IBPS | SBI | RRB PO | Clerk) Mains Exams

Directions (29-30):
29. (d);

30. (c);

Directions (31-35):
31. (c);

32. (d);

33. (b);

33 Adda247 Publications For any detail, mail us at


Publications@adda247.com
Cracker Book for Bank (IBPS | SBI | RRB PO | Clerk) Mains Exams

34. (a);

35. (e);

36. (a);

37. (e);

38. (c);

34 Adda247 Publications For any detail, mail us at


Publications@adda247.com
Cracker Book for Bank (IBPS | SBI | RRB PO | Clerk) Mains Exams

Directions (39-40):
39. (d);

40. (d);

Directions (41-45):
41. (c);

42. (e);

43. (d);

35 Adda247 Publications For any detail, mail us at


Publications@adda247.com
Cracker Book for Bank (IBPS | SBI | RRB PO | Clerk) Mains Exams

44. (c);

45. (e);

46. (e);

47. (a);

48. (d);

36 Adda247 Publications For any detail, mail us at


Publications@adda247.com
Cracker Book for Bank (IBPS | SBI | RRB PO | Clerk) Mains Exams

49. (b);

50. (c);

 Key Points to Remember for New Pattern Questions


➢ Syllogism topic is a quite scoring one and also quite easy
one to deal with but one should only have to be clear with
the concepts.
➢ The most common points seen in the exam are ‘can’, ‘can
never’ and ‘possibility’. As all these are the recently seen
key words in exam.
➢ Reverse Syllogism is also a mostly seen topic. In this the
diagram of all the statements should be drawn to find out
the one which follows the given conclusion.
➢ An another type comprises of syllogism in which one of
the conclusion does not follow the given statement and
you have to find that by drawing the diagram first.
➢ The probable changes that can be seen in the upcoming
examination is the coded form of Syllogism. You should
draw the diagram by following the codes.

37 Adda247 Publications For any detail, mail us at


Publications@adda247.com
1 Adda247 Publications For any detail, mail us at
Publications@adda247.com
Chapter

7 Inequalities

BEST APPROACH TO SOLVE THE QUESTIONS

It is a quiet easy topic as compared to the others and also can help you to
score well in exams. One should only has to follow the given statement and
just go with the basic knowledge of this topic and you will find your answer.
If you find the questions of inequality in the exams try not to skip this as it
will help you to attain good marks.

Example-1: In the following question each of the following questions,


assuming the given statements to be true, find which of the two conclusions
I and II given below them is/are definitely true. Give answer.
(RBI Grade B-2016)
(a) If only conclusion I is true.
(b) If only conclusion II is true.
(c) If either conclusion I or II is true.
(d) If neither conclusion I nor II is true.
(e) If both conclusions I and II are true.

1. Statements: P=R, Y≥T, U≥V


Conclusions: I. V>U II. Y=R

2. Statements: P≤D, D>Z, D≤X


Conclusions: I. D≥P II. P≤Z

3. Statements: L≤M, N=O, N≥A


Conclusions: I. N≥M II. O≥M

4. Statements: P≥H, Z≤J, H≥J


Conclusions: I. J≤P II. Z≤H

5. Statements: A>B, P≥L, L=S


Conclusions: I. B≥L II. L<B
2 Adda247 Publications For any detail, mail us at
Publications@adda247.com
Solutions:
1. (d); V > U does not follow Y = R does not follow
2. (a); D ≥ P follows P ≤ Z does not follows
3. (d); N ≥ M does not follow O ≥ M does not follow
4. (e); J ≤ P follows Z ≤ H follows
5. (d); L ≤ B does not follow B > L does not follow

Example-2: In these questions, a relationship between different elements is


shown in the statements. The statements are followed by two conclusions.
Give answer.
(Canara PO-2017)

1. Statement: L≤T≤I≥M<X, W<P≤L≥B≥K


I. K<X II. W>M
(a) if only conclusion II is true.
(b) if only conclusion I is true.
(c) if neither conclusion I nor II is true.
(d) if either conclusion I or II is true.
(e) if both conclusions I and II are true.
2. Statement: Z<U≤D≤A≤M<S, Q>A≤Y<G
I. Z<Y II. S>Q
(a) if both conclusion I and II are true.
(b) if only conclusion I is true.
(c) if neither conclusion I nor II is true.
(d) if either conclusion I or II is true.
(e) if only conclusion II is true.

3. Statement: L≤T≤I≥M<X, W<P≤L≥B≥K


I. K≥M II. P>M
(a) if only conclusion II is true.
(b) if either conclusion I or II is true.
(c) if neither conclusion I nor II is true.
(d) if only conclusion I is true.
(e) if both conclusions I and II are true.

3 Adda247 Publications For any detail, mail us at


Publications@adda247.com
4. Statement: Z<U≤D≤A≤M<S, Q>A≤Y<G
I. M≥U II. G>Z
(a) if only conclusion II is true.
(b) if only conclusion I is true.
(c) if neither conclusion I nor II is true.
(d) if either conclusion I or II is true.
(e) if both conclusions I and II are true.

5. Statement: J>K≥H=U≥B≤T<F≤R
I. J>B II. H<R
(a) if only conclusion II is true.
(b) if either conclusion I or II is true.
(c) if neither conclusion I nor II is true.
(d) if only conclusion I is true.
(e) if both conclusions I and II are true.

1. (c); Statement-L≤T≤I≥M<X, W<P≤L≥B≥K


I. K<X (false) II. W>M (false)

2. (b); Statement: Z<U≤D≤A≤M<S, Q>A≤Y<G


I. Z<Y(True) II. S>Q(false)

3. (c); Statement: L≤T≤I≥M<X, W<P≤L≥B≥K


I. K≥M(false) II. P>M(false)

4. (e); Statement: Z<U≤D≤A≤M<S, Q>A≤Y<G


I. M≥U(True) II. G>Z(True)

5. (d); Statement: J>K≥H=U≥B≤T<F≤R


I. J>B(True) II. H<R(false)

4 Adda247 Publications For any detail, mail us at


Publications@adda247.com
Practice Exercise Based on new Pattern

Directions (1-5): Study the following information carefully to answer the


questions given below.
P+Q means ‘P’ is greater than ‘Q’.
P-Q means ‘P’ is smaller than ‘Q’.
P/Q means ‘P’ is either greater than or equal ‘Q’.
P&Q means ‘P’ is either smaller than or equal to ‘Q’.
P*Q means ‘P’ is equal to ‘Q’

1. Statements: A-B, B&C, C*D, D+E


Conclusions: (I) A+E (II) B*D
(a) If only conclusion I is true
(b) If only conclusion II is true
(c) If either conclusion I or II is true
(d) If neither conclusion I nor II is true
(e) If both conclusions I and II are true

2. Statements: A/B, B&C, C*D, D+E


Conclusions: (I) A + C (II) C + E
(a) If only conclusion I is true
(b) If only conclusion II is true
(c) If either conclusion I or II is true
(d) If neither conclusion I nor II is true
(e) If both conclusions I and II are true

3. Statements: A+B, B&C, C+P, D/A


Conclusions: (I) D+B (II) A+P
(a) If only conclusion I is true
(b) If only conclusion II is true
(c) If either conclusion I or II is true
(d) If neither conclusion I nor II is true
(e) If both conclusions 1 and 2 are true
5 Adda247 Publications For any detail, mail us at
Publications@adda247.com
4. Statements: A&B, C+D, D*B, A&E
Conclusions: (I) D/A (II) C+B
(a) If only conclusion I is true
(b) If only conclusion II is true
(c) If either conclusion I or II is true
(d) If neither conclusion I nor II is true
(e) If both conclusions I and II are true

5. Statements: B+A, C*B, D*C, D+E


Conclusions: (I) A+C (II) B+E
(a) If only conclusion I is true
(b) If only conclusion II is true
(c) If either conclusion I or II is true
(d) If neither conclusion I nor II is true
(e) If both conclusions I and II are true

Directions (6-10): In each of these questions, relationship between some


elements is shown in the statement (s). These statements are followed by
two conclusions. Read the statements and give answer:
(a) If only conclusion I follows.
(b) If only conclusion II follows.
(c) If either conclusion I or II follows.
(d) If neither conclusion I nor II follows.
(e) If both conclusions I and II follow.

6. Statement: K < E = L, T ≥ G < 𝐿 ≥ D


Conclusions: I. E > T II. T ≥ L

7. Statement: P ≥ I ≤ O < Z < R > A


Conclusions: I. P ≤ R II. I > A

8. Statement: U < Q, V = E, R ≥ V, N < R, N > Q


Conclusions: I. Q > R II. R > E

9. Statement: S ≤ E, N ≤ Z, E ≥ N, N > Q
Conclusions: I. S > Z II. Q < E

10. Statement: N > A ≥ M = B < E ≥ R ≤ D


Conclusions: I. N > B II. M ≥ D
6 Adda247 Publications For any detail, mail us at
Publications@adda247.com
Directions (11 –15): In the following questions, the symbols @, $, , #,
and % are used with the following meaning as illustrated below.
‘P $ Q’ means ‘P is not smaller than Q’.
‘P @ Q’ means ‘P is neither smaller than nor equal to Q’.
‘P # Q’ means ‘P is neither greater than nor equal to Q’.
‘P %Q’ means ‘P is neither greater than nor smaller than Q’.
‘P  Q’ means ‘P is not greater than Q’.
Now, in each of the following questions assuming the given statements to
be true, find which of the four Conclusions I, II, III and IV given below them
is/are definitely true and give your answer accordingly.

11. Statements: N % B, B $ W, W # H, H  M
Conclusions: I. M@W II. H@N
III. W%N IV. W # N
(a) Only I is true
(b) Only III is true
(c) Only IV is true
(d) Either III or IV is true
(e) Either III or IV and I are true

12. Statements : RD, D $ J, J # M, M @ K


Conclusions: I. K # J II. D @ M
III. R # M IV. D @ K
(a) None is true
(b) Only I is true
(c) Only II is true
(d) Only III is true
(e) Only IV is true

13. Statements: H @ T, T # F, F % E, E V
Conclusions: I. V $ F II. E @ T
III. H @ V IV. T # V
(a) I, II and III are true
(b) I, II and IV are true
(c) II, III and IV are true
(d) I, III and IV are true
(e) All are true
7 Adda247 Publications For any detail, mail us at
Publications@adda247.com
14. Statements: D # R, RK, K @ F, F $ J
Conclusions: I. J # R II. J # K
III. R # F IV. K @ D
(a) I, II and III are true
(b) II, III and IV are true
(c) I, III and IV are true
(d) All are true
(e) None of these

15. Statements: M $ K, K @ N, NR, R # W


Conclusions: I. W @ K II. M $ R
III.K @ W IV. M @ N
(a) I and II are true
(b) I, II and III are true
(c) III and IV are true
(d) II, III and IV are true
(e) None of these

Directions (16-20): In the following questions, the symbols $, %, @, © and


 are used with the following meaning as illustrated below.
‘P%Q’ means ‘P is neither greater than nor smaller than Q’.
‘P$Q’ means ‘P is neither smaller than nor equal to Q’.
‘P©Q’ means ‘P is neither greater than nor equal to Q’.
‘PQ’ means ‘P is not greater than Q’.
‘P@Q’ means ‘P is not smaller than Q’.
Now in each of the following questions assuming the given statements to be
true, find which of the three conclusions I, II and III given below them is/are
true and give your answer accordingly.

16. Statements: V©K, K@B, B$M


Conclusions:
I. V©B II. M©K III. M©V
(a) None is true (b) Only I is true
(c) Only II is true (d) Only III is true
(e) Only II and III are true
8 Adda247 Publications For any detail, mail us at
Publications@adda247.com
17. Statements: N@D, DK, K$A
Conclusions:
I. K@N II. A©D III. N$A
(a) None is true (b) Only I is true
(c) I and II are true (d) I and III are true
(e) All are true
18. Statements: K@T, T$N, N©R
Conclusions:
I. R$K II. NK III. K$N
(a) None of true (b) Only I is true
(c) Only II is true (d) Only III is true
(e) All are true.

19. Statements: DR, R%F, F$T


Conclusions:
I. F%D II. F$D III. T©R
(a) Only I is true
(b) Only II is true
(c) Only III is true
(d) Only either I or II is true
(e) Only either I or II and III are true

20. Statements: W%K, K©F, D$F


Conclusions:
I. D$K II. D$W III. F@W
(a) Only I and III are true
(b) Only I and II are true
(c) Only II and III are true
(d) All, I, II and III are true
(e) None of above

Directions (21-25): In these questions, the relationship between different


elements is shown in the statements. The statements are followed by two
conclusions. Give answer
9 Adda247 Publications For any detail, mail us at
Publications@adda247.com
(a) if only conclusion I is true.
(b) if only conclusion II is true.
(c) if either conclusion I or II is true.
(d) if neither conclusion I nor II is true.
(e) if both conclusions I and II are true.

21. Statements: A ≥ B > C, D ≤ A < P


Conclusions: I. P > C II. D < P

22. Statements: A ≤ B < C, C ≥ M > N


Conclusions: I. M > C II. B < N

23. Statements: M = N ≥ O > P < R


Conclusions: I. M < R II. R ≥ M

24. Statements: A ≥ B > C, D ≤ A < P


Conclusions: I. B > P II. P > B

25. Statements: F ≥ G = H ≥ M ≥ C
Conclusions: I. F ≥ M II. G = C

Directions (26–30): In these questions relationship between different


elements is shown in the statements. These statements are followed by
conclusions. You have to find out which of the conclusions logically follows
from the given statements.
26. Statements: X ≤ Y, Y ≥W, W < P , P≤M
Conclusions: I. M > W II. P > X III. X ≤ M
(a) Only I follows
(b) Only II follows
(c) Only III follows
(d) Only I and III follow
(e) None of these

27. Statements: P ≥ Q, N ≤ S, Q > N, S<T


Conclusions: I. T > Q II. P ≥S III.Q>T IV. P > N

10 Adda247 Publications For any detail, mail us at


Publications@adda247.com
(a) Only I and II follow
(b) Only I, II and III follow
(c) Only III and IV follow
(d) Only II, III and IV follow
(e) None of these

28. Statements: C < Q, Q ≤ J, J > E, E ≥ I


Conclusions: I. J < Q II. I < J III. Q <E IV. J > C
(a) Only I, II and III follow
(b) Only II, III and IV follow
(c) Only I, III and IV follow
(d) All I, II, and IV follow
(e) None of these

29. Statements: X ≥ Y= Z ≤ A < B > C


Conclusions: I. C < Z II. B < Y
(a) Only I follows
(b) Only II follows
(c) Only either I or II follows
(d) Neither I nor II follow
(e) Both I and II follow

30. Statements: P < Q ≤ R < S < T


Conclusions: I. R ≤P II. T > R
(a) Only I follows
(b) Only II follows
(c) Only either I or II follows
(d) Both follow
(e) None follows

Directions (31-35): In these questions, relationship between different


elements is shown in the statements. These statements are followed by two
conclusions:
(a) If only conclusion I follows.
(b) If only conclusion II follows.

11 Adda247 Publications For any detail, mail us at


Publications@adda247.com
(c) If either conclusion I or II follows.
(d) If neither conclusion I nor II follows.
(e) If both conclusion I and II follow.

31. Statements: K≥Q, U>V=I, K >J≤V, U<S


Conclusion: I. I<S II. I≤Q

32. Statements: D≥Q, P>J, O>Q>J, T>D


Conclusion: I. O>T II. T>J

33. Statements: Z<W=V, Y≥W, Z≥U, X≤V


Conclusion: I. Y≥U II. W≥X

34. Statements: A≥U, C=B<U, D≥A>O


Conclusion: I. B≤D II. O<U

35. Statements: Q=B, K>E≥B, J≥E, R≤Q


Conclusion: I. J>R II. R=J

Direction (36-40): Read the information /statement given in each question


carefully and answer the questions.

36. Which of the following expressions will be true if the expression ‘W ≤


V > T ≥ S’ is definitely true?
(a) W < S (b) T < W (c) V > S
(d) W ≤ T (e) None is true

37. Which of the following expressions will not be true if the expression
‘A ≥ B = C > D’ is definitely true?
(a) A D (b) CA (c) D<A
(d) B>D (e) All are true

38. In which of the following expression will the expression F  R’ be


definitely true?
(a) F < Q  G < R (b) R  Q < G  F (c) R  G = Q  F
(d) F = Q  G < R (e) None of these
12 Adda247 Publications For any detail, mail us at
Publications@adda247.com
39. In which of the following expressions does the expressions L < P’ does
not hold true?
(a) P > N  M = L (b) L = N  M < P (c) P < N  M  L
(d) L < M = N  P (e) L < M < N < P

40. Which of the following expressions will be true, if the expression ‘J  H


= I > K’ is definitely true?
(a) J  K (b) K < H (c) I  J
(d) K = J (e) None of These

Directions (41-45): In these questions, relationship between different


elements is shown in the statements. These statements are followed by two
conclusions.
Mark answer as
(a) If only conclusion I follows.
(b) If only conclusion II follows.
(c) If either conclusion I or II follows.
(d) If neither conclusion I nor II follows.
(e) If both conclusions I and II follow.

41. Statements: J>K≤S, Y≤T>Q, T≥S


Conclusions: I. K ≤ T II. Q < J

42. Statements: A>B<C, G≥Q, Q>A


Conclusions: I. G > B II. Q >B

43. Statements: I>R, S>Q≤T, R≤Q<K


Conclusions: I. R > S II. K > R

44. Statements: Y>Z≥W, U>Q>W>R


Conclusions: I. Y > R II. U > R

45. Statements: A>B<C, Q>A, G≥Q


Conclusions: I. C > G II. B ≥ Q

13 Adda247 Publications For any detail, mail us at


Publications@adda247.com
Directions (46-50): In each question, a relationship between different
elements is shown in the statements. The statements are followed by two
conclusions. Study the conclusion for the given statement and select the
appropriate answer.

46. Statements: P<L≤A=N≥E≥D; Q≥N<O


Conclusions: I. L≤E II. P<Q
(a) Only conclusion I is true.
(b) Only conclusion II is true.
(c) Neither conclusion I nor II is true.
(d) Both conclusion I and II are true.
(e) Either conclusion I or II is true.

47. Statements: P≤U=N≤C≥H>S; K≥C


Conclusions: I. P≤C II. U>H
(a) Only conclusion II is true.
(b) Only conclusion I is true.
(c) Both conclusion I and II are true.
(d) Neither conclusion I nor II is true.
(e) Either conclusion I or II is true.

48. Statements: P<L≤A=N≥E≥D; Q≥N<O


Conclusions: I. Q≥D II. A<D
(a) Both conclusion I and II are true.
(b) Either conclusion I or II is true.
(c) Only conclusion II is true.
(d) Only conclusion I is true.
(e) Neither conclusion I nor II is true.

49. Statement: D≥I>S≥M≤A<L


Conclusions: I. D≥A II. L>I
(a) Only conclusion I is true.
(b) Either conclusion I or II is true.
(c) Only conclusion II is true.
(d) Neither conclusion I nor II is true.
(e) Both conclusion I and II are true.

14 Adda247 Publications For any detail, mail us at


Publications@adda247.com
50. Statements: P≤U=N≤C≥H; K≥C
Conclusions: I. K>U II. U = K
(a) Both conclusion I and II are true.
(b) Either conclusion I or II is true.
(c) Only conclusion II is true.
(d) Only conclusion I is true.
(e) Neither conclusion I nor II is true.

Solutions

1. (d); (I) A+E (False) (II) B*D (False)

2. (b); (I) A + C (False) (II) C + E (True)


3. (a); (I) D+B (True) (II) A+P (False)
4. (e); (I) D/A (True) (II) C+B(True)
5. (b); (I) A+C (False) (II) B+E (True)

Directions (6-10):

6. (c); 7. (d); 8. (d);

9. (b); 10. (a);

Directions (11-15):
11. (e); 12. (a); 13. (b);

14. (e); 15. (e);

Directions (16-20):

16. (c); I. V < K ≥ B(FALSE)


II. M < B ≤ K(TRUE)
III. M < B ≤ K > V(FALSE)
15 Adda247 Publications For any detail, mail us at
Publications@adda247.com
17. (a); I. K ≥ D ≤ N(FALSE)
II. A < K ≥ D(FALSE)
III. N ≥ D ≤ K > A(FALSE)
18. (d); I. R > N < T ≤ K(FALSE)
II. N < T ≤ K(FALSE)
III. K ≥ T > N(TRUE)
19. (e); I. F = R ≥ D(FALSE)
II. F = R ≥ D(FALSE)
III. T < F = R(TRUE)

20. (b); I. D > F > K(TRUE)


II. D > F > K = W(TRUE)
III. F > K = W(FALSE)

Directions (21-25):

21. (e); I. P > C (True) II. D < P (True)

22. (d); I. M > C (False) II. B < N (False)

23. (d); I. M < R (False) II. R ≥ M (False)

24. (b); I. B > P (False) II. P > B (True)

25. (a); I. F ≥ M (True) II. G = C (False)

Directions (26 – 30):


26. (a); 27. (e); 28. (e);

29. (d); 30. (b);


Direction (31-35):

31. (a); I. I<S (True) II. I≤Q (False)

32. (b); I. O>T (False) II. T>J (True)


16 Adda247 Publications For any detail, mail us at
Publications@adda247.com
33. (b); I. Y≥U (False) II. W≥X (True)

34. (d); I. B≤D (False) II. O<U (False)

35. (c); I. J>R (False) II. R=J (False)

Directions (36 – 40):


36. (c); 37. (a); 38. (c);

39. (c); 40. (b);

Directions (41-45):

41. (a); I. K ≤ T (True) II. Q < J (False)

42. (e); I. G > B (True) II. Q >B (True)

43. (b); I. R > S (False) II. K > R (True)

44. (e); I. Y > R (True) II. U > R (True)

45. (d); I. C > G (False) II. B ≥ Q (False)

Directions (46 – 50):


46. (b); I. 𝐿 ≤ 𝐸 (False) II. 𝑃 < 𝑄 (True)

47. (b); I. 𝑃 ≤ 𝐶 (True) II. 𝑈 > 𝐻 (False)

48. (d); I. 𝑄 ≥ 𝐷 (True) II. 𝐴 < 𝐷 (False)

49. (d); I. 𝐷 ≥ 𝐴 (False) II. 𝐿 > 𝐼 (False)

50. (b); I. 𝐾 > 𝑈 (False) II. 𝑈 = 𝐾 (False)

17 Adda247 Publications For any detail, mail us at


Publications@adda247.com
 Key Points to Remember for New Pattern Questions
➢ Inequality consist of different mathematical symbols
which define the relation among different elements.
➢ In some cases there may be more than one statement so
to solve such question you have to combine these to get
the exact relation between two elements.
➢ An another variety in questions of Inequality is coded
form of it in which all the relations are given in coded
form. You first have to decode those operators to find the
relation between the element.

18 Adda247 Publications For any detail, mail us at


Publications@adda247.com
1 Adda247 Publications For any detail, mail us at
Publications@adda247.com
Chapter

8 Logical Reasoning

Practice Exercise Based on New Pattern

1. Statement: A sting operation by a news organisation called Cobrapost


claims to have revealed a deeply engrained bias towards the ruling
BJP within many of India's leading media groups, as well as a
willingness among some of the country's most senior media
executives and journalists to take money in return for pushing a
political agenda.
Which of the following can be inferred from the above statement?
(i) India’s leading media companies are more concerned with TRP
rather than showing the truth.
(ii) The ruling party is trying to manipulate the content provided by
the media.
(iii) Some of the journalists are corrupt.
(a) Both (i) and (ii) (b) Both (ii) and (iii) (c) Only (ii)
(d) Only (iii) (e) None of these

2. Statement: In county A, there have been numerous reports over the


past three months about people falling sick after consuming the beer
imported from country B. The beer manufacturers in county B are
claiming a conspiracy by the local manufacturers of country A.
Which of the following is a most appropriate course of action in light
of the above statement?
(a) Country A should stop all its imports from country B.
(b) Heavy taxes should be implemented by country A on the beer
that is imported from country B.
(c) People should be advised to cut down on their beer
consumption.
(d) The samples of beer imported from country A should be sent for
chemical analysis.
(e) A ban should be imposed on the sales of beer imported from
country B until the matter is investigated.
2 Adda247 Publications For any detail, mail us at
Publications@adda247.com
3. Statement: Some of the recent researches points to the addictive
nature of grains. Neurologist and author of Grain Brain, Dr. David
Perlmutter, discussed the addictive effects grains have on our brain in
his book. While the book doesn’t directly say “carbs are addictive,” it
does say that when you eat carbs you will crave more carbs. And
absolutely, wheat for many people feels like something they could
never give up.
Which of the following most strengthens the above statement?
(a) Emotional eating doesn’t fix your emotional problems, in fact, it
usually makes them worse.
(b) Grains are not meant for human consumption, a close look at our
digestive autonomy suggest that we are “frugivores” or “fruit
eaters”.
(c) Many birds have a distinct organ, called the gizzard, which helps
grind grains and seeds. Grains are meant to be bird food for
parrots, pigeons, and doves.
(d) Our digestive tract cannot handle the mixture of starch and
sugar found in most recipes of cereals, breads, bagels, etc. And as
a consequence, most cooked grain we eat tends to ferment
inside of us producing gas and alcohol.
(e) Most people could easily go for months without eating certain
fruits or vegetables and it wouldn’t be a problem for them but
that’s not the case with processed grains.

Direction (4): Study the following information in which a statement is


followed by five assumptions, read carefully and answer the question
below-
The banking sector is facing tough times. But the sector is facing only short-
term pain as the rise in non-performing assets (NPAs) is proving a key
challenge for the industry at large. The regulator wanted some stringent
measures to be taken, and the government is coming out with so many
reforms and policies.
Which of the following can be assumed from the given statement?

3 Adda247 Publications For any detail, mail us at


Publications@adda247.com
(a) NPA was 2.4% more in last two years as compared to the present one.
(b) The steps taken by the government are not enough to revamp the
banking sector.
(c) Manufacturing sector is performing far better than the banking sector
for last five years.
(d) After the implementation of suitable expedient, the things are likely to
be changed for banking sector.
(e) The increase in NPA is the only factor responsible for the difficult time
of banking industry.

Direction (5): Study the following information in which a statement is


followed by five conclusions, read carefully and answer the question below-
Other than being an essential source of water for Indian agriculture, the
monsoon plays a critical role in flushing out pollutants over Asia. However,
recently increased pollution — particularly from coal burning — could
potentially weaken this ability
of the monsoon.

Which of the following can be deduced from the given statement?


(a) The unusually high concentration of particulate matter in the last few
days in India clearly shows that air pollution is emerging as a big
problem.
(b) As the climate gets warmer and frequency of rains reduces, such
spurts in coarse particles making breathing difficult will become a
new normal and the government is not waking up to the alarm.
(c) Both the periodicity and duration of dry spells in the country were
rising as total rainfall events in a year had fallen even though the
average rainfall in a year has not changed much, a direct consequence
of climate change.
(d) The annual average rainfall has remained the same because the
frequency of heavy downpours increased in the past two decades.
(e) The air quality in the region deteriorated because of dust storms in
western India, particularly Rajasthan.

4 Adda247 Publications For any detail, mail us at


Publications@adda247.com
6. Statement-The "kidnapping" case of a retired Pakistan army officer in
Nepal could not be taken to the ICJ or the UN on the basis of
"assumptions" as there is no "concrete evidence" suggesting India's
hand in it, a minister has told Parliament.
Which of the following can be inferred from the given statement?
(I) The issue appeared to have a link with the conviction of
Kulbushan Jadhav.
(II) It can be said that Indian intelligence agency RAW is behind the
kidnapping.
(III) The FO (foreign officers) is trying to obtain solid evidence and
the Nepalese government had been asked to get in touch with
three Indians who had received the Pakistan army officer at the
airport.
(a) Only (I) (b) Only (III) (c) Only (II)
(d) Both (II) and (III) (e) None of these

7. Statement-Despite stagnant foreign direct investment (FDI) inflow of


$44 billion in 2016, India will most likely remain most favoured
destination due to its attractiveness among MNCs for cross-border
mergers and acquisitions, a UN trade report has said.

Which of the following negates the following statement?


(I) An improved economic outlook in major economies, such as
ASEAN, China and India, will likely boost investor confidence,
propping up the region's FDI prospects for 2017.
(II) There are tax related concerns in India that may pose as
deterrent to some foreign investors.
(III) Cross-border mergers and acquisitions deals have become
increasingly important for foreign multinational enterprises to
enter the rapidly-growing Indian market.
(a) Only (I) (b) Only (III) (c) Only (II)
(d) Both (II) and (III) (e) None of these

5 Adda247 Publications For any detail, mail us at


Publications@adda247.com
Directions (8-9): In each of the following questions, three statements
numbered I, II and III are given. There may be cause and effect relationship
among the statements. These statements may be the effect of the same
cause or independent causes. These statements may be independent causes
without having any relationship. Read all the statements in each question
and mark your answer as-
(a) Statement I will be cause and Statement II and III will be its effects.
(b) Statement II and III will be causes and I will be its effect.
(c) Statement III is cause and Statement I is its effect and Statement II is
effect of independent cause.
(d) Statement II is cause and Statement I is its effect and Statement III is
effect of independent cause.
(e) Statement II is cause and Statement III is its effect and Statement I is
an independent cause.

8. Statement I- China has approved a plan to build an underwater


observation network across the disputed East and South China seas,
in yet another provocative move that will anger Beijing's neighbors.
Statement II-China is likely to establish additional military bases in
Pakistan and other countries with which it has longstanding friendly
ties and similar strategic interests.
Statement III-China's military build-up initiative, along with regular
naval vessel visits to foreign ports, both reflects and amplifies China's
growing influence, extending the reach of its armed forces.
Which of the following may be cause/effect among the above
statements?

9. Statement I-International pop sensation Justin Bieber arrived in


Mumbai on Wednesday for his maiden concert in India.
Statement II-A rage among the youth, especially teenagers, Bieber is
touring to promote his critically acclaimed album ‘Purpose.’
Statement III-Grammy Award winning singer Justin Bieber fought
back tears during a short speech, after he performed acoustic versions
of Love Yourself and Cold water at the One Love Manchester concert.
Which of the following may be cause/effect among the above
statements?
6 Adda247 Publications For any detail, mail us at
Publications@adda247.com
10. Statement-The menacing spread, starting last Friday, of the malicious
software Wanna Cry, is a frightening reminder of the vulnerabilities of
a connected world.
Which of the following can be postulated from the given statement?
(I) It is more than obvious now that cyber vulnerabilities have
massive global implications.
(II) While the state of preparedness is a cause for worry, the likely
origin of Wanna Cry forces stakeholders to revisit a long-
standing issue and raises uneasy questions regarding the actions
of governments.
(III) The hackers have used cyber weapons and believed to have
linkage with terrorist organizations.
(a) Only (I) (b) Only (III) (c) Only (II)
(d) Both (I) and (II) (e) None of these

11. In the following question a statement is given followed by three


statements numbered as I, II and III. You have to read all three
statements and decide which of them strengthens or weakens the
statement.
Statement: Among the governments, there is a strong policy emphasis
on improving living conditions in urban areas as more and more
people are looking to migrate to semi-urban or rural areas. People are
willing to compromise on their standard of living in order to live in a
healthier/pollution free environment.
(I) Cities are economically vibrant spaces around the world and
drawing a large number of rural migrants looking for better
prospects. This is a sustained trend, particularly in developing
countries now a day.
(II) A fresh look at urban governance is necessary as migration from
rural areas picks up pace.
(III) The rate of migration has shown a consecutive decline in the last
three Censuses. It was possible that this slow-down signalled the
worsening conditions in urban slums, where most of the rural
immigrants were concentrated.

7 Adda247 Publications For any detail, mail us at


Publications@adda247.com
(a) Only statement (I) strengthen but statement (II) and (III) is the
neutral statement.
(b) Both statement (I) and statement (III) strengthen the given
statement but statement (II) is neutral statement.
(c) Both statement (I) and statement (II) are neutral statements and
statement (III) strengthen the statement.
(d) Both statement (I) and (II) weakens but statement (III)
strengthen the given statement.
(e) All statement (I), (III) and statement (II) weaken the given
statement.

12. The revenue of Company A has been decreasing from past one year
while the prices of the products offered by the company have also
decreased from past six months.
Which of the following cannot be the reason of downfall of the
revenue of Company A?
(a) Company B is offering same product at much lower prices than
Company A.
(b) The quality of the products of Company A is not upto a desirable
level.
(c) The demand of the product offered by Company A has been
decreasing due to the usage of an old technology in it.
(d) There is an abrupt rise in the availability of the product offered
by Company A but the demand has not changed accordingly.
(e) Chinese companies offered better and cheap products than
Indian Companies.

13. Today’s children are different from the children of previous


generations. They are more interested in playing video games or
sitting hours on the play stations than in playing outdoor games. They
are more technology oriented. These things are depriving the child
from developing their personality. They lack the stamina and strength
of the previous generations.
Which of the following statement weakens the given statement?

8 Adda247 Publications For any detail, mail us at


Publications@adda247.com
(I)
The children of today are mentally active as this has made
children much more organized and mature than in previous
times.
(II) These gadgets are addicting. They have made the children so
much addicted to PC and PS that they do not enjoy the outdoor
games anymore.
(III) The children of present time are not physically active. These
children are not being a team player anymore, they grow up to
become shy and introvert instead of being bold and extrovert.
(a) Only I (b) Both III and I (c) Only II
(d) Both II and III (e) All I, II and III
14. A survey conducted recently shows that religious people are more
financially successful than atheists. This proves that following
religious practices or having religious beliefs is not only important for
your spiritual health but also beneficial in maintaining a healthy bank
account.
Which of the following, if true, would most seriously weaken the
above conclusion?
(a) The survey does not tell us if those people became financially
successful after they started practicing religion or they started
practicing religion after they became successful.
(b) Some people embrace religion and spirituality mainly because
they believe that somehow being religious or spiritual can help
them achieve financial success.
(c) Most of the poor people are too busy thinking about earning
their daily livelihood.
(d) Percentage of atheists among the total population is highest in
most of the developed countries.
(e) Spiritual and religious people lead a more disciplined life.

15. A committee on reduction of air pollution in the cities of country X has


suggested a ban on the use and sales of all the diesel based electricity
generators in the cities.
Which of the following will be the repercussions, if the suggestion of
the committee is implemented?

9 Adda247 Publications For any detail, mail us at


Publications@adda247.com
(a) The prices of diesel will reduce sharply.
(b) Pollution levels will come down to safer level in all the cities.
(c) Residents living in the cities will face frequent power outages.
(d) There will be a reduction in the revenues of the companies
manufacturing diesel generators.
(e) All of the above

16. Statement: Around 1,100 cinemas across Tamil Nadu will be closed
from Monday as the theatre owners have called an indefinite strike
against a new municipal tax of 30 per cent in addition to the Goods
and Services Tax or GST.
Which of the following could be the repercussions of the above-
mentioned strike?
(i) People may not be able to watch movies.
(ii) Theatre owners in other parts of the country may follow the
suite.
(iii) The central government may be forced to reduce the tax rates.
(a) Only (i) (b) Only (ii)
(c) Only (ii) and (iii) (d) Only (i) and (iii)
(e) None of these

Directions (17-18): In each of the following questions, two statements


numbered I and II are given. There may be cause and effect relationship
between the two statements. These two statements may be the effect of the
same cause or independent causes. These statements may be independent
causes without having any relationship. Read both the statements in each
question and mark your answer as-
(a) If statements I is the cause and statement II is its effect.
(b) If statements II is the cause and statement I is its effect.
(c) If both the statements I and II are independent causes.
(d) If both the statements I and II are effects of independent causes.
(e) If both the statements I and II are effects of some common cause.

10 Adda247 Publications For any detail, mail us at


Publications@adda247.com
17. Statement I: The former UB group chief Mr. Mallya fled the country
last year and has unpaid debts close to the tune of Rs. 9000 crore
taken as loans from various Indian banks.
Statement II: Fugitive businessman Vijay Mallya was booed by a
section of India supporters as he entered the Oval arena to watch
India’s crucial ICC Champions Trophy encounter against South Africa.

18. Statement I: The National Council of Educational Research and


Training (NCERT) will review all its books.
Statement II: NCERT books were created over a decade ago and need
to be updated, the books were created in 2007.

19. Statement: Highlighting his achievements in present in rooting out


corruption, Prime Minister Narendra Modi said that there has been
"not even one taint or blot" on his government in these past three
years.
Which of the following abrogates the above statement?
(I) Technology has brought about transparency and is being used to
achieve great heights across sectors in India in Modi governance.
(II) In several parameters, the country is moving at a rapid pace
under PM Modi’s government. Noting that infrastructure is
essential for sustainable development. The increase in
aspiration of the common people is the greatest cause of this fast
track development of the country.
(III) The billion Indians in India are still below poverty line and they
are also finding the way to get rid of biggest problem of India
which is corruption.
(IV) India is now receiving record number of foreign direct
investment and all credit agencies and multilateral fora are
giving a positive rating about India.
(a) Only II (b) Only III and IV (c) Only III
(d) Only IV (e) Only I and III

11 Adda247 Publications For any detail, mail us at


Publications@adda247.com
20. Statement: “In a big relief to liquor companies which are holding
their stocks worth more than Rs. 200 crore in Bihar, the Supreme
Court on Monday granted them two months more time to transport
liquor out of the state and directed them to dispose off their stocks by
July 31.”
I. Beverages Companies, contended that it was possible even to
destroy the existing stocks by the end of this month.
II. The apex court had earlier fixed a deadline of May 31 for the
companies to dispose off their stocks.
III. Seeking more time for transporting the stocks out of state, the
confederation said in its petition that delay in disposing off
stocks was beyond its control as clearances were required from
various agencies.
Which of the following can be the Presupposition of the given
statement?
(a) Only I (b) Only II and III (c) Only I and III
(d) All of the above (e) None of the above

21. In the following question a statement is given followed by some more


statements. You have to read all the statement and decide which of
them weakens the statement.
The NITI Aayog released the results of a study warning that India is
facing its “worst” water crisis in history and that the demand for
potable water will outstrip supply by 2030, if steps are not taken.
Which of the following most weakens the above statement?
(a) Currently 600 million Indians face high to extreme water
shortage and about two lakh people die every year due to
inadequate access to potable water.
(b) People may face water scarcity and the country will suffer a loss
of up to 6 per cent of the GDP.
(c) The country's nearly 70 per cent of water is contaminated,
which puts India at the 120th position among 122 countries in
the water quality index.
12 Adda247 Publications For any detail, mail us at
Publications@adda247.com
(d) The ranks of all states in the index on the composite water
management, comprising 9 broad sectors with 28 different
indicators covering various aspects of ground water, restoration
of water bodies, irrigation, farm practices, drinking water, policy
and governance.
(e) The condition of availability of water currently in India is better
than the water crisis which India had faced in 1960.

Direction (22): Study the following information in which a statement is


followed by five assumptions, read carefully and answer the question
below-
Statements: All the population of village A is relocating to city B, near to
village A.
Assumption: (I) City B is rich in terms of resources and employment
opportunities.
(II) A critical virus has been spread in village A and more than 10 people
died from it.
Which of the following can be assumed from the given statement?
(a) Both I and II follow
(b) Neither II nor I follow
(c) Only II follow
(d) Only I follow
(e) Either I or II follow

Direction (23): Study the following information in which a statement is


followed by five assumptions, read carefully and answer the question
below-
“If we slightly increase the price of our product then we can improve the
quality of our product to increase the number of customers from our only
rival company V,”-A statement by owner of company U.
Which of the following can be assumed from the given statement?
(a) Without increasing the price, the quality of the product offered by
company U can’t be increase.
(b) Quality offered by company V of the product is better than the quality
offered by company U.

13 Adda247 Publications For any detail, mail us at


Publications@adda247.com
(c) Improvising the quality is the only way to attract more number of
customers.
(d) The price offered by the rival company V is already higher than the
price offered by company U.
(e) Superior quality even for bit higher price will definitely increase the
demand of the product in the market.

Direction (24): Study the following information in which a statement is


followed by some statement, read carefully and answer the question below-
A company has X decided to increase the HRA allowances of its employees
due to reallocation of the company to NCR region.
Which of the following can be inferred from the given statement?
(I) The HRA allowances directly depend on the region of living of the
employee.
(II) The employees of the company X are on strike due to the reallocation
of the company.
(III) The profit amount of company X has increased so it has also increased
the amount of HRA of its employees.
(a) Both (I) and (II) (b) Both (II) and (III) (c) Only (I)
(d) Only (III) (e) None of these

25. In the following question a statement is given followed by some more


statements. You have to read all the statement and decide which of
them strengthen the statement.
Out of all the major quadrennial international sporting events, the
Football World Cup is the one that gives Indians the purest pleasure.
The Cricket World Cup douses Indians in a broth of tension. The
Olympics continue to visit humiliation on the idea of India as an
athletic nation. The FIFA jamboree, though, is devoid of any such
freight.
Which of the following most strengthen the above statement?
(a) Cricket has always been the first love of Indians. Indians breath
Cricket and enjoy every bit of it.
(b) Cricket is a religion here and it makes a lot of us happy. People
need a Sachin in football. Indians want to talk about the
popularity of football players in the national side as well.
14 Adda247 Publications For any detail, mail us at
Publications@adda247.com
(c) With the people expressing their support, this surely earmarks a
turning point for the rise of football in India and exploit the
potential of our youth and give impetus to the rising status of
this sport.
(d) So far, Hockey is the superior sport. Its players took pride in
'their' way of playing - of their touch and superb body language.
(e) The performance of Indian Women who reach the Olympics are
thus likely to already have exceptional skills, relative to men in
India. Therefore, India should field more make an effort to field
more women and give them better access to trainers and
facilities.

26. Google is trying to turn its search engine into an employment engine.
Job hunters will be able to go to Google and see help-wanted listings
that its search engine collects across the internet. The results will aim
to streamline such listings by eliminating duplicate jobs posted on
different sites. Google will also show employer ratings from current
and former workers, as well as typical commute times to job locations.
(I) Now a days Google market is not good that’s why Google
launched new service.
(II) Google is no more interested in providing other services to its
consumer, now Google is focused only on turning its search
engine into an employment engine.
(III) Google’s search engine results will aim to well-organized
employment listings by removing duplicate jobs.
(IV) Google is not interested to show any kind of ratings in its new
employment search engine.
Which of the following can be deduced from the given passage?
(a) Only II and IV (b) Only I and III (c) Only II and III
(d) Only III (e) None of these

Directions (27-28): Travis Kalanick spent most of the past decade turning
a taxi app (Uber) into the world's most valuable startup. Uber is ride-hailing
service that he helped found in 2009 and built into a transportation
colossus.

15 Adda247 Publications For any detail, mail us at


Publications@adda247.com
Travis Kalanick stepped down Tuesday as chief executive of Uber, after a
seemingly endless series of scandals raised doubts over his leadership.
Kalanick’s resignation doesn’t come as a surprise -- after he was caught on
video chewing out an Uber driver who questioned the company’s
compensation policies in February. Meanwhile, Shareholders released a
damning report on the firm's management culture.
(I) Uber is a riding service that Chief Executive of Uber helped built into
transportation prodigious.
(II) Shareholders ordered Travis Kalanick to relinquish position of chief
executive of Uber.
(III Holder released a report against firm’s management culture that was
condemnatory.

27. Which of the following can be hypothesized from the above passage?
(a) Only II (b) Only I and III (c) Only I and II
(d) All of these (e) None of these

28. Which of the followings is not in the line with the passage?
(I) Kalanick’s resignation came under the pressure of damning
report which is released by the Shareholders on management’s
culture, this one of the reason of resignation of Chief executive of
Uber.
(II) Travis Kalanick stepped down as chief executive of Uber,
because endless series of scandals raised doubts over his
leadership as well as he was caught on video chewing out an
Uber driver and shareholder’s report was not in a favour of
management.
(III) Travis Kalanick spent most of the past decade turning a taxi app
(Uber) into the world's most valuable startup.
(a) Only II (b) Only I and III (c) Only I and II
(d) All of these (e) None of these
16 Adda247 Publications For any detail, mail us at
Publications@adda247.com
29. Statement: An elite Delhi Golf club has apologised after its staff told a
woman who turned up dressed in a traditional north-eastern Indian
outfit to leave. A report said- The staff asked her to leave because they
said she looked "like a maid".
Which of the following is an appropriate course of action in light of the
above incident?
(i) The club should seek an explanation from the staff member for
such a behavior and take disciplinary action against that person.
(ii) Club should apologize for its actions pay fine of 1000 rs to the
women.
(iii) Club should set up a system to verify the actual profiles of new
entrants, so that the staff members won’t have to judge the
entrants on the basis of their dresses.
(a) Only (iii) (b) Only (ii) (c) Only (i)
(d) Both (ii) and (iii) (e) All of these

30. Statement: Low-risk category Indian travellers to the US from now on


would experience speedy entry into the country after landing, with
India making a formal entry into an American initiative. US President
Donald Trump welcomed India's entry into the International
Expedited Traveler Initiative (Global Entry Program).
Which of the following could be the repercussions of India’s entry into
IETI?
(i) It would facilitate closer business and educational ties between
the citizens of India and the US.
(ii) India too will launch a similar programme for tourists entering
the country,
(iii) Travelling to United States will become less risky for Indians.
(a) Only (iii) (b) Only (ii) (c) Only (i)
(d) Both (ii) and (iii) (e) All of these

Direction (31): Study the following information in which a statement is


followed by some course of action read carefully and answer the question
below-

17 Adda247 Publications For any detail, mail us at


Publications@adda247.com
The Uttarakhand high court has banned paragliding and white-water
rafting and other water sports. This move aimed at safeguarding both the
environment and people engaging in these activities. The court’s decision
comes as a jolt to adventure tourism and water sports in the Himalayan
state.

Course of Action
I. An appeal should be filed against this decision as It’s an industry that
provides indirect jobs to no less than 10,000 families.
II. The tourism must be promoted but it is required to be regulated and
for this the State government should be directed to prepare the
transparent policy.
III. All the villages and cities near the banks of the rivers should be ceased
to reduce the pollution in the rivers.
Which of the following is an immediate course of action for the given
statement?
(a) Only I and III
(b) Only II and III
(c) All of the above
(d) Only II
(e) None of these

Direction (32): Study the following information in which a statement is


followed by some assumptions, read carefully and answer the question
below-
National Crime Records Bureau (NCRB) Director Ish Kumar on Thursday
requested the government to provide limited access to the Aadhaar card
database, which would help the investigating agencies in conducting a
probe into inter-State criminals more effectively with the help of
fingerprints.
Assumption: I. The Aadhar card database consist of basic information of a
citizen including his fingerprints.
II. If the number of explorers to a confidential database increases, its
authenticity will be subjected questionable.
III. NCRB do not have its own maintained record containing the realizable
information of criminals.
Which of the following can be assumed from the given statement?
(a) Only I and II (b) Only II and III (c) All of the above
(d) Only II (e) None of these
18 Adda247 Publications For any detail, mail us at
Publications@adda247.com
Direction (33): Study the following information which is followed by some
statement read carefully and answer the question below-
The number of hungry people in the world has risen for the first time in
more than a decade, according to a United Nations report released on
Wednesday. After a prolonged decline, world hunger appears to be on the
rise again. Conflict, drought and disasters linked to climate change are
among the key factors causing this reversal in progress,” said the report.
Which of the following statement strengthen the give statement?
(a) The increasing impact of extreme events related to a changing climate,
economic losses attributed to disasters were estimated at over $300
billion.
(b) More than nine out of 10 people living in urban areas around the
world are breathing polluted air, with southern Asia scoring the worst
in this area.
(a) Only I (b) Only II (c) All of the above
(d) Either I or II (e) None of these

Direction (34): Study the following information which is followed by some


statement read carefully and answer the question below-
Delhiites breathed the cleanest air this week, with monsoon winds bringing
the air quality in the satisfactory level for the first time in nearly a year,
authorities said.
Which of the following is the reason of the given statement?
(I) The monsoon winds cleaned the dusty air and brought the air quality
to satisfactory level.
(II) There was a dip in the air quality level due to dust storms in western
India that pushed the air quality level to "severe plus," bringing to
light that emergency level pollution could be a "summer-time
problem.
(III) Strong anti-cyclonic winds blowing from Rajasthan caused ‘hazy’
conditions in Delhi.
(a) Only I and II (b) Only II and III (c) All of the above
(d) Only I (e) None of these
19 Adda247 Publications For any detail, mail us at
Publications@adda247.com
Direction (35): In each of the following questions, two statements
numbered I and II are given. There may be cause and effect relationship
between the two statements. These two statements may be the effect of the
same cause or independent causes. These statements may be independent
causes without having any relationship. Read both the statements in each
question and mark your answer as-
(a) If statements I is the cause and statement II is its effect.
(b) If statements II is the cause and statement I is its effect.
(c) If both the statements I and II are independent causes.
(d) If both the statements I and II are effects of independent causes.
(e) If both the statements I and II are effects of some common cause.
(I) DMRC's 9,000-strong workforce of non-executive employees has been
taking part in sit-in protests at different metro stations.
(II) Demand of Delhi metro employees is a revision of salaries and pay-
grades and payment of arrears, apart from other demand like the
right to form a union.

Direction (36-38): In making decisions about important questions, it is


desirable to be able to distinguish between ‘strong’ arguments and ‘weak’
arguments. 'Strong’ arguments are those which are important and directly
related to the question. ‘Weak’ arguments are those which are of minor
importance and also may not be directly related to the question or may be
related to a trivial aspect of the question. Each question below is followed
by two arguments numbered I and II. You have to decide which of the
arguments is a strong argument and which is a weak argument.
Give answer
(a) if only Argument I is strong
(b) if only Argument II is strong
(c) if either Argument I or II is strong
(d) if neither Argument I nor II is strong
(e) if both Argument I and II are strong
20 Adda247 Publications For any detail, mail us at
Publications@adda247.com
36. Statement: Should banks be allowed to offer different interest rates
in India?
Arguments:
I. Yes, this is the only way to eventually lower interest rates in the
country.
II. No, this will discourage the customers and they might be
compelled to switch over from one bank to another.

37. Statement: Should the NCERT withdraw its directive for deletion of
passages from History books in the CBSE curriculum?
Arguments:
I. Yes, it will adversely affect school children.
II. No, History should not be taught to school children in a one-
sided manner and any distortions would have to be rectified.

38. Statement: Should Indian government allow the neighbouring


country X to dump huge quantities of Vanaspati ghee into India at a
price which is cheaper than market price?
Arguments:
I. No. it is a violation of the fundamental right of the farmer who
grow crops and that of the industry to operate with a level
playing held.
ll. Yes, it essential for India to provide shelter to all the industries
of X at all costs because it is a neighbouring country.

Directions (39-40): Below in each question are given two statements I and
II. These statements may be either independent causes or may be effects of
independent causes or a common cause. One of these statements may be
the effect of the other statement. Read both the statements and decide
which of the following answer choice correctly depicts the relationship
between these two statements.
Given answer
(a) if statements I is the effect and Statement II is the immediate cause
(b) if statements I is the immediate cause and statement II is its effect
(c) if Statement I is the effect but Statement II is not its immediate cause
(d) if Statement II is the effect but Statement I is not its immediate cause
(e) if Both the Statements I and II are independent cause
21 Adda247 Publications For any detail, mail us at
Publications@adda247.com
39. I. In the world’s biggest conference on AIDS in Barcelona, officials
in their declaration called for $ 10 billion per year to be donated
to the global AIDS fund and for at least two million people with
HIV in developing countries to receive anti-retroviral treatment
by 2012.
II. According to recent 2012 UN report, about 70 million people
will die of AIDS over the next two decades in the 45 worst
affected countries.
40. I. 500000 people were forced to flee to Rwanda from the Eastern
Congo town of Goma.
II. Fires burned out of control in the Eastern Congo town of Goma,
after a nearby volcano sent 100-foot-wide rivers of lava flowing
through the centre of the town and into Lake Kivu.

Solutions

1. (d); Statement (i) cannot be inferred as nothing relating to the TRP is


mentioned in the statement.
Statement (ii) can be inferred as the statement is only concerned
with the bias of media executives and journalists as they take
money in return for pushing a political agenda it means they are
trying to show news which they want.
The fact that some of the senior media executives and journalists
were willing to take money in return for pushing a political
agenda clearly points to their corrupt intentions. So statement
(iii) can be inferred.

2. (e); It is not appropriate to ban all the imports from country B which
might include some essential commodities so (a) not correct.
Option (b) is not appropriate as implementing heavy taxes might
reduce the consumption of beer imported from country B but
the consumption by even one person can be dangerous to him.
Similarly we don’t know the quantity of beer consumed by
people who fell sick after drinking the beer imported from
country B so (c) is not appropriate. Option (d) is irrelevant as
22 Adda247 Publications For any detail, mail us at
Publications@adda247.com
nothing has been mention in the statement about the imports
from country A. Option (e) will be most effective as a corrective
and preventive measure.

3. (e); Statement (b), (c) and (d) talks about the digestibility of grains
or their bad effects while the statement is only concerned with
the addictive nature of grains.
Statement (a) is irrelevant. Statement (e) points to a fact which
proves the addictive nature of grains.
4. (d); All the statement except (d), cannot be assumed from the given
statement as data provided in the statement is not enough to
assume the increase or decrease in NPA from the previous year
also it is not clear whether NPA is the only cause of pain to
banking sector. Further we also cannot assume from the given
statement that steps taken by the government are not enough or
some other sector is performing better than it. But it can be
clearly assumed from the given statement that after the proper
implementation of the policies and reforms there will be some
improvement in the condition of the banking sector.

5. (a); Only statement (a) can be deduced from the given statement as
by burning the coal the concentration of its particles is increased
which lead to excessive amount of air pollution. But for others
(b) and (e) can be assumed but cannot be deduced. While for (c)
and (d) the amount of average rainfall and its consistency in not
directly given in the statement.

6. (e); In the above question we have to find the statement which can
be indirectly deduced from the given statement. None of the
above statement can be inferred because this issue requires
solid evidences regarding Pakistan army officer’s kidnapping in
Nepal so, both (I) and (II) are false because we cannot say that if
this issue has a link with Kulbushan Jadhav’s case or this issue
has some involvement of RAW and in statement (III) it is given
that FO was trying obtain solid evidence which is more of an
assumption.

23 Adda247 Publications For any detail, mail us at


Publications@adda247.com
7. (c); In the above question we have to find the statement which will
make it ineffective or nullifies the given statement.
For I-False, because It supports the given statement as it shows
it will push investors towards FDI.
For II-True, because It opposes the given statement as it states
that tax related issues will intend to discourage the foreign
investors regarding investment in India which contradicts the
given statement.
For III- False, because It also supports the given statement as it
describes the importance of mergers and deals across the
border for foreign investors in growing Indian market.

Direction (8-9):
8. (e); In this question Statement II is the cause and Statement III will
be its effect because China is going to establish military bases in
Pakistan and other countries as its effect will be that the reach of
its armed forces is going to be enhanced. Statement I will be a
independent cause because it discusses about the plans of China
regarding its disputed East and South China seas.

9. (d); In the above question statement II will be the cause and


statement I will be its effect as Justin Bieber is touring to
promote his album will be the cause of his arrival to Mumbai for
his concert in India which is its effect. And Statement III will be
the effect of a different cause as it discusses about him being
emotional at a concert.

10. (d); In the above question we have to find which statement can be
assumed from the given statement.
For I- True, because cyber-attacks had a tremendous impact on
all the countries as given in statement that it infected thousands
of computer system.
For II-True, as it is clear from the above statement that it is a
frightening reminder all over the world which raises questions

24 Adda247 Publications For any detail, mail us at


Publications@adda247.com
regarding government’s action and it’s preparations to deal with
this issue.
For III- False, because it is not clear from the above statement
about its linkage with any terrorist organizations

11. (d); For I- Statement I weaken the given statement as according to it


large number of people migrating to urban areas for better job
opportunities whereas the given statement suggests that people
are willing to compromise on their standard of living in order to
live in a healthier/pollution free environment
For II- Statement II weaken the given statement as according to
it migration from the rural areas is increasing whereas
according to the statement, more and more people are willing to
migrate to rural/semi-urban areas.
For III- Statement III strengthens the statement is it states that
there has been a decline in number of migrants but it is due to
the poor condition of urban slum areas, mostly acquired by the
migrants and because of the policies of the government.

12. (e); As all options other than (e) can be the reason of the downfall of
the revenue of a company but option (e) cannot be a reason as
nothing regarding the Chinese/Indian companies have been
stated in the given statement and is not related with the
downfall of revenue of company A.

13. (a); Only I weakens the given statements as the statement suggest
that the today’s children are weaker than previous generation
and are not as much active and also lack in developing their
personality whereas the statement I states that today’s children
are mentally more active so it weakens the given statement.

14. (a);
(a) mention a flaw in the survey which weakens the
conclusion drawn from it.

25 Adda247 Publications For any detail, mail us at


Publications@adda247.com
(b) supports the statement by pointing out that some people
embrace religion to achieve financial success.
(c) poor people might not have enough time to engage in
religious practices but it does not eliminate the possibility
of them having religious beliefs.
(d) mentions a fact which is irrelevant as the given statement
talks about the general population of financially successful
people which includes both developed and developing
countries.
(e) is irrelevant to the statement.

15. (d);
(a) The prices of diesel are governed by various factors and
we cannot say if the reduction in the prices will be sharp or
minor.
(b) We don’t know the current levels of pollution in the cities
and how much will be the effect of the ban.
(c) There is no doubt the ban will increase the demand for
electricity but we don’t know if the government or the
private agencies involved in the production will or will not
be able to meet the increased demands.
(d) As there will be no further sale of diesel generators in the
cities, there will definitely be a reduction in the revenues.

16. (e); In this question, we have to find out the consequences of the
strike by the theater owners.
Statement (i) is not a correct option as people may be able to
watch movies in mediums other than cinema halls.
Statement (ii) is also not a correct option as the strike will be
against the municipal tax which is being levied by the local state
government.
Statement (iii) is also not true for the same reason as mentioned
above.
26 Adda247 Publications For any detail, mail us at
Publications@adda247.com
Directions (17-18):
17. (a); In the above question it is clear that statement I will be the cause
and statement II will be its effect. Because Mallya has taken loan
from various banks in India and has not been able to repay it for
some time. So this will be the reason of Vijay Mallya’s
disapproval by the audience at the stadium

18. (b); It is clear from the above statement that statement II will be
cause and statement I will be its effect. Because books were
created in 2007 quite a while before it will be a cause and it
need to be updated now so it will be its effect.

19. (c); In the above question we have to find which statement negates
the given statement.
For I: This statement supports the given statement as it
describes about the revolution that technology has bring in India
which is a mark of achievement of Modi’s government.
For II-This statement also supports the given statement as it
describes the country’s growth in Modi government’s tenure
which is considered as victory of his government.
For III-This negates of the given statement as it defines the
problems of poverty and corruption that India is still facing
which can be considered as flaw of Modi’s government.
For IV-It supports the given statement as number of FDI’s has
increased in India in current scenario as given in the statement.

20. (b); In the statement we have to pick the statement which can be
assumed from above statement.
For Statement I. Wrong, Because it was not possible to destroy
the existing stocks by the end of this month that is why the
companies pleaded to the Supreme Court.
For Statement II. Right, Because it is clear from the above
statement that the court had fixed the deadline 31 may earlier
and there after extended it to 31 july.
For Statement III. Right, Because to dispose off stock and
transporting it out of the state requires clearance.

27 Adda247 Publications For any detail, mail us at


Publications@adda247.com
21. (e); Except (e), all the statement strengthens the given statement by
pointing out the effects of water scarcity India is currently facing
and problems related to it but option (e) states that the crisis
was more in 1960 from now which contradicts the given
statement as it states that India is facing its “worst” water crisis
in history. So (e) weakens the given statement.

22. (e); Either (I) or (II) can be assumed from the given statement as
better opportunities or virus can be the reason behind the major
step taken by the population of village A.

23. (e); Statement (e), can be assumed from the given statement as
better quality is always a major factor of concern for customers
and for it paying a bit higher amount is convenient. All other
option except (e) cannot be assumed quality and price offered
by company V for the product is not mention in the given
statement. And also quality is only parameter to increase the
number of customer or it is the only way also cannot be
hypothesized from the given statement.

24. (c); Only statement I can be inferred from the given statement as
HRA allowance increase after reallocation which clearly
indicates that both are directly related to each other. But II and
III cannot be inferred as we cannot state that strike is the reason
behind the rise in allowance. And also increase in profit cannot
be a factor to increase the allowance which the statement also
states that the decision has been taken after the reallocation.

25. (c); Only (c) strengthen the given statement as it states that interest
of people of India is rising in football which will help the skilled
youth to indulge in this game. But (a) and (b) suggest that
Cricket is the most lovable game and football require star
players to become as popular as cricket players among Indians.
Further (d) is about the extraordinary players of Hockey in India
and (e) is about superiority of Indian Women players over Men
players in Olympics game

28 Adda247 Publications For any detail, mail us at


Publications@adda247.com
26. (d); In the above question we have to find which statement can be
concluded from the given passage.
For I-This is not true because it is not given in the above passage
that Google’s market is not good that’s why Google has decided
to launch new service.
For II-This statement is irrelevant. Because Google is going to
launch new service it does not mean that Google is not
interested to provide other services, we can’t conclude this from
the given passage.
For III- There is a sentence in the passage “The results will aim
to streamline such listings by eliminating duplicate jobs posted
on different sites.” It clarifies the statement III. So we can
conclude statement III from the given passage.
For IV- This is vague in the given context because Google will
also show employer ratings from current and former workers, it
is given in the passage.

Direction (27-28):
27. (b); In this question we have to choose that statement which can be
assumed from the passage.
For Statement I: Right, As passage says that Travis Kalanick
helped Uber built into colossus (enormous importance).
Statement I says same thing that Travis Kalanick helped Uber
built into prodigious.
For Statement II: Wrong, from passage we can’t assume that
shareholders forced or ordered Travis Kalanick to give-up from
the post of chief executive of Uber. He stepped down as chief
executive, after a seemingly endless series of scandals raised
doubts over his leadership, no one forced him to do so.
For Statement III: Right, There is a sentence in a passage
“Meanwhile, Shareholders released a damning report on the
firm's management culture”, it clarifies that the Shareholders
released damning (fatal, condemnatory) report against
management’s culture.

29 Adda247 Publications For any detail, mail us at


Publications@adda247.com
28. (e); In this question, we have to select that statement which is out of
the context in regarding to the above passage.
For Statement I: from passage we can get an idea that Travis
Kalanick stepped down as chief executive because of the reason
and there were some other reasons behind the resignation.
For Statement II: As in passage these all are the reasons behind
the Travis Kalanick stepped down as chief executive of Uber.
For Statement III: As in passage it is mentioned, Travis Kalanick
spent most of the past decade turning a taxi app (Uber) into the
world's most valuable startup.

29. (c); In this question we have to find out which of the given courses of
actions will be appropriate in light of the statement given.
Statement (i) is an appropriate course of actions as it is
necessary to take some disciplinary actions against the staff
member so that such incident do no repeat in future.
Statement (ii) is not appropriate as the club has already
apologized the woman as stated in the given media report.
Statement (iii) is also not an appropriate course of action as the
main issue was discrimination. The club apologized not because
the given lady was not a maid but because of a discriminatory
remark made by its employee.

30. (c); In this question we have to find out the possible consequences of
India’s entry into IETI.
Statement (i) is a possible outcome as speedy entry would
definitely encourage more people to visit US.
Statement (ii) not correct as there is not enough evidence in the
statement to make that deduction, Just because any other
country did so does not mean it has to be repeated by India also.
Statement (iii) is also not a correct option as it lacks sufficient
evidence.

30 Adda247 Publications For any detail, mail us at


Publications@adda247.com
31. (d); Only II course of action follows as safety of the people and
environment protection require transparent reforms and
policies. But I and III does not follow as even by considering the
fact that this tourism industry provide employment to so many
people but the lives of so many others cannot be put at stake for
it. And also, III is not feasible as we cannot relocate app. lakhs of
people to reduce the pollution level near rivers.

32. (a); Only I and II can be assumed from the given statement as I is
directly given in the statement that investigating agencies are
going to use the Aadhar database for their investigations which
includes the fingerprints of criminals. And II can also be
assumed from the given statement which states that limited
access leads to assurance and clearly with increase in access,
vulnerability also increases. While III cannot be assumed as it is
not mentioned in the statement.

33. (a); Statement I strengthen the given statement as is states about the
amount invested on disasters and climate related
mishappenings which is also a cause for rising the number of
hungry people. But statement II cannot be assumed from the
given statement as it is about increase in pollution level.

34. (d); Only I can be the reason as the monsoon cleans the air particles
which caused the satisfactory level of wind blow. But II and III
cannot be the reason as it states about the dusty or severe air
flow due to emergency level of air pollution.
35. (b); Clearly Statement I is the effect of statement II as the Delhi
metro employees are protesting which is the effect and the
cause of this protest is hike in the salaries and pay grades.

Direction (36-38):
36. (d); Argument I is not strong due to the word ‘only’. Also it cannot be
said that Different interest rates encourage the customers to
switch the banks. So, Argument II is also not strong.

31 Adda247 Publications For any detail, mail us at


Publications@adda247.com
37. (b); Argument I is not strong because it is not directly related with
the statement. Argument II is strong because it states that
History should not be taught to school children in a one-sided
manner and also if its content is twisted than it should be
corrected.

38. (a); Argument I is strong because when huge quantity of cheaper


Vanaspati ghee will enter in the market, then it will affect the
interest of farmers and industries. Argument II is absurd so it is
not strong.

Directions (39-40):
39. (a); Statements II is cause and I is its effect. As statement II states
that 70 million people will die of AIDS in upcoming time in worst
affected countries and statement I states that officials donated
some amount to the global AIDS fund and it will help 2 million
people for their treatment so it is an effect which will turn out
from the report of UN.

40. (a); Statements II is the cause and Statements I is its effect as after
the fire burned out in eastern Goma, its people were forced to
relocate to Rwanda.

32 Adda247 Publications For any detail, mail us at


Publications@adda247.com
 Key Points to Remember for New Pattern Questions
➢ These questions are of much importance for the mains
exam. As now a days these questions are mostly related
to the general life or current issues.
➢ In the recent exams you would have seen the questions
which strengthen or weakens the given statement.
These also includes the neutral statements, so you have
to be careful and analytical while answering the
questions.
➢ You have to stick to the statement as you cannot be
judgmental, politically correct or emotional while
answering the question as you have to answer the
questions only by considering the statement 100% true.
➢ In recent exams sometimes, you would have seen
reverse in pattern, as the course of action or the
assumption is given, and you have to find the statement
which should be followed by the given course of action
or assumption.
➢ There is a point that requires attention i.e. there is a
difference between conclusion, inference and
assumption. Conclusion is something that you can
directly obtain from a statement. Inference is something
which can be inferred indirectly i.e. on the basis of
evidence and reasoning.

33 Adda247 Publications For any detail, mail us at


Publications@adda247.com
Cracker Book for Bank (IBPS | SBI | RRB PO | Clerk) Mains Exams

1 Adda247 Publications For any detail, mail us at


Publications@adda247.com
Cracker Book for Bank (IBPS | SBI | RRB PO | Clerk) Mains Exams

Chapter

9 Direction

BEST APPROACH TO SOLVE THE QUESTIONS

Sometimes a topic can help you to score good marks in that particular
section. So, you should not miss any of them. The topic of direction may also
play an important role for you to score good marks. These days the
questions asked from this topic are quiet tricky but if you have practiced
enough and your concepts are clear then it can be a game changer for you.

Example-1:

Directions: In the following questions, the symbols #, &, @ and $ are used
with the following meanings as illustrated below. Study the following
information and answer the given questions:
(IBPS PO Mains 2017)
Note: The directions which are given indicates exact directions.
P#Q:- Q is in the south direction of P.
P@Q:- Q is in the north direction of P
P&Q:- Q is in the east direction of P at distance of either 12m or 6m
P$Q:- Q is in the west direction of P at distance of either 15m or 3m.
P#&Q:- Q or P is in the southeast direction of P or Q.
P@&Q:- P or Q is in the northeast direction of Q or P.

1. If A&B#&C$D&E@F are related to each other such that F is placed


exactly between A and B on line AB. Similarly D and A are vertically
inline then what is the possible shortest distance between F and B
when DE (length of segment DE) < DC/2 and EC < 10m?
(a) 12m (b) 5 m (c) 4 m
(d) 6m (e) None of these

2 Adda247 Publications For any detail, mail us at


Publications@adda247.com
Cracker Book for Bank (IBPS | SBI | RRB PO | Clerk) Mains Exams

2. If A&B#&C$D&E@F are related to each other such that D, A are inline,


DE=6 m and a perpendicular drawn from E on AB divides AB in two
equal parts and D@&B then what is the probable direction of A with
respect to F?
(a) North-west (b) West (c) South-west
(d) east (e) Can’t be determined

3. K#&T$M#&S&K&Z are related to each other such that K is in north of


M then what is the probable direction of Z with respect to M?
(a) North-east (b) West (c) South-west
(d) east (e) Can’t be determined

4. K#&T$M#&S&K&Z are related to each other such that K is in north of


M. M and K are inline vertically when MT>SK then what is the distance
between S and Z When KZ=12 m?
(a) 24m (b) 18m (c) 15m
(d) Either (a) or (b) (e) None of these

5. K#&T$M#&S&K&Z are related to each other such that K is in north of


M. M and K are inline vertically when MT>SK then what is the sum of
SK and MT?
(a) 27m (b) 21m (c) 15m
(d) Either (a) or (b) (e) Either (b) or (c)

Explanations (The Approach)

1. (d); It is given that DE < DC/2 so the value of DC = 15m and AB= 12m
or 6m
D and A are inline so DE=AF,
If DE = 6m, EC = 15-6 = 9m
If DE= 3M, EC= 15-3 = 12m
It is given that EC<10 so DE= 6m and AB = 12m
Hence FB =6m
3 Adda247 Publications For any detail, mail us at
Publications@adda247.com
Cracker Book for Bank (IBPS | SBI | RRB PO | Clerk) Mains Exams

2. (e); The direction of F with respect to E is given but the exact


distance from E is not given so we can’t find out the exact
position of F hence the direction of F with respect to A can’t be
determined.

3. (a); There are four possible possibilities but two will cancel out by
the condition that K is in east of S and K is in north of M so the
final figure is given below.
It is clear that Z is in north east from M.

4. (d); It is given that MT>SK so MT=15m KZ= 12m and SK =12m or 6m


so the value of SZ
= (12+12) or (12+6) = 24m or 18 m
4 Adda247 Publications For any detail, mail us at
Publications@adda247.com
Cracker Book for Bank (IBPS | SBI | RRB PO | Clerk) Mains Exams

5. (d); It is given that MT>SK so MT=15m and SK =12m or 6m


SK + MT = (12+15) or (15+6) =27m or 21m

Example-2:

Directions: Study the following information and answer the questions


given below:
There are AB axis in such a way that A is in north and B is in south
direction. There is XY axis in such a way that X is in west direction and Y is
in east direction. AB axis and XY axis intersect at a point Q in such a way
that AQ is 15m, QB is 17m, QX is 12m, QY is 24 m.

Mehul starts from point X and walks 20m in south direction and then he
turns his left and walks 32m. Arun starts from point A and walks 20m in
east direction. Raju starts from point Y and walks 5m in north direction and
then he turns his left and walk 4m and again he turns his left and walks
22m. (IBPS Clerk Mains 2017)

1. Point B is in which direction with respect to Arun’s current position?


(a) south (b) south-east (c) south-west
(d) west (e) north-west
5 Adda247 Publications For any detail, mail us at
Publications@adda247.com
Cracker Book for Bank (IBPS | SBI | RRB PO | Clerk) Mains Exams

2. Point Y is in which direction with respect to Mehul’s current position?


(a) north (b) east (c) north-east
(d) north-west (e) south

3. What is distance between Raju’s current position and Mehul’s current


position?
(a) 3m (b) 5m (c) 13m
(d) 22m (e)27m

Explanation (The Approach):

Solutions (1-3):

1. (c); 2. (c); 3. (a);

Example-3:
Directions: Study the following information carefully and answer the
questions given below.
Six cars C, D, P, Q, R, S are parked in a row facing north at a distance which
is successive multiple of 4m in an increasing order from the left end from
each other. Car S is second to the right of car P. The total distance between
car R and Q is 52m. Only one car is parked in between car D and R. Car S
6 Adda247 Publications For any detail, mail us at
Publications@adda247.com
Cracker Book for Bank (IBPS | SBI | RRB PO | Clerk) Mains Exams

and car Q are parked next to each other. Car C and car S are not parked next
to D. Now Car D starts moving towards north direction after moving 10m its
takes a right turn and stops at point T after moving 28m. Car C starts
moving in east direction and after going 12m it turns right and move 20m
and then again turn right and move 72m and stops there at point H. Car P
starts moving in south direction and after moving 10m it takes a left turn
and moves 20m then it again takes a left turn and moves 5m. From there it
takes a right turn and moves 24m and stops at point V. (SBI Clerk Mains
2018)

1. How many cars are parked there in between cars ‘D’ and ‘Q’?
(a) Two (b) None (c) More than three
(d) One (e) Three

2. What is the distance between point ‘V’ and point ‘H’?


(a) 25 m
(b) 15 m
(c) They don’t align in the same straight line
(d) 20 m
(e) 10 m

3. What is the distance and direction of initial position of Car ‘C’ with
respect to the initial position of Car ‘P’?
(a) 100 m towards east
(b) 50 m towards west
(c) 104 m towards east
(d) 72 m towards south-east
(e) None of these

4. Which car will be met first, if ‘car D’ moves through the shortest
distance from point ‘T’?
(a) R (b) S (c) Q
(d) P (e) None of these

5. What is the current position of car ‘R’ with respect to initial position of
car ‘C’?
(a) Immediate right (b) Second to the left (c) Third to the left
(d) Second to the right (e) None of the above
7 Adda247 Publications For any detail, mail us at
Publications@adda247.com
Cracker Book for Bank (IBPS | SBI | RRB PO | Clerk) Mains Exams

Explanations (The Approach):

Solutions (1-5):
Car S is second to the right of car P. The total distance between car R and Q
is 52m. Only one car is parked in between car D and R. Car S and car Q are
parked next to each other. Car C and car S are not parked next to D.

Now Car D starts moving towards north direction after moving 10m its
takes a right turn and stops at point T after moving 28m. Car C starts
moving in east direction and after going 12m it turns right and move 20m
and then again turn right and move 72m and stops there at point H.

Car P starts moving in south direction and after moving 10m it takes a left
turn and moves 20m then it again takes a left turn and moves 5m. From
there it takes a right turn and moves 24m and stops at point V.

1. (e); 2. (b); 3. (c);

4. (a); 5. (c);
8 Adda247 Publications For any detail, mail us at
Publications@adda247.com
Cracker Book for Bank (IBPS | SBI | RRB PO | Clerk) Mains Exams

Practice Exercise Based on new Pattern

Directions (1-2): Read the given information carefully to answer the


following questions.

Tom start walking in a certain direction to catch Jerry. After walking 5m


in the same direction he turned left and walk 4m. From there he turns
towards his right and walk 3m. Then he takes another right turn and
walked 6m. Now Jerry and Tom are facing each other as they are inline 6m
apart from each other at point B and A respectively. From point B, Jerry
immediately takes a left turn and after walking a certain distance, he
stopped. Now the shortest distance between Tom and Jerry is 10m and Tom
who is still at point A, is to the north-west of Jerry.

1. In which direction did Tom started walking?


(a) North (b) West (c) South
(d) East (e) None of these

2. What is the distance between the points at Jerry stopped from the point
he starts walking?
(a) 7m (b) 5m (c) 10m
(d) 8m (e) None of these

Direction (3-4): In the following questions, the symbols #, *, @ and $ are


used with the following meanings as illustrated below. Study the following
information and answer the given questions:

Note: The directions which are given indicated exact directions.


P*Q :– Q or P is in the south direction of P or Q at distance of 6m.
P@Q :– Q or P is in the north direction of P or Q at distance of 3m
P#Q :– Q or P is in the east direction of P or Q at distance of 5m
P$Q :– Q or P is in the west direction of P or Q at distance of 4m.
P*#Q :– Q or P is in the southeast direction of P or Q.
P*$Q :– Q or P is in the southwest direction of P or Q
P@#Q :– Q or P is in the northeast direction of P or Q.
P@$Q :– Q or P is in the northwest direction of P or Q.
9 Adda247 Publications For any detail, mail us at
Publications@adda247.com
Cracker Book for Bank (IBPS | SBI | RRB PO | Clerk) Mains Exams

3. If B@$C#D*A is related to each other, point A, D and B are inline and


Point B is the midpoint of AD and C is not in the northwest of A then
find out the probable distance between point B and C?
(a) √35 m (b) √39 m (c) 5√2 m
(d) √34 m (e) None of these

4. If B@#C#D*A is related to each other, point B is to the east of point A


and the distance between them is 8m and B is to the northeast of D
then find out the how far and in which direction is B with respect to C?
(a) 2√5 m, South
(b) 3√5 m, North-east
(c) √35 m, West
(d) 5 m, Southeast
(e) None of these

Directions (5-6): Study the following information carefully and answer the
questions given below.

Six persons i.e. A, B, C, D, E and F are sitting in a open ground such that
they are at certain distance with their immediate neighbours. F sits 4m
northeast direction to B. B is in the east of E, who is 4m in the east of person
A. C is 4m in the north of E. The distance between B and E is same as the
distance between D and A. D is 3m south of A.

5. In which direction and what distance is D with respect to E?


(a) North 5m (b) South 4m (c) Southwest, 5m
(d) Southeast, 5m (e) None of these

6. What is the shortest distance between B and A?


(a) 6m (b) 7m (c) 4m
(d) 3m (e) None of these

10 Adda247 Publications For any detail, mail us at


Publications@adda247.com
Cracker Book for Bank (IBPS | SBI | RRB PO | Clerk) Mains Exams

Directions (7-9): Study the following information carefully to answer the


questions given below:

Six cars P, Q, R, S, U, V are parked in the parking and each of them is of


different lengths. Four cars are parked in a rectangular formation such that
the car which is just longer than Q is in north east of it and the car which is
just longer than V is in north west of it. Both Q and V are 8km apart and are
horizontally inline. Only two cars are shorter than R. Car S is longer than P
and car V is longer than Q. Two cars are in north of car U. Car R and U are
not in line. The longest car is in north-west of P. U is not the second longest
car. Longest car is not in the west of the third shortest car. The distance
between the second shortest and second longest car is 6km.

7. What is the shortest distance between car R and Q?


(a) 6km (b) 7km (c) 10km
(d) 14km (e) None of these

8. If the shortest distance between car U and V is 10km then what is the
distance between car P and U?
(a) 16km (b) 17km (c) 20km
(d) 12km (e) None of these

9. The longest car is in which direction with respect to the shortest car?
(a) North (b) South (c) northwest
(d) Southeast (e) None of these

Directions (10-12): Study the following information carefully and answer


the questions given below:

A man was initially walking along the diagonal of a rectangular playground


such that the longer side was in horizontal axis and the shorter on the
vertical axis. He walked only along the perimeter of the rectangle, diagonal
or along the median of the rectangle. He started from the midpoint of the
ground towards southwest and walked 6.5km, then took a turn and walked
for 6km along the perimeter and 2.5 km along the median to reach the

11 Adda247 Publications For any detail, mail us at


Publications@adda247.com
Cracker Book for Bank (IBPS | SBI | RRB PO | Clerk) Mains Exams

starting point. He then turn towards right and walked 4km and finally took
a left turn and walked 1 km. Finally, he is 2km away from the shorter side
and 1.5 km away from the longer side.

10. In which direction is he finally walking?


(a) North (b) South (c) Northeast
(d) Southeast (e) None of these

11. What is the total length of the shorter side of the rectangle?
(a) 10km (b) 4km (c) 8km
(d) 5km (e) None of these

12. In which direction is A is in with respect to its starting position?


(a) Southeast (b) South (c) Northeast
(d) Southwest (e) None of these

Direction (13-14): Study the following information carefully and answer


the question given below-
Point F is to the north of E. Point A is to the west of B. Point B is to the south
of point C. Point F is to the north east of point C which is in-line with point
E. Point D is exactly in between point C and E. Point J is to the north of point
D. Point K is 5m away from point J and is exactly horizontally in line with
point J. Point B is 8m in the south of Point K. The distance between point F
and E is 4m.

13. If the distance between point K and F is 8m and both points are inline,
then what will the shortest distance between point J and E?
(a) 6 m (b) 5 m (c) 7 m
(d) 8 m (e) None of these

14. Point F is in which direction from point A?


(a) North East (b) North West
(c) South West (d) South East
(e) None of these
12 Adda247 Publications For any detail, mail us at
Publications@adda247.com
Cracker Book for Bank (IBPS | SBI | RRB PO | Clerk) Mains Exams

Direction (15-17): Study the following information carefully and answer


the question below-
Point A is 5km in north of point B. Point B is 6km in west point C. Point C is
exactly between point E and F. Point E is horizontally inline with point A.
Point F is as far as from point D (horizontally inline) as Point A from point
E. Point D is vertically not inline with point B.

15. If point G is 6km east of point E then point G is how far and in which
direction from point D?
(a) 6km, North (b) 5km, North-east (c) 10km, North
(d) 14km, South (e) None of these

16. Point A is in which direction from point F?


(a) North (b) South (c) northwest
(d) Southeast (e) None of these

17. How far is point E from point B?


(a) 6 km (b) √87 km (c) √61 km
(d) 8 km (e) None of these

Directions (18-20): Read the following information carefully and answer


the questions that follows:
Six persons i.e. A, B, D, E, G and H are standing in a ground at seven different
points.
(i) B is standing 3m to the west of A.
(ii) E is standing 7m west of person D who is standing 3m east of G.
(iii) H is standing 4m south of D.
(iv) A is standing in 5m south east direction of point E.

18. What is the shortest distance between G and H?


(a) 4m (b) 5m (c) 2m
(d) 3m (e) Cannot be determined

19. How many persons are standing in the south west direction of G if E is
perpendicular to B?
(a) Three (b) More than three (c) Two
(d) None of these (e) Cannot be determined
13 Adda247 Publications For any detail, mail us at
Publications@adda247.com
Cracker Book for Bank (IBPS | SBI | RRB PO | Clerk) Mains Exams

20. How many Persons are standing to the west of G?


(a) None (b) Two (c) Four
(d) Three (e) None of these

Directions (21-22): Read the following information carefully to answer the


questions that follow. The questions are based on following coding formats:
P1Q means – P is in North of Q
P2Q – P is in South of Q
P3Q – P is in East of Q
P4Q – P is in West of Q
@ means – Either 2 or 7 m
# – Either 5 or 10 m
& – Either 8 or 13 m
Conditions given are as:
I. A @1 B
II. A &3 D
III. F #3 E
IV. G @4 F
V. E #1 D

21. What could be the shortest distance between E and B considering the
smallest among the given two-possible distances?
(a) √113 (b) √111 (c) √115
(d) √117 (e) none of these

22. In which direction is point G with respect to D?


(a) south west (b) north west (c) north
(d) north east (e) can’t be determined

Directions (23-24): Following questions are based on the information


given below.

Ajay start driving from his house to his friend Jeet’s house. He start driving
in the west direction from his house by a car. After going 15m he takes a
right turn from point N and continue driving for 20m and reaches the point

14 Adda247 Publications For any detail, mail us at


Publications@adda247.com
Cracker Book for Bank (IBPS | SBI | RRB PO | Clerk) Mains Exams

P. From point P he takes a right turn and after driving 30m, reaches the
point Q. from point Q he takes a left turn and continue moving 20 m to
reach Jeet’s house. In the way Ajay stop a Shop at point O which is exactly
between P and Q.

23. Find the shortest distance between jeet’s house and point O?
(a) 35m (b) 30m (c) 25m
(d) 10√13 (e) None of these

24. Jeet’s house is in which direction from Ajay’s house?


(a) North (b) North East
(c) North West (d) South West
(e) South

Directions (25-27): These questions are based on the information given


below:
Eight persons A, B, C, D, E, F, G and H are standing in such a way that C is 20
m apart from B towards West, B is 30 m South with respect to A. A is 40 m
towards West with respect to E. D is 50 m towards South with respect to E.
F is 15 m apart from G towards North. H is 20 m towards East with respect
to G. F is 40 m towards West with respect to D.

25. In which direction is B standing with respect to F?


(a) North-West
(b) North
(c) North-East
(d) Cannot be determined
(e) None of these

26. If one more person I is standing towards South-West with respect to


D, then in which direction is H, standing with respect to I?
(a) South-West
(b) North-East
(c) North-West
(d) Cannot be determined
(e) None of these
15 Adda247 Publications For any detail, mail us at
Publications@adda247.com
Cracker Book for Bank (IBPS | SBI | RRB PO | Clerk) Mains Exams

27. What is the direction of C with respect to H?


(a) North-West (b) North (c) West
(d) Cannot be determined (e) None of these

28. Mohan starts walking in the west direction and walks for 8 km, then
he takes a right turn and walks for 5 km again he takes a right turn
and walks for 2.5 km and at last he takes a right turn walks for 5 km,
then in which direction and at what distance is Mohan from his initial
position?
(a) 5 km west (b) 5.5 km East (c) 5 Km East
(d) None of these (e) 5.5 km west

Direction (29-30): Two friends Silva and Jones started walking from two
different points. Silva, who started from point A moved along a circular
track and after walking 11km he reached point B, from where he turned left
and walked 5km to reach point C after which he stopped. Jones started from
point F and after walking 5km, he reached point E. From point E he turned
45 degree towards his left and walked √8 km to reach point D. After
reaching point D, he turned 45 degrees in clockwise direction. Then, he
moved ahead 5km to reach point C which is towards the west of point D.
Point B, C and D are in straight line.

29. In which direction was Silva facing at instant he travelled three-fourth


of his total journey?
(a) East (b) West (c) North East
(d) South East (e) Cannot be determined

30. If Silva keeps on walking ahead from point C and reached point G after
walking 12 km, Point F is to the north of point G then what is the
shortest distance between point F and G?
(a) 7km (b) 4km (c) 2km
(d) More than 7km (e) None of these

16 Adda247 Publications For any detail, mail us at


Publications@adda247.com
Cracker Book for Bank (IBPS | SBI | RRB PO | Clerk) Mains Exams

Solutions

Solutions (1-2):
As from the given conditions we get that only possible condition is that Tom
start walking in North direction.

1. (a);

2. (d); distance between the points at Jerry stopped from the point he
starts walking= √102 − 62 m = 8m

Solutions (3-4):
3. (d); The distance between point B and C= √32 + 52 = √34 m

17 Adda247 Publications For any detail, mail us at


Publications@adda247.com
Cracker Book for Bank (IBPS | SBI | RRB PO | Clerk) Mains Exams

4. (b); The distance between B and C = √62 + 32 = √45 m

Solutions (5-6):

5. (c); 6. (b);

Solutions (7-9):
Four cars are parked in a rectangular formation such that the car which is
just longer than Q is in north east of it and the car which is just longer than
V is in north west of it. Both Q and V are 8km apart and are horizontally
inline. Only two cars are shorter than R. So, from this we get that----

> > > R> >


Car S is longer than P and car V is longer than Q. So, from this we know that
either S or U is the longest Car. Two cars are in north of car U. Car R and U
are not in line. The longest car is in north-west of P. U is not the second
longest car. Longest car is not in the west of the third shortest car. The
18 Adda247 Publications For any detail, mail us at
Publications@adda247.com
Cracker Book for Bank (IBPS | SBI | RRB PO | Clerk) Mains Exams

distance between the second shortest and second longest car is 6km. So, Q
can be either shortest or second shortest.
Consider case-1, Q is the shortest, so S or U can be longest but we know that
Two cars are in north of car U and the longest car is in north-west of P
which is not possible so it will get eliminated. Further if S is the longest car,
the longest car is in north-west of P and Longest car is not in the west of the
third shortest car, so this will also get eliminated.
In case-2, Q is the second shortest car then R will be just longer than it and
will be in north of car V. And U can’t be the longest car as already seen from
the given conditions, so clearly S will be the longest car and U will be the
shortest car. So, the final arrangement is----------

S>P>V>R>Q>U

7. (c); Distance between car R and Q = √82 + 62 = 10km

8. (d); shortest distance between car U and V is 10km


Then distance between Q and U is = √102 − 82 = 6km
Distance between car P and U = 6 + 6 = 12km

9. (c);

Solutions (10-12):

10. (a); 11. (d); 12. (c);


19 Adda247 Publications For any detail, mail us at
Publications@adda247.com
Cracker Book for Bank (IBPS | SBI | RRB PO | Clerk) Mains Exams

Solutions (13-14):

13. (b); DE = JF = KF – KJ
DE = JF = 8 – 5 = 3m
JD = FE=4m
JE = √32 + 42 =5m

14. (a);

Solutions (15-17):

15. (c); 16. (c); 17. (c);

Solutions (18-20):

18. (b); 19. (c); 20. (e);


20 Adda247 Publications For any detail, mail us at
Publications@adda247.com
Cracker Book for Bank (IBPS | SBI | RRB PO | Clerk) Mains Exams

Solutions (21-22):

21. (a); 22. (e);

Solutions (23-24):

23. (c); 24. (b);

25. (b);

B is standing in North direction with respect to F.

21 Adda247 Publications For any detail, mail us at


Publications@adda247.com
Cracker Book for Bank (IBPS | SBI | RRB PO | Clerk) Mains Exams

26. (d);

As total distance between I and D is not given, so the position of


H with respect of I cannot be determined.
27. (a);

C is in the North-West direction from H.


28. (e);

29. (a);

30. (c);

22 Adda247 Publications For any detail, mail us at


Publications@adda247.com
Cracker Book for Bank (IBPS | SBI | RRB PO | Clerk) Mains Exams

 Key Points to Remember for New Pattern Questions


➢ The direction-based puzzle is also newly introduced in
the exam, in this there are certain symbol used to define
the directions with the distance. Sometimes the distance
and direction is given by combining it with either/or so
start solving it by drawing both the conditions
simultaneously.
➢ Solving questions of direction requires blindly following
the path as you have to draw as per the directions given
in the question.
➢ After examining from the recent exam, you may get to
know that each and every question is different from the
other one. And a separate diagram is required to be made
from it.
➢ A new kind of questions that has been seen in the recent
exam comprises Axes with the direction information. In
this we require to draw the diagram on the axes XY.
➢ A puzzle form of direction question comprising of a linear
row having different elements placed in it as it is also a
new pattern question. In this the type of question first the
elements should be arranged and then their movement in
different direction should be consider.

23 Adda247 Publications For any detail, mail us at


Publications@adda247.com
Cracker Book for Bank (IBPS | SBI | RRB PO | Clerk) Mains Exams

1 Adda247 Publications For any detail, mail us at


Publications@adda247.com
Cracker Book for Bank (IBPS | SBI | RRB PO | Clerk) Mains Exams

Chapter

10 Blood Relation

BEST APPROACH TO SOLVE THE QUESTIONS

Now a days as you can see there has been a constant change in the
pattern of each and every topic. So, you have to be prepared for all those
changes. That is reason why we are providing you questions based on
recent exams, once you go through all these you may be able to understand
the recent changes in the current examination.

Example-1:
In the following questions, the symbols #, &, @, * , $, % and © are used with
the following meanings as illustrated below. Study the following
information and answer the given questions: (IBPS PO Mains 2017)

P#Q - P is the son of Q.


P@Q - Q is the child of P.
P©Q - P is the parent of Q.
P$Q - P is elder than Q.
P*Q- P is the husband of Q.
P&Q- Q is the daughter-in-law of P.
P%Q- P is the wife of Q.

1. If A@B*D&G%E$F#D then how F is related A?


(a) Grandfather (b) Grandson (c) Daughter
(d) Wife (e) None of these

2. If H*M©O$N#M, the age of N is 20 years and of H is 40 years then


what is the probable age of O?
(a)17 years (b)15 years (c)23 years
(d) 45 years (e)12 years

2 Adda247 Publications For any detail, mail us at


Publications@adda247.com
Cracker Book for Bank (IBPS | SBI | RRB PO | Clerk) Mains Exams

Direction (1-2):
1. (b);

2. (c);

Example-2:
Study the information carefully and answer the questions given below.
P is the husband of Q. R is the grandchild of P.P has only one child(son) who
is married to T’s child. T has only two children one son and one daughter. X
is grandson of T. S is brother in law of son of T. U and V are children of T. W
is married to the son of T. X is son of U’s brother. (SBI PO Mains 2017)
1. If T is married to Y than how is T related to R?
(a) Grandfather (b) Grandmother
(c) Maternal Grandfather (d) Maternal Grandmother
(e) Either (c) or (d)

2. How is X related to V?
(a) Son (b) Daughter (c) Son in law
(d) Daughter in law (e) Husband

Directions (1-2):

1. (e); 2. (a);
3 Adda247 Publications For any detail, mail us at
Publications@adda247.com
Cracker Book for Bank (IBPS | SBI | RRB PO | Clerk) Mains Exams

Example-3:
Study the following information carefully and answer the questions given
below.
Six persons J, L, R, O, X, Z of a family born in six different years 1978, 1982,
1995, 1997, 2013 and 2015. (All the ages of the given persons have been
calculated on base year 2018 and all the persons are considered to be born
on same date of same month). There is a difference of 18 years between the
ages of O and J. O is the father of J. L is younger than R. X is not the oldest
person of the family. R is not married to Z. Z was born in an even numbered
year. L is the only granddaughter of the mother of O. Father of O is 4 years
older than O’s mother. J is the son of R. (SBI PO Mains 2018); Z was born in
an even numbered year. L is the only granddaughter of the mother of O.
Father of O is 4 years older than O’s mother. There is a difference of 18
years between the ages of O and J. O is the father of J. J is the son of R.
Case-1: Case-2:
Years Persons Years Persons
1978 1978
1982 1982
1995 O 1995
1997 1997 O
2013 J 2013
2015 2015 J

L is younger than R. X is not the oldest person of the family. R is not married
to Z. Z was born in an even numbered year. Father of O is 4 years older than
O’s mother.

4 Adda247 Publications For any detail, mail us at


Publications@adda247.com
Cracker Book for Bank (IBPS | SBI | RRB PO | Clerk) Mains Exams

Years Persons
1978 Z
1982 X
1995 O
1997 R
2013 J
2015 L

Practice Exercise Based on new Pattern

Direction (1-2): In the following questions, the symbols #, &, @, *, $, % and


© are used with the following meanings as illustrated below. Study the
following information and answer the given questions:
A@B- A is the child of B.
A©B- A is the parent of B
A%B- A is elder to B
A&B- A is younger to B
A$B- A is brother of B
A*B- A is wife of B
A#B- A is sister-in-law of B

1. If X©F$D&Q@X©E%D, the age of E is 25 years and age of Q is 32


years, so what can be the age of D?
(a)27 years (b)23 years (c)29 years
(d)30 years (e)34 years

2. If G*J$K©Y@V&C and V#G then how K is related to Y?


(a) father (b) Brother-in-law
(c) daughter-in-law (d) Sister (e) None of these
5 Adda247 Publications For any detail, mail us at
Publications@adda247.com
Cracker Book for Bank (IBPS | SBI | RRB PO | Clerk) Mains Exams

Directions (3-5): Study the following information carefully and answer the
questions given below.
In a family of ten people C, G, H, Q, R, S, T, K, U, Y. G and H are married
couple. Q is the only sister of R. S is the grandfather of T. U is the son-in-law
of H. C is the only child of G. R is grandchild of G. T is the child of U. Q is
unmarried. H is the sister-in-law of K. H has no sibling and G is the only son
of Y.

3. If J is the son-in-law of C, then how is R related to C?


(a) Grandson (b) Brother-in-law (c) Daughter
(d) Father-in-law (e) None of these

4. How is Y related to C?
(a) Grandfather (b) Grandmother (c) Mother
(d) Can’t be determined (e) Sister

5. How is Q related to S?
(a) Grandson (b) Brother-in-law (c) Granddaughter
(d) Father-in-law (e) None of these

Directions (6-7): Study the following information carefully and answer the
questions given below.

There are some members in the family. L is brother of M, who has only one
son. A is the sister-in-law of R and is the mother of D. R is the child of M. A is
daughter-in-law of K. G and T are siblings. G is the son of L. S is the sister-
in-law of M and has only two sons. D is not the female member of the family.
T and G are the cousins of J. S is not the sister of K, who is the grandmother
of E. J has only one daughter. R is the daughter of K.

6. How is M related to J?
(a) Brother (b) Mother (c) Father
(d) Sister (e) None of these

7. How is M related to ‘T’?


(a) Father (b) Brother-in-law (c) Mother
(d) Grandfather (e) None of these
6 Adda247 Publications For any detail, mail us at
Publications@adda247.com
Cracker Book for Bank (IBPS | SBI | RRB PO | Clerk) Mains Exams

Direction (8-10): In the following questions, the symbols #, &, @, * , $, %


and © are used with the following meanings as illustrated below. Study the
following information and answer the given questions:
A@B- A is the child of B.
A©B- A is the parent of B
A%B- A is elder to B
A&B- A is younger to B
A$B- A is brother of B
A*B- A is wife of B
A#B- A is sister-in-law of B

8. If Y%H@J$U&K%Y, the age of H is 22 years and age of K is 33 years, so


what can be the age of Y?
(a)17 years (b)13 years (c)29 years
(d)40 years (e)36 years

9. If G©A@T#J*O$L@P©G then how is J related to A?


(a) Uncle (b)Brother-in-law (c) daughter-in-law
(d)Aunt (e) None of these

10. If G©A@T#J*O$L@P©G and P is the wife of U then how is P related to


T?
(a) Mother (b)Mother-in-law (c) sister-in-law
(d) daughter-in-law (e) None of these

Directions (11-12): Study the following information carefully and answer


the questions given below:
All the persons mentioned belong to one family. Z is the only sibling of Y,
who has only one daughter. M is married to N. O is grandchild of Y, who is
not in same generation as M. L is aunt of O. N is mother-in-law of V. V and Z
are sister-in-law of each other. S is married to R but is not a female.

11. How many married couples are there in the family?


(a) one (b) two (c) three
(d) four (e) can’t be determined

7 Adda247 Publications For any detail, mail us at


Publications@adda247.com
Cracker Book for Bank (IBPS | SBI | RRB PO | Clerk) Mains Exams

12. How many generations are there in the given family?


(a) one (b) two (c) three
(d) four (e) can’t be determined

Direction (13-14): Study the following information carefully and answer


the question given below-
There are nine members in the family. M is unmarried sibling of F. B is
sister-in-law of D. C is mother-in-law of E, who is parent of F. E has a
daughter and a son. H is the only child of F. B has a sister and a brother. The
maximum number of members are in the oldest generation. G is the
maternal grandmother of H. A is the unmarried.

13. Four of the five are alike in a certain way, who among the following
does not belongs to that group?
(a) A (b) B (c) C
(d) D (e) F

14. How is D related to M?


(a) uncle
(b) aunt
(c) Maternal grandfather
(d) Maternal grandmother
(e) None of these

Directions (15-16): Study the following information carefully and answer


the questions that follow:
Ten persons P, Q, R, S, T, U, V, W, X and Z are there in a family of three
generation. There are three married couple. S is the sister-in-law of R but
not married to P. P is the son of V. W is the brother of T. Z is the father-in-
law of Q. U is the mother of X and married to the brother of R. Q has only
one sister, who is unmarried. X has no sibling. Z has only one grandson and
no girl child. T is the youngest person of the family.

15. What is the ratio of male members to female members of the family
respectively?
(a) 5:4 (b) 1:1 (c) 3:2
(d) 2:3 (e) none of these
8 Adda247 Publications For any detail, mail us at
Publications@adda247.com
Cracker Book for Bank (IBPS | SBI | RRB PO | Clerk) Mains Exams

16. How is S related to T?


(a) mother (b) father (c) uncle
(d) aunt (e) none of these

Direction (17-18): Study the following information carefully and answer


the question below-

Eight persons K, L, M, N, O, P, Q, R are from three generations and four


married couple. R is the mother-in-law of O. P is married to the brother of
M. Q is the father-in-law of L. N is married to Q. K is the father of M. O is the
sister-in-law of L.

17. How is P related to M?


(a) Son-in-law (b) Son (c) sister -in-law
(d) Daughter (e) None of these

18. If J is the only child of L then how is N related to J?


(a) Maternal Grand mother
(b) mother
(c) Aunt
(d) Father
(e) None of these

Directions (19-20): Following questions are based on the information


given below.
‘P × Q’ means ‘P is father of Q’
‘P – Q’ means ‘P is sister of Q’
‘P + Q’ means ‘P is mother of Q’
‘P ÷ Q’ means ‘P is the brother of Q’

19. In the expression A × R –G + D ÷ N, how is D related to A?


(a) Granddaughter
(b) Son
(c) Grandson
(d) Granddaughter or Grandson
(e) None of these
9 Adda247 Publications For any detail, mail us at
Publications@adda247.com
Cracker Book for Bank (IBPS | SBI | RRB PO | Clerk) Mains Exams

20. Which of the following represents ‘Q’ is son of ‘U’?


(a) Q ÷ I – G × U (b) Q + I – G × U (c) Q ÷ M – N × U
(d) Q ÷ J – N + U (e) None of these

Directions (21-22): Following questions are based on the information


given below.
‘P × Q’ means ‘P is Son of Q’
‘P – Q’ means ‘P is sister of Q’
‘P + Q’ means ‘P is mother of Q’
‘P ÷ Q’ means ‘P is the brother of Q’

21. In the expression R ÷ Q + M ÷ N × G, how is N related to R?


(a) Granddaughter
(b) Son
(c) Niece
(d) Granddaughter or Grandson
(e) None of these

22. In which of the following expression Q is wife of Z?


(a) A–M÷Q+N×Z (b) A–Q÷M+N×Z (c) Q–M÷A+N×Z
(d) Q–R÷A÷N×Z (e) None of these

Directions (23-24): Following questions are based on the information


given below.
‘A × B’ means ‘A is Son of B’
‘A – B’ means ‘A is sister of B’
‘A + B’ means ‘A is mother of B’
‘A ÷ B’ means ‘A is the brother of B’

23. In the expression ‘R + L – S × M ÷ N’, how is N related to R?


(a) Brother (b) Brother-in-law (c) Sister
(d) Grandson (e) None of these

24. In which of the following expression N is brother-in-law of C?


(a) N÷D+R÷S×C (b) N÷D+C÷S×R (c) S÷D+R÷N×C
(d) M÷D+N÷S×C (e) None of these

10 Adda247 Publications For any detail, mail us at


Publications@adda247.com
Cracker Book for Bank (IBPS | SBI | RRB PO | Clerk) Mains Exams

25. A is father of X; B is mother of Y. The sister of X and Z is Y. Which of


the following statements is definitely not true?
(a) B is the mother of Z
(b) X is the sister of Z
(c) Y is the son of A
(d) B has one daughter
(e) B is the wife of A

26. Pointing to a woman, Meenu says to Ankit, “she is my mother who has
only two child’s and her only son is your maternal uncle.” In this
relation Meenu is Female. How is the narrator related to Ankit’s
father?
(a) Brother (b) Wife (c) Sister
(d) Mother (e) Father

Directions (27-29): These questions are based on the following


information:
‘A @ B’ means ‘A is father of B’.
‘A $ B’ means ‘A is mother of B’.
‘A * B’ means ‘A is sister of B’.
‘A # B’ means ‘A is daughter of B’.
‘A = B’ means ‘A is son of B’

27. How is D related to H, in the given expression


D@E=T$G*H
(a) Father (b) Mother (c) Sister
(d) Son (e) None of these

28. Which of the following statements is/are true, if the given expression
Q = W # V @ M = P is true?
(a) Q is father of M
(b) V is husband of P
(c) P is grandmother of Q
(d) Only (b) and (c) are true
(e) All are true

11 Adda247 Publications For any detail, mail us at


Publications@adda247.com
Cracker Book for Bank (IBPS | SBI | RRB PO | Clerk) Mains Exams

29. What is the relation between E and W in the given expression?


E@H$N*P=W
(a) E is father of W
(b) E is daughter of W
(c) E is father-in-law of W
(d) E is brother of W
(e) None of these

Directions (30-32): Each of these questions is based on the following


information:
There are seven members in a family Ayan, Brad, Axelsen, Rishi, Roman,
Andy, and Piyush among them only two are married couples. Brad is
daughter in law of Roman. Andy is grandmother of Rishi. Piyush is sister in
law of Roman. Ayan has two children Axelsen and Rishi. Andy and Piyush
are siblings. Brad has no son.

30. Who among the following is son of Piyush’s brother in law?


(a) Rishi (b) Axelsen (c) Ayan
(d) Brad (e) None of these

31. How is Axelsen related to Ayan’s mother?


(a) Grandson (b) Granddaughter (c) Son
(d) None of these (e) Daughter

32. How is Piyush related to Axelsen’s father?


(a) Maternal aunt (b) Paternal aunt (c) Mother
(d) None of these (e) Son

Direction (33-34): Read the following information carefully and answer


the questions given below:
Amit, Browni, Cendy, David, Julie, Feriah, Gautam, Henry, and Iram are
family members among them only six male members. Amit is married to
David. Each female has two sons and one daughter except one female. David
is grandmother of Iram who has no children. Browni is daughter of Amit,
who is married. Iram is daughter of Gautam. Henry and Feriah are brothers
and both are child of Gautam.
12 Adda247 Publications For any detail, mail us at
Publications@adda247.com
Cracker Book for Bank (IBPS | SBI | RRB PO | Clerk) Mains Exams

33. Who among the following is a brother-in-law of Gautam?


(a) Amit (b) Henry (c) Feriah
(d) Cendy (e) None of these

34. Who among the following is a grandson of David?


(a) Amit (b) Henry (c) Cendy
(d) Julie (e) None of these

Directions (35-36): Each of these questions is based on the following


information:
(i) A % B means A is the mother of B.
(ii) A @ B means A is the sister of B.
(iii) A $ B means A is the father of B.
(iv) A * B means A is the son of B.
(v) A + B means B is brother of A
35. If the expression F $ M@ N * G + P is definitely true, then which of the
following is true?
(a) P is sister in law of F
(b) P is sister of G
(c) P is brother of G
(d) None of these
(e) N is niece of P

36. If the expression S + Q $ T+ R * U is definitely true, then which of the


following is not true?
(a) Q is father of R
(b) R is nephew of S
(c) S is aunt of T
(d) None of these
(e) U is sister in law of S

Directions (37-39): These questions are based on the following


information. Study it carefully and answer the questions.
(i) ‘A × B’ means ‘A is father of B’
(ii) ‘A ÷ B’ means ‘A is daughter of B’
(iii) ‘A + B’ means ‘A is sister of B’
(iv) ‘A – B’ means ‘A is husband of B’
13 Adda247 Publications For any detail, mail us at
Publications@adda247.com
Cracker Book for Bank (IBPS | SBI | RRB PO | Clerk) Mains Exams

37. In F÷ R × H – L, how is H related to F ?


(a) Father (b) Brother (c) Sister
(d) cannot be determined (e) None of these

38. Which of the following indicates ‘N is mother of K’?


(a) K + L ÷ N × F (b) K + L ÷ N – M (c) H × K ÷ N
(d) N × F+ K (e) None of these

39. In F – R + H ÷ T, how is F related to T?


(a) Son-in-law (b) Daughter-in-law (c) Son
(d) Daughter (e) None of these

Direction (40): Study the following information to answer the given


question.
P + Q means P is the mother of Q.
P÷Q means P is the father of Q.
P×Q means P is the sister of Q.
P — Q means P is the brother of Q.

40. Which of the following should come in place of question mark in the
given expression to establish that A is the maternal grandfather of D?
‘A÷C×B ? D’
(a) + (b) — (c) ÷
(d) Either + or ÷ (e) None of these

Solutions

Direction (1-2):
1. (b);

And D is younger than both E and Q. So, the probable age of D is


23 yrs.
14 Adda247 Publications For any detail, mail us at
Publications@adda247.com
Cracker Book for Bank (IBPS | SBI | RRB PO | Clerk) Mains Exams

2. (a);

And V is younger than C.

Direction (3-5):

3. (c) 4. (d) 5. (c)

Directions (6-7):

6. (c); 7. (e);

Direction (8-10):
8. (c);

15 Adda247 Publications For any detail, mail us at


Publications@adda247.com
Cracker Book for Bank (IBPS | SBI | RRB PO | Clerk) Mains Exams

9. (d);

10. (b);

Directions (11-12):

11. (c); 12. (d);

Direction (13-14):

13. (e); 14. (c);

16 Adda247 Publications For any detail, mail us at


Publications@adda247.com
Cracker Book for Bank (IBPS | SBI | RRB PO | Clerk) Mains Exams

Direction (15-16):

15. (d); 16. (d);

Direction (17-18):

17. (c); 18. (a);

19. (c);

20. (e);

21. (e); After de-coding the given coded blood relation we get the
relation between N and R in which N is the nephew of R.

17 Adda247 Publications For any detail, mail us at


Publications@adda247.com
Cracker Book for Bank (IBPS | SBI | RRB PO | Clerk) Mains Exams

22. (a); From Expression 1 we get our final answer and deduce the
blood relation given below:

Directions (23-24):
23. (e);

24. (a);

25. (c);

26. (b);

27. (a);

18 Adda247 Publications For any detail, mail us at


Publications@adda247.com
Cracker Book for Bank (IBPS | SBI | RRB PO | Clerk) Mains Exams

28. (d);

29. (c);

Solutions (30-32):

30. (c); 31. (b); 32. (a);

Direction (33-34):

33. (d); 34. (b);


19 Adda247 Publications For any detail, mail us at
Publications@adda247.com
Cracker Book for Bank (IBPS | SBI | RRB PO | Clerk) Mains Exams

Directions (35-36):
35. (c);

36. (c);

Directions (37-39):
37. (b);

38. (c);

39. (a);

40. (a);

 Key Points to Remember for New Pattern Questions


➢ A Blood relation tree is nothing but a tree having its
branches which is considered as the different
generation. And the fruits or leaves of this tree are the
persons or members of that family.
20 Adda247 Publications For any detail, mail us at
Publications@adda247.com
Cracker Book for Bank (IBPS | SBI | RRB PO | Clerk) Mains Exams

➢ In blood relation the new concept that has been seen in


the question is the coded form of blood relation. It
consists of the codes for the different relations and we
have to decode the relations first then draw the tree.
➢ A few changes that has been seen is the blood relation
consist of different years of births of the family member
as it forms a puzzle. In this both the years of birth and
blood relation have to be solved simultaneously.
➢ The blood relation concept has also been comprised in
the seating arrangement in which the blood relation
plays an important role and the arrangement cannot be
solved without solving the blood relation.
➢ In some cases, the relations are already defined in the
puzzle and you have to arrange the persons in that
relation accordingly.

21 Adda247 Publications For any detail, mail us at


Publications@adda247.com
Cracker Book for Bank (IBPS | SBI | RRB PO | Clerk) Mains Exams

1 Adda247 Publications For any detail, mail us at


Publications@adda247.com
Cracker Book for Bank (IBPS | SBI | RRB PO | Clerk) Mains Exams

Chapter

11 Miscellaneous

Practice Exercise Based on new Pattern

Directions (1-5): Study the following alphanumeric series carefully and


answer the questions given below:
7 A 6 P & R $ 4 Y Q % T @ 3 9 S I O 9 9 J L E U *
K # 3

STEP I- The letters which are immediately preceded and immediately


followed by a symbol are arranged in the end of the series in the
alphabetical order. (They are arranged just after 3)

STEP II- The numbers which are immediately preceded by the letter and
immediately followed by the Symbol are arranged between 9 and S in the
increasing order.

STEP III- The numbers which are immediately followed by letter are
interchanged its position with respect to the element just after it.
(STEP II is applied after STEP I and STEP III is applied after STEP II)

1. How many letters are arranged at the end of the series in the step-1?
(a) one (b) Three (c) Four
(d) Five (e) More than five

2. Which among the following are the elements of the series which are
second position from the left end and fifth position from the right end
in step-III?
(a) 63 (b) 7# (c) P#
(d) AK (e) 6#

3. How many symbols are immediately followed by numbers in step-III?


(a) one (b) Three (c) Four
(d) Five (e) Two
2 Adda247 Publications For any detail, mail us at
Publications@adda247.com
Cracker Book for Bank (IBPS | SBI | RRB PO | Clerk) Mains Exams

4. Which of the following element is third to the left of the seventh


element from the right in step III?
(a) E (b) J (c) 9
(d) L (e) None of these

5. Which of the following is the third letter from the right end in step II?
(a) R (b) S (c) T
(d) K (e) None of these

Direction (6-10): In every question two rows are given and to find out the
resultant of a particular row you need to follow the following steps: -

Step 1: If an odd number is followed by a perfect square then the resultant


will be the subtraction of the square number from the odd number.

Step 2: If an even number is followed by an odd (prime) number then the


resultant will be the addition of both the numbers.

Step 3: If an even number is followed by an even number then the resultant


will be the difference of both the numbers.

Step 4: If an odd number is followed by another odd number then the


resultant will be the addition of both the numbers.

Step 5: If an even number is followed by an odd (non-prime) number


except (1) then the resultant will be the subtraction of the odd number
from the even number.

Step 6: If an odd number is followed by an even number then the resultant


comes by multiplying the numbers.

6. 8 7 6
11 9 5
Find the difference of two rows
(a) 35 (b) 83 (c) 31
(d) 63 (e) None of these

3 Adda247 Publications For any detail, mail us at


Publications@adda247.com
Cracker Book for Bank (IBPS | SBI | RRB PO | Clerk) Mains Exams

7. Find the resultant of second row if M is the resultant of first row.


3 7 9
M 12 8
(a) 11 (b) 20 (c) 4
(d) 3 (e) None of these

8. If the sum of the resultants of two rows is 275. Then find the value of
K?
15 20 42
9 K 5
(a) 4 (b) 9 (c) 3
(d) 5 (e) None of these

9. Find the multiple of the resultant of first and second row.


5 4 15
21 16 4
(a) 36 (b) 16 (c) 10
(d) 13 (e) None of these

10. If Q is the resultant of second row, then find the difference of the
resultant of two rows.
9 8 Q
12 9 4
(a) 68 (b) 72 (c) 73
(d) 71 (e) None of these

Direction (11-15): In every question two rows are given and to find out
the resultant of a particular row you need to follow the following steps: -
Note: (All the resultant value consider as a positive integer)

Step 1: If an odd number is followed by an even number then the resultant


will be the subtraction of both the numbers.

Step 2: If an even number is followed by a perfect cube then the resultant


will be the sum of the numbers.

4 Adda247 Publications For any detail, mail us at


Publications@adda247.com
Cracker Book for Bank (IBPS | SBI | RRB PO | Clerk) Mains Exams

Step 3: If an odd number is followed by another odd number then the


resultant will be the subtraction of both the numbers.

Step 4: If an even number is followed by an odd (prime) number then the


resultant will be the subtraction of both the number.

Step 5: If above four conditions are not applied than simply add both the
number.

11. Find the resultant sum of two rows?


8 27 2
15 7 3
(a) 18 (b) 25 (c) 35
(d) 38 (e) None of the above

12. Find the resultant of second row if D is the resultant of first row.
9 7 2
13 D 3
(a) 5 (b) 7 (c) 9
(d) 3 (e) 6

13. If the resultants value of second rows is 7, then find the value of L?
24 27 12
12 3 L
(a) 12 (b) 16 (c) 15
(d) 9 (e) None of the above

14. Find the multiplication of the resultant value of first and second row.
4 8 8
27 6 7
(a) 441 (b) 156 (c) 625
(d) 400 (e) 280

5 Adda247 Publications For any detail, mail us at


Publications@adda247.com
Cracker Book for Bank (IBPS | SBI | RRB PO | Clerk) Mains Exams

15. If Z is the resultant of second row, then find the sum of the resultant of
two rows.
Z 15 36
6 512 100
(a) 633 (b) 518 (c) 618
(d) 597 (e) 1215

Directions (16-20): Study the following alphanumeric series carefully and


answer the questions given below:
12AIFR 42 WBOL 63EOCN
15TVMR 75GYTR

16. How many meaningful words can be formed from the letters of the
words which are attached with the numbers in which At least one
digit is even digit in the given alphanumeric series?
(a) one (b) two (c) three
(d) four (e) More than four

17. The words are arranged according to the descending order of the
numbers which are attached to them from left to right, then how
many alphabets are between the letters of second letter from the left
end and seventh letter from the right end?
(a) one (b) two (c) three
(d) four (e) More than five

18. The letters which are attached with the numbers in which at least one
is odd digit are arranged according to the ascending orders of their
numbers from left to right and then all letters of the words together
are arranged in the alphabetical order, then which of the following is
the 9th letter from the right end?
(a) M (b) O (c) N
(d) R (e) None of these

19. How many meaningful words can be formed from the letters of the
words which are attached with the numbers in which both the digits
are even digit in the given alphanumeric series?
(a) Three (b) Two (c) Four
(d) One (e) None of these
6 Adda247 Publications For any detail, mail us at
Publications@adda247.com
Cracker Book for Bank (IBPS | SBI | RRB PO | Clerk) Mains Exams

20. How many meaningful words can be formed from the letters of the
words which are attached with the numbers in which both the digit
are odd digit in the given alphanumeric series??
(a) Three (b) Two (c) Four
(d) None (e) None of these

Direction (21-25): In every question two rows are given and to find out
the resultant of a particular row you need to follow the following steps: -

Condition I: If an odd number is followed by a perfect square then the


resultant will be the multiplication of both the numbers.

Condition II: If an even number is followed by an odd (prime) number then


the resultant will be the division of both the numbers.

Condition III: If an even number is followed by an even number then the


resultant will be the difference of both the numbers.

Condition IV: If an odd number is followed by another odd number then


the resultant will be the addition of both the numbers.

Condition V: If an even number is followed by an odd (non-prime) then the


resultant will be the subtraction of both the numbers.

Note: Resultant value is always considered as positive integer. And Greater


number is dividing by the smaller number.

21. 12 8 8
7 16 5
Find the sum of two rows?
(a) 135 (b) 133.4 (c) 134.5
(d) 134.4 (e) None of these

22. Find the value of the multiplication of both row’s resultant value than
addition of 1.
12 27 5
1 3 23
(a) 116 (b) 120 (c) 114
(d) 113 (e) None of these
7 Adda247 Publications For any detail, mail us at
Publications@adda247.com
Cracker Book for Bank (IBPS | SBI | RRB PO | Clerk) Mains Exams

23. If the multiplication of the resultants of two rows is 84. Then find the
value of X?
12 9 4
4 X 9
(a) 4 (b) 9 (c) 3
(d) 5 (e) None of these

24. Find the division of the sum of the resultant of first and second row.
30 5 2
17 11 12
(a) 4 (b) 2 (c) 8
(d) 6 (e) None of these

25. If X is the resultant of the first and second digit of the second row,
then find the sum of the resultant of two rows.
24 9 X
7 9 3
(a) 142 (b) 144 (c) 154
(d) 152 (e) None of these

Direction (26-30): There are two rows given and to find out the resultant
of a particular row we need to follow the following steps: -

Step 1: If an even number is followed by an odd number then the resultant


will be the addition of both the numbers.

Step 2: If an odd number is followed by a perfect square then the resultant


will be the difference of that square number and the odd number.

Step 3: If an odd number is followed by another odd number (but not a


perfect square) then the resultant will be the addition of both the numbers.

Step 4: If an odd number is followed by an even number (but not a perfect


square) then the resultant comes by multiplying the numbers.

Step 5: If an even number is followed by another even number then the


resultant will be the division of first number by the second number.
8 Adda247 Publications For any detail, mail us at
Publications@adda247.com
Cracker Book for Bank (IBPS | SBI | RRB PO | Clerk) Mains Exams

26. Find the sum of two rows


8 4 1
11 6 7
(a) 78 (b) 52 (c) 64
(d) 76 (e) None of the above

27. If the sum of the resultants of two rows is 46. Then find the value of X.
9 2 7
24 4 X
(a) 16 (b) 27 (c) 8
(d) 15 (e) None of the above

28. Find the difference between the resultant of first and second row.
13 3 7
4 11 12
(a) 117 (b) 126 (c) 157
(d) 96 (e) None of the above

29. Find the multiplication of the resultant of first and second row.
21 19 8
16 13 9
(a) 110 (b) 85 (c) 100
(d) 120 (e) None of the above

30. If the multiplication of the resultant of first and second row is 39, then
find the value of ‘X’.
7 5 4
6 2 X
(a) 11 (b) 15 (c) 16
(d) 12 (e) None of the above

9 Adda247 Publications For any detail, mail us at


Publications@adda247.com
Cracker Book for Bank (IBPS | SBI | RRB PO | Clerk) Mains Exams

Solutions

Solution (1-5):
Input :
7 A 6 P & R $ 4 Y Q % T @ 3 9 S I O 9 9 J L E U
* K # 3

Step 1:
7 A 6 P & $ 4 Y Q % @ 3 9 S I O 9 9 J L E U * #
3 K T R

Step 2:
7 A 6 P & $ 4 Y Q % @ 3 9 S I O 9 9 J L E U * #
3 K T R

Step 3:
A 7 P 6 & $ Y 4 Q % @ 3 S 9 I O 9 J 9 L E U * # K
3 T R

1. (b); 2. (b); 3. (a);


4. (c); 5. (d);

Solutions (6-10):
6. (b); In row-1
Even number is followed by an odd prime number so=8+7= 15
15 6
Odd number is followed by an even number so=15*6= 90
In row-2
Odd number is followed by a perfect square so=11-9= 2
2 5
Even number is followed by an odd prime number so=2+5= 7
So the difference of both row = 90 - 7 = 83
10 Adda247 Publications For any detail, mail us at
Publications@adda247.com
Cracker Book for Bank (IBPS | SBI | RRB PO | Clerk) Mains Exams

7. (c); In row-1
Odd number is followed by another odd number so=3+7= 10
10 9
Even number is followed by an odd (non-prime) number so=10-
9= 1
From row-1 M value is 1 so,
In row-2
Odd number is followed by an even number so=1*12= 12
12 8
Even number is followed by an even number so=12-8= 4
So the resultant of the second row = 4

8. (c); In row-1
Odd number is followed by an even number so=15*20= 300
300 42
Even number is followed by an even number so=300-42= 258
In row-2
When put K=3 then,
3 5
Odd number is followed by another odd number so=9+3= 12
12 5
Even number is followed by an odd (prime) number so=12+5=
17
So the Sum of the resultant of the both rows = 275
So this condition can satisfy only when K=3.

9. (b); In row-1
Odd number is followed by a perfect square so=5-4= 1
1 15
Odd number is followed by another odd number so=1+15= 16
In row-2
Odd number is followed by a perfect square so=21-16= 5
5 4
Odd number is followed by a perfect square so=5-4 = 1
So the multiple of the resultant of the both rows = 16*1= 16

11 Adda247 Publications For any detail, mail us at


Publications@adda247.com
Cracker Book for Bank (IBPS | SBI | RRB PO | Clerk) Mains Exams

10. (b); In row-2


Even number is followed by an odd (non-prime) number so=12-
9= 3
3 4
Odd number is followed by a perfect square so=3-4= (-1)
So the value of Q= (-1)
In row-1
Odd number is followed by an even number so=9*8= 72
72 (-1)
Even number is followed by an odd (prime) number so=72+ (-1)
= 71
So difference of the resultant of two rows = 71-(-1) = 72

Solution (11-15):
11. (d); Row-I: Even number is followed by a perfect cube= 8+27=35
Now 35 2 odd number is followed by an even number=35-2=33
Row-II: Odd number is followed by another odd=15-7=8
8 3 even number is followed by an odd (prime) number=8-3=5
Sum of both the row= 33+5=38

12. (e); Row-I: odd number is followed by another odd number= 9-7=2
2 2 above four conditions are not applied=2+2=4
Row-II: odd number is followed by another even number=13-
4(D)=9
9 3 odd number is followed by another odd number=9-3=6

13. (b); Row-I: even number is followed by an odd (prime) number=3


3 12 odd number is followed by an even number= 12-3=9
Row-II: even number is followed by an odd (prime)
number=12-3=9
9 L=16 odd number is followed by an even number number 16-
9=7

12 Adda247 Publications For any detail, mail us at


Publications@adda247.com
Cracker Book for Bank (IBPS | SBI | RRB PO | Clerk) Mains Exams

14. (e); Row-I: even number is followed by a perfect cube 4+8=12


12 8 even number is followed by a perfect cube 12+8=20
Row-II: odd number is followed by an even number 27-6=21
21 7 odd number is followed by another odd number=21-7=14
Multiplication= 20*14=280

15. (e); Row-II: even number is followed by a perfect cube= 512+6=518


518 100 above four conditions are not applied=518+100=618
Row-I: Z=618
Above four conditions are not applied = 618 + 15 = 633
Odd number is followed by an even number=633-36=597
Resultant of two row= 618+597=1215

Solutions (16-20):
16. (b); There are Five meaningful words- Fair, Bowl, Blow, Cone, Once

17. (b);

18. (c);

19. (b); There are two words can be made ‘Bowl and Blow’.

20. (d); There are two such words ‘TVMR and GYTR’.
But no meaningful words can be formed.

Solutions (21-25):
21. (e); Row-I: Even number is followed by an even number= 12 – 8 = 4
Even number is followed by an even number = 8 – 4 = 4
Row-II: odd number is followed by a perfect square= 7*16=112
Even number is followed by an odd (prime)
number=112/5=22.4
Sum of both the row= 26.4

22. (a); Row-I: even number is followed by an odd (non-prime) =27-


12=15

13 Adda247 Publications For any detail, mail us at


Publications@adda247.com
Cracker Book for Bank (IBPS | SBI | RRB PO | Clerk) Mains Exams

Odd number is followed by another odd number= 15+5=20


Row-II: odd number is followed by another odd number= 1+3=4
Even number is followed by an odd (prime) number=23/4=5.75
Multiplication of both row’s resultant value than addition of 1=
5.75*20=115+1=116

23. (e); Row-I: even number is followed by an odd (non-prime) = 12-


9=3
Odd number is followed by a perfect square = 3*4=12
Row-II: Even number is followed by an even number = 6-4=2
Even number is followed by an odd (non-prime) = 9-2=7
Multiplication of both the row’s resultant value = 12*7 = 84

24. (a); Row-I: even number is followed by an odd (prime) number =


30/5=6
Even number is followed by an even number = 6-2=4
Row-II: odd number is followed by another odd number =
17+11= 28
Even number is followed by an even number = 28-12 = 16
Division= 16/4 = 4

25. (b); Row-II: odd number is followed by a perfect square = 7*9 = 63


Odd number is followed by another odd number = 63+3=66
Row-I: Even number is followed by an odd (non-prime) = 24-
9=15
Odd number is followed by another odd number = 15+63 = 78
Sum of the resultant of two rows = 78+66= 144

Solutions (26-30):
26. (d); Row-I: even number is followed by an even number = 8/4=2
Even number is followed by an odd number = 2+1=3
Row-II: odd number is followed by even number = 11*6= 66
Even number is followed by an odd number = 66+7 = 73
Sum of two rows= 73+3= 76

14 Adda247 Publications For any detail, mail us at


Publications@adda247.com
Cracker Book for Bank (IBPS | SBI | RRB PO | Clerk) Mains Exams

27. (d); Row-I: odd number is followed by an even number = 9*2= 18


Even number is followed by an odd number = 18+7=25
Row-II: even number is followed by another even number =
24/4= 6
Difference of total and first row is = 46-25= 21
The resultant of second row is 21
Hence, if X= 15 then,
Even number followed by odd number= 6+15= 21.

28. (c); Row-I: odd number is followed by an odd (not perfect square)
number = 13+3= 16
Even number is followed by an odd number = 16+7= 23
Row-II: even number is followed by odd number = 4+11= 15
odd number is followed by an even number = 15*12 = 180
The difference of the resultants= 180-23= 157

29. (c); Row-I: odd number is followed by an odd number = 21+19 = 40


Even number is followed by an even number = 40/8 = 5
Row-II: even number is followed by another odd number =
16+13= 29
odd number is followed by an odd (perfect square) number =
29-9 = 20
Multiplication= 20*5 = 100

30. (c); Row-I: odd number is followed by an odd number = 7+5= 12


Even number is followed by an even number = 12/4=3
Resultant of both rows is 39 i.e. 3*13= 39 (resultant of second
row is 13).
Row-II: even number is followed by another even number =
6/2= 3
Odd number is followed by an even (perfect square) number =
16-3 = 13
So, X= 16

15 Adda247 Publications For any detail, mail us at


Publications@adda247.com

You might also like